NCERT Books

MCQ Questions on World Geography Miscellaneous | Practice Miscellaneous World Geography GK Quiz Questions with Answers

MCQ Questions on World Geography Miscellaneous

Are you looking over the internet for instant help on the world geography miscellaneous topic? Then, this is the right page for you. Here, we have shared our best collection of MCQ Questions on World Geography Miscellaneous to clarify your queries and crack all your competitive exams. Provided most important objective-type questions on Miscellaneous world geography will help you stand at the top in scoring the highest marks in the exams.

World Geography Miscellaneous Multiple Choice Questions & Answers

1. The Radcliff Line demarcates the boundary between
A. India and China
B. India and Pakistan
C. Pakistan and Afghanistan
D. India and Afghanistan

Answer

Answer: Option B
Explanation : The Radcliff Line demarcates the boundary between India and Pakistan. The demarcation line was published on 17 August 1947 upon the Partition of India.


2. World’s highest urban density of population occurs in ________
A. Kolkata
B. Tokyo
C. New York
D. London

Answer

Answer: Option B
Explanation : World’s highest urban density of population occurs in Tokyo. As has been the case for nearly six decades, Tokyo remains the largest urban area in the world, with approximately 37 million.


3. International date line is a/an
A. Straight line
B. Curved line
C. Zigzag line
D. Elliptical line

Answer

Answer: Option C
Explanation : International date line is a Zigzag line. The International Date Line is an imaginary zig zag line on the surface of the earth roughly following the 180 degree meridian. Crossing the date line causes the date to be changed exactly by one day. Crossing the line from East to West , one loses a day . And from West to East, one gains a day.


4. The latitude also known as the
A. Tropic of Capricorn
B. Tropic of Cancer
C. Antarctic Circle
D. Arctic Circle

Answer

Answer: Option D
Explanation : The latitude also known as the Arctic Circle. The Arctic Circle is the parallel of latitude that runs 66° 33´ 39″ (or 66.56083°) north of the Equator and is one of the five major circles of latitude that mark maps of the Earth. Everything north of this circle is known as the Arctic, and the zone just to the south of this circle is the Northern Temperate Zone.


5. Boll-Weevil is a pest that attack the ________ crop.
A. Jute
B. Cotton
C. Hemp
D. Flax

Answer

Answer: Option B
Explanation : Boll-Weevil is a pest that attack the Cotton crop. The weevil is a pest which primarily attacks flowers and bolls. A boll weevil can destroy the cotton lint with its feces and consume the cotton seeds.


6. What is a graticule?
A. The network of intersecting parallels and meridians on a map
B. The network of intersections of Equator and Prime Meridian
C. The intersection of roads
D. The intersection of Tropic of Cancer and Tropic of Capricorn

Answer

Answer: Option A
Explanation : The network of intersecting parallels and meridians on a map is known as graticule. The graticule of latitude and longitude lines is an angular measurement system. All features on the earth’s surface are located using measurements that are relative to the center of the earth. Latitude lines are parallel to each other while longitude lines converge at the poles.


7. The years 1700, 1800 and 1900 were
A. Leap years
B. Not leap years
C. Famine years
D. Years of natural calamities

Answer

Answer: Option B
Explanation : The years 1700, 1800 and 1900 were not leap years. A normal year is defined as 365 days. Putting all of these rules together, you can see that a year is a leap year not only if it is divisible by 4 — it also has to be divisible by 400 if it is a centurial year. So 1700, 1800 and 1900 were not leap years.


8. White coal refers to
A. Hydroelectricity
B. Petroleum
C. Uranium
D. Coal

Answer

Answer: Option A
Explanation : White coal refers to Hydroelectricity. White coal was used in England to smelt lead ore from the mid-16th to the late 17th centuries. It produces more heat than green wood but less than charcoal and thus prevents lead evaporating. White coal was produced in distinctive circular pits with a channel, known as Q-pits.


9. Which country is known as the “Emerald Isle”?
A. Great Britain
B. Ireland
C. Japan
D. New Zealand

Answer

Answer: Option B
Explanation : Ireland is known as the “Emerald Isle”. Ireland’s beautiful, lush countryside is what earned the country the nickname, “The Emerald Isle.” The island is located in the Northern Atlantic Ocean, just west of Great Britain. Ireland’s only bordering country is Northern Ireland.


10. ‘Recycling’ is a method for conserving the resources of
A. Coal
B. Oil
C. Copper
D. Limestone

Answer

Answer: Option C
Explanation : ‘Recycling’ is a method for conserving the resources of Copper. To date only about 12% of known copper resources have been mined. However copper ore is a finite resource and it makes sense to conserve ore by recycling.


11. The moat abundant element in sea water is
A. Sodium
B. Chlorine
C. Magnesium
D. Sulphur

Answer

Answer: Option B
Explanation : The moat abundant element in sea water is Chlorine. Seawater contains 19,000 mg of chlorine per liter.


12. Which among these is a form of precipitation?
A. Dew
B. Fog
C. Mist
D. Hail

Answer

Answer: Option D
Explanation : Hail is a form of precipitation. Hail is a form of precipitation that occurs when updrafts in thunderstorms carry raindrops upward into extremely cold areas of the atmosphere where they freeze into ice.


13. The tropical rain forests in the Amazon basin is known as
A. Savanna
B. Prairies
C. Selvas
D. Pampas

Answer

Answer: Option C
Explanation : The tropical rain forests in the Amazon basin is known as Selvas. The Amazon rainforest, covering much of northwestern Brazil and extending into Colombia, Peru and other South American countries, is the world’s largest tropical rainforest, famed for its biodiversity. It’s crisscrossed by thousands of rivers, including the powerful Amazon.


14. The depositional feature at the point where the river comes down from hills to plain is
A. Meander
B. Water fall
C. Levee
D. Alluvial fan

Answer

Answer: Option D
Explanation : The depositional feature at the point where the river comes down from hills to plain is Alluvial fan. They are found in the middle course of a river at the foot of slope/ mountains. When the stream moves from the higher level break into foot slope plain of low gradient, it loses its energy needed to transport much of its load.


15. In which of the following countries, is the ratio of hydroelectric generation to the total electricity generation the highest?
A. Norway
B. Sweden
C. Italy
D. Denmark

Answer

Answer: Option A
Explanation : In Norway the ratio of hydroelectric generation to the total electricity generation the highest. Norway is one of the world’s leading countries for renewable energy, and most of it comes from the water.


16. Horizontal transfer of heat is known as
A. Advection
B. Convection
C. Conduction
D. Elliptical line

Answer

Answer: Option A
Explanation : Horizontal transfer of heat is known as Advection. This type of heat transfer is not powered by variations in density, but rather requires an outside force, such as wind or currents, to displace the particles of the medium.


17. The Trans-Siberian railway links
A. The Baltic coast with the North Sea
B. Leningrad with Oldiotsk
C. Buanos Aires with Valparaiso
D. Leningrad with Vladivostok

Answer

Answer: Option D
Explanation : The Trans-Siberian railway links Leningrad with Vladivostok. The Trans-Siberian rail tour is one of the most interesting adventures anyone can take and Russian Trains.


18. What is the average salt content in a litre of seawater?
A. 39 gm.
B. 37 gm.
C. 35 gm.
D. 32 gm.

Answer

Answer: Option C
Explanation : The average salt content is 35 gm. in a litre of seawater. This means that for every 1 litre (1000 mL) of seawater there are 35 grams of salts (mostly, but not entirely, sodium chloride) dissolved in it.


19. The USA is deficient in domestic supplies of
A. Copper
B. Lead
C. Zinc
D. Manganese

Answer

Answer: Option D
Explanation : The USA is deficient in domestic supplies of Manganese.


20. Which among these is an intermontane plateau?
A. Deccan
B. Anatolia
C. Columbia
D. Tibet

Answer

Answer: Option D
Explanation : A plateau which is enclosed or surrounded by mountain ranges is known as an intermontane plateau. The Plateau of Tibet and the Plateau of Mongolia are the two intermontane plateaus in Asia. The Plateau of Tibet is surrounded by the Kunlun Mountains in the north and the Himalayas in the south.


21. The greatest diversity of animal and plant species occurs
A. Temperate deciduous forests
B. Tropical moist forests
C. Heavily polluted rivers
D. Deserts and savannas

Answer

Answer: Option B
Explanation : The greatest diversity of animal and plant species occurs in Tropical moist forests. The tropical rainforests contain the greatest diversity of species of all biomes on earth. They are found around the equator, between 23.5 degrees N latitude and 23.5 degrees S latitude.


22. When humidity is expressed in terms of percentage then it is known as
A. Absolute humidity
B. Specific humidity
C. Relative humidity
D. Ratio humidity

Answer

Answer: Option C
Explanation : When humidity is expressed in terms of percentage then it is known as Relative humidity. Relative humidity is expressed as a percentage. The relative humidity is 100% if the air is saturated with water vapor and 0% if no water vapor is present in the air at all.


23. The snowline is the line on mountain or hill slope which
A. Is the highest limit of perpetual snow .
B. Is the lowest limit of perpetual snow
C. Represents the highest amount of snowfall received
D. Represents the lowest amount of snowfall received

Answer

Answer: Option B
Explanation : The snowline is the line on mountain or hill slope which is the lowest limit of perpetual snow. The Imaginary line on a mountain or a hill which forms the lower limit of perpetual snow is called snow line. Below this line the snow melts away and above this line snow never melts. The snow line does not exist at the same level everywhere on earth. The average altitude of snow line is 5500m in the equatorial region, 4000-5000m in the Himalayan region, 2700m in the Alps and 0m in the polar regions.


24. The world’s largest inland sea is ________.
A. The Black Sea
B. The Dead Sea
C. The Caspian Sea
D. The Aral Sea

Answer

Answer: Option C
Explanation : The Caspian Sea is the world’s largest inland water body. The sea covers roughly 143,200 square miles (371,000 square kilometers) and borders five countries: Iran, Russia, Kazakhstan, Turkmenistan and Azerbaijan.


25. A geographical mile is a measure of length equal to one sixtieth of a degree or one minute (1′) of latitude. It varies with latitudes, but is approximately equals
A. 7200 feet
B. 6080 feet
C. 4500 feet
D. 6000 feet

Answer

Answer: Option B
Explanation : A geographical mile is a measure of length equal to one sixtieth of a degree or one minute (1′) of latitude. It varies with latitudes, but is approximately equals to 6080 feet.


26. Albert Canal links ________ and ________
A. Manchester and Antwerp
B. Antwerp and Frankfurt
C. Antwerp and Liege
D. Manchester and Liege

Answer

Answer: Option C
Explanation : Albert Canal links Antwerp and Liege. Albert Canal, waterway connecting the cities of Antwerp and Liège in Belgium. The Albert Canal is about 130 km (80 miles) long.


27. What is the usual altitude of the plains?
A. Sea level
B. Less than 500 metres above sea level
C. Less than 600 metres above sea level
D. Less than 800 metres above sea level

Answer

Answer: Option B
Explanation : The usual altitude of the plains less than 500 metres above sea level.


28. Outwash plain is a depositional feature associated with
A. Rivers
B. Wind
C. Glacier
D. Wave

Answer

Answer: Option C
Explanation : Outwash plain is a depositional feature associated with Glacier. It is a plain formed of glacial sediments deposited by meltwater outwash at the terminus of a glacier. As it flows, the glacier grinds the underlying rock surface and carries the debris along.


29. Strait of Malacca separates ________
A. Sumatra and Malaysia
B. Java and Brunei
C. Sumatra and Java
D. Malaysia and Brunei

Answer

Answer: Option A
Explanation : Strait of Malacca separates Sumatra and Malaysia. The Strait of Malacca (also called the Straits of Malacca, and in Malay Selat Melaka) is a narrow stretch of water between Peninsular Malaysia (West Malaysia) and the Indonesian island of Sumatra.


30. Ladang refers to
A. Shifting cultivation
B. Plantation agriculture
C. Subsistence type of agriculture
D. Dry farming

Answer

Answer: Option A
Explanation : Ladang refers to Shifting cultivation. Shifting cultivation is an agricultural system in which areas of land are cultivated for a short time. Then they are left to grow back their natural vegetation, while the farmer moves to another area.


31. Fish farming is practised in
A. Africa
B. South Africa
C. China
D. Peru and Brazil

Answer

Answer: Option C
Explanation : Fish farming is practised in China. China, with one-fifth of the world’s population, accounts for two-thirds of the world’s reported aquaculture production. Aquaculture is the farming of fish and other aquatic life in enclosures, such as ponds, lakes and tanks, or cages in rivers and coastal waters.


32. Copper alloys with tin to give
A. Brass
B. Bronze
C. German silver
D. Zinc

Answer

Answer: Option B
Explanation : Copper alloys with tin to give Bronze. Bronze is an alloy that consists primarily of copper with the addition of other ingredients. In most cases the ingredient added is typically tin, but arsenic, phosphorus, aluminum, manganese, and silicon can also be used to produce different properties in the material.


33. The international air route from West Europe to East Asia passes through India because ________
A. India is a busy centre situated between the two places
B. It provides the shortest route
C. India is the only country in Asia providing refuelling capacity
D. None of these

Answer

Answer: Option A
Explanation : The international air route from West Europe to East Asia passes through India because India is a busy centre situated between the two places.


34. The approximate depth of the Mariana Trench or the Pacific Ocean is
A. 5530 m
B. 7530 m
C. 11030 m
D. 13030 m

Answer

Answer: Option C
Explanation : The approximate depth of the Mariana Trench or the Pacific Ocean is 11030 m. The maximum known depth is 10,994 metres (36,070 ft) (± 40 metres [130 ft]) at the southern end of a small slot-shaped valley in its floor known as the Challenger Deep.


35. Which one the following condition is not favourable for the formation of delta?
A. Active vertical and lateral erosion in the lower course of the rivers
B. Coast should be preferably tideless
C. Sea adjoining the delta should be shallow
D. There should be no strong current running at right angles to the river mouths

Answer

Answer: Option A
Explanation : Active vertical and lateral erosion in the lower course of the rivers is not favourable for the formation of delta. Favourable conditions are Active vertical & lateral erosion in upper course of the river to provide extensive sediments. Coast should be sheltered preferably tideless & no strong current at right angle to the mouth of the river washing away the sediments. Sea adjoining the delta should be shallow or else the load will disappear in deep waters.


36. Wavy hair is characteristic or
A. Neanderthal man
B. Nordic race
C. Mongoloid race
D. Cromagnon man

Answer

Answer: Option B
Explanation : Wavy hair is characteristic of Nordic race. The Nordic race was one of the putative sub-races into which some late-19th to mid-20th century anthropologists divided the Caucasian race.


37. In the Himalayas the melting or the snow is more on the
A. Southern slopes
B. Northern slopes
C. Eastern slopes
D. Western slopes

Answer

Answer: Option A
Explanation : In the Himalayas the melting or the snow is more on the Southern slopes. In winter, the sun is always towards the South and the northern facing parts of any mountain are in perpetual shadow. However, in summer, the sun will not migrate as much to the North as it migrated to the South in winter.


38. The Doctor is a wind which
A. Is enervating
B. Withers plants
C. Is injurious to health
D. Is conducive to health

Answer

Answer: Option D
Explanation : The Doctor is a wind which is conducive to health. The Harmattan Wind is also known as “the Doctor”, as it’s cool winds can bring relief from oppressive heat.


39. Which of the following gives the correct decreasing order in numbers in the food chain pyramid?,1. Primary Producers,2. Herbivores,3. Carnivores,4. Man
A. 1 2 3 4
B. 1 3 2 4
C. 4 3 2 1
D. 1 2 4 3

Answer

Answer: Option A
Explanation : The correct decreasing order in numbers in the food chain pyramid are Primary Producers, Herbivores, Carnivores and Man.


40. The World’s biggest concrete structure “Three Gorges Dam” is located in which of the following countries?
A. China
B. Taiwan
C. Malaysia
D. Thailand

Answer

Answer: Option A
Explanation : “Three Gorges Dam” is a hydroelectric dam built across Yangtze River in Hubei Province of China.


41. The trees of spruce, fir, pine, etc are found in which of the following natural regions of the Earth?
A. Siberian Type
B. Tundra Type
C. Taiga Type
D. Laurentian Type

Answer

Answer: Option C
Explanation : The trees of spruce, fir, pine, etc are found in Taiga Type natural regions of the Earth. Taigas are thick forests. Coniferous trees, such as spruce, pine, and fir, are common. Coniferous trees have needles instead of broad leaves, and their seeds grow inside protective, woody cones.


42. India is developing Chabahar Port in which of the following countries?
A. Iraq
B. Oman
C. Iran
D. Afghanistan

Answer

Answer: Option C
Explanation : As the counter measure to the Gwadar port being developed by rival nations China & Pakistan in Pakistan, India is developing the strategically important Chabahar port in Iran. This port will provide India an access to Central Asian countries & Afghanistan.


43. You find some local people living in log cabins in
A. Tropical rainforest
B. Taiga regions
C. Deserts
D. Mediterranean regions

Answer

Answer: Option B
Explanation : We can find some local people living in log cabins in Taiga regions. Taiga, also known as coniferous or boreal forest, is the largest terrestrial biome on earth. It extends in a broad band across North America, Europe, and Asia to the southern border of the arctic tundra.


44. ‘The Scented Isle’ in the Mediterranean Sea is
A. Sardinia
B. Corsica
C. Sicily
D. Crete

Answer

Answer: Option B
Explanation : ‘The Scented Isle’ in the Mediterranean Sea is Corsica. Corsica is the fourth largest island (after Sicily, Sardinia, and Cyprus) in the Mediterranean. It lies 105 miles (170 km) from southern France and 56 miles (90 km) from northwestern Italy, and it is separated from Sardinia by the 7-mile (11-km) Strait of Bonifacio. Ajaccio is the capital.


45. When does winter solstice occur in the Northern Hemisphere?
A. 21st June
B. 22nd December
C. 21st March
D. 23rd March

Answer

Answer: Option B
Explanation : The winter solstice occurs during the hemisphere’s winter. In the Northern Hemisphere, this is the December solstice (usually 21 or 22 December).


46. Which of the following is the longest Railway Tunnel?
A. St. Gothard (Switzerland
B. East Finchley (London)
C. Cascade (U.S.
D.)
E. Tanna (Japan)

Answer

Answer: Option D
Explanation : Tanna (Japan) is the longest Railway Tunnel. Tanna Tunnel is a railroad tunnel in Shizuoka Prefecture, Japan operated by JR Central’s Tōkaidō Main Line. This 7.8 km long tunnel shortened the trunk route between Tokyo and Kobe by omitting a detour round the mountains between Atami and Numazu.


47. Among the following countries, which one is the most densely populated?
A. Belgium
B. France
C. Germany
D. The Netherlands

Answer

Answer: Option D
Explanation : Netherlands is the most densely populated. With over 16.5 million people and a population density of 488 people per km2, the Netherlands is the most densely populated country of the European Union and one of the mostly densely populated countries in the world. The total size of the Netherlands is 41,500 km2.


48. The countries with the lowest and worst economic conditions in the world referred to as the ‘Pole of hunger’ are
A. European Countries
B. Middle East Countries
C. Mid-West Countries
D. South East Asian Countries

Answer

Answer: Option A
Explanation : The countries with the lowest and worst economic conditions in the world referred to as the ‘Pole of hunger’ are European Countries.


49. Hammada is the name given to ________.
A. An icy desert
B. A sandy desert
C. A stony desert
D. None of the above

Answer

Answer: Option C
Explanation : Hammada is the name given to A stony desert. Rocky deserts are also known as Hamada. HAMMADA IS AN Arabic word used to describe “desert pavement.” The deserts of the world that have sand dunes capture the imagination, but many of the world’s desert areas are bleak, stony deserts.


50. Which one of the following descriptions denotes the phenomenon of El Nino ?
A. A violent rotating column of air extending from a thunderstorm to the ground
B. The deflection of the ocean currents in the northern and southern hemispheres due to the rotation of Earth
C. The Anomalous widespread warming of the sea surface of the tropical east and central Pacific Ocean
D. The revolving tropical storms of the Caribbean and Gulf of Mexico

Answer

Answer: Option C
Explanation : The Anomalous widespread warming of the sea surface of the tropical east and central Pacific Ocean denotes the phenomenon of El Nino.


51. The Earth’s population has ________ since the turn of the century.
A. Stayed the same
B. Doubled
C. Trebled
D. Multiplied ten times

Answer

Answer: Option C
Explanation : The Earth’s population has Trebled since the turn of the century. It wasn’t until the early 19th century that the world population reached its first big milestone: 1 billion people. Then, as the industrial revolution took hold and living standards improved, the rate of population growth increased considerably. The 20th century, however, is where population growth really took off, and over the past 100 years, the planet’s population has more than tripled in size.


52. Where are most or the Earth’s active volcanoes concentrated at present?
A. South America
B. Africa
C. Pacific Ocean
D. Europe

Answer

Answer: Option C
Explanation : In Pacific Ocean, the Earth’s active volcanoes concentrated at present. Volcanic eruptions occur only in certain places and do not occur randomly. This is because the Earth’s crust is broken into a series of slabs known as tectonic plates. Most volcanoes are found along a belt, called the “Ring of Fire” that encircles the Pacific Ocean.


53. The largest country in the world (area-wise) is
A. China
B. India
C. Russia
D. The USA

Answer

Answer: Option C
Explanation : The largest country in the world (area-wise) is Russia. Covering an expanse of over 6.6 million square miles, Russia is the world’s largest country by landmass, beating out runner-up Canada by around 2.8 million square miles. It includes nine different time zones and shares land borders with 14 neighboring countries.


54. Of the total water on the Earth, fresh water reserve a constitute approximately
A. 70%
B. 3%
C. 10%
D. 9%

Answer

Answer: Option B
Explanation : Of the total water on the Earth, 3% fresh water reserve a constitute approximately. Over 68 percent of the fresh water on Earth is found in icecaps and glaciers, and just over 30 percent is found in ground water. Only about 0.3 percent of our fresh water is found in the surface water of lakes, rivers, and swamps.


55. What percentage or all water available on the Earth is contained in the oceans?
A. 50%
B. 70%
C. 86%
D. 97%

Answer

Answer: Option D
Explanation : About 97 percent of the Earth’s water can be found in our oceans. Of the tiny percentage that’s not in the ocean, about two percent is frozen up in glaciers and ice caps. Less than one percent of all the water on Earth is fresh. A tiny fraction of water exists as water vapor in our atmosphere.


56. Sunda Trench lies in ________
A. Atlantic Ocean
B. Pacific Ocean
C. Indian Ocean
D. Antarctic Ocean

Answer

Answer: Option C
Explanation : Sunda Trench lies in Indian Ocean. The Sunda Trench, earlier known as and sometimes still indicated as the Java Trench, is an oceanic trench located in the Indian Ocean near Sumatra, formed where the Australian-Capricorn plates subduct under a part of the Eurasian Plate. It is 3,200 kilometres (2,000 mi) long.


57. The first oil well in the world was drilled at
A. Assam
B. Japan
C. Pensylvania
D. Former USSR

Answer

Answer: Option C
Explanation : The first oil well in the world was drilled at Pensylvania. On August 28, 1859, George Bissell and Edwin L. Drake made the first successful use of a drilling rig on a well drilled especially to produce oil, at a site on Oil Creek near Titusville, Pennsylvania.


58. If A and B are two stations antipode to each other then their
A. Seasons are opposite
B. Day and night are opposite
C. Seasons day and night are opposite
D. Climates are opposite

Answer

Answer: Option C
Explanation : If A and B are two stations antipode to each other then their seasons day and night are opposite. At antipodes, both the seasons and the pattern of day and night are thus opposite, i.e. winter at one and the summer at the other, and noon at one is simultaneous with midnight at the other point.


59. “Great Barrier Reef, the world’s largest Coral reef is located in
A. Caribbean Islands
B. Australia
C. Philippines
D. Indonesia

Answer

Answer: Option B
Explanation : The reef is located off the coast of Queensland, Australia, in the Coral Sea. Stretching for 1,429 miles over an area of approximately 133,000 square miles , the Great Barrier Reef is the largest coral reef system in the world.


60. Which is the smallest Country (in population)?
A. Vatican city
B. Nauru
C. Monaco
D. Palau

Answer

Answer: Option A
Explanation : With the population of around 500, European country Vatican City has smallest population in the world. Vatican City is the smallest country in the world, measuring just 0.2 square miles, almost 120 times smaller than the island of Manhattan. Situated on the western bank of the Tiber River, Vatican City’s 2-mile border is landlocked by Italy.


61. Which of the following are not pastoral nomads?
A. Pygmies
B. Kazaks
C. Masai
D. Lapps

Answer

Answer: Option A
Explanation : Pygmies are not pastoral nomads. Pygmies are nomadic peoples living in the equatorial rainforests of central Africa, with smaller numbers in south-east Asia. They are characterized by their short height (below 1.5 meters, or 4.5 feet, on average). They prefer their specific ethnic names, such as Baka or Mbuti, rather than the generic “pygmy.”


62. Collective Farm of Ukraine is also known as
A. Kolkhoz
B. Dom
C. Ryckay Eezik
D. Sovkhoz

Answer

Answer: Option A
Explanation : Collective Farm of Ukraine is also known as Kolkhoz. Kolkhoz, also spelled kolkoz, or kolkhos, plural kolkhozy, or kolkhozes, abbreviation for Russian kollektivnoye khozyaynstvo, English collective farm, in the former Soviet Union, a cooperative agricultural enterprise operated on state-owned land by peasants from a number of households who belonged to the collective and who were paid as salaried employees on the basis of quality and quantity of labour contributed.


63. Nautical mile is a unit of distance used in
A. Space travel
B. Navigation
C. Aviation
D. Communication

Answer

Answer: Option B
Explanation : Nautical mile is a unit of distance used in Navigation. A nautical mile is a unit of measurement used on water by sailors and/or navigators in shipping and aviation. It is the average length of one minute of one degree along a great circle of the Earth. One nautical mile corresponds to one minute of latitude.


64. The age of a tree can be estimated by
A. Its height
B. The girth of its trunk
C. The number and size of its branches
D. The number of concentric rings on a cross-section of its trunk

Answer

Answer: Option D
Explanation : The age of a tree can be estimated by The number of concentric rings on a cross-section of its trunk. A tree’s age can be determined by counting the annual growth rings in its trunk. Each ring represents one year, and the ring’s thickness reveals the relative amount of rainfall that year.


65. “Tin Bigha” corridor connects
A. India and Pakistan
B. India and China
C. Bangladesh and Pakistan
D. Bangladesh and India

Answer

Answer: Option D
Explanation : Tin Bigha Corridor is a strip of land in West Bengal on Indian border which was leased to Bangladesh in 2011 so as to provide it access to its Dahagram-Angarpota enclaves.


66. What is the administrative capital of South Africa?
A. Cape Town
B. Johannesburg
C. Kimberley
D. Pretoria

Answer

Answer: Option D
Explanation : Pretoria is the administrative capital of South Africa. Pretoria, city in Gauteng province and administrative capital of the Republic of South Africa. Pretoria stretches along both sides of the Apies River and extends into the western foothills of the Magaliesberg on the east.


67. Which of the following reflects radio waves back to the Earth?
A. Ionosphere
B. Thermosphere
C. Tropopause
D. Mesopause

Answer

Answer: Option A
Explanation : Ionosphere reflects radio waves back to the Earth. The low-frequency end of the window is limited by signals being reflected by the ionosphere back into space, while the upper limit is caused by absorption of the radio waves by water vapor and carbon dioxide in the atmosphere. The ionized part of the Earth’s atmosphere is known as the ionosphere.


68. A country lying within the Arctic Circle having geysers is
A. Greenland
B. Iceland
C. Norway
D. Denmark

Answer

Answer: Option B
Explanation : A country lying within the Arctic Circle having geysers is Iceland. The mainland of Iceland is just a few degrees south of the Arctic Circle (66°30’N). The Arctic Circle does however pass through Grímsey island, which lies 40 kilometres (25 mi) off the north coast of Iceland.


69. Which of the following is known as “the cold pole” of the world?
A. Laurentian region of Canada
B. Verkhoyansk in Russia
C. Antarctica
D. Greenland

Answer

Answer: Option B
Explanation : Verkhoyansk in Russia is known as “the cold pole” of the world. Verkhoyansk is the iciest city on the globe — having once measured minus 67.8 degrees Celsius. It’s now trying to use the frigid record to attract tourists.


70. The internationally recognised scale for describing wind speeds is ________.
A. The Richter Scale
B. The Moho’s Scale
C. The Modified Mercalli Scale
D. The Beaufort Scale

Answer

Answer: Option D
Explanation : The internationally recognised scale for describing wind speeds is The Beaufort Scale. Beaufort scale, in full Beaufort wind force scale, scale devised in 1805 by Commander (later Admiral and Knight Commander of the Bath) Francis Beaufort of the British navy for observing and classifying wind force at sea.


71. Bangkok lies on river
A. Kizil
B. Spree
C. Karnaphuli
D. Maenam

Answer

Answer: Option D
Explanation : Bangkok lies on river Maenam. The Chao Phraya (Maenam) begins at the confluence of the Ping and Nan rivers at Nakhon Sawan (also called Pak Nam Pho) in Nakhon Sawan Province. After this it flows south for 372 kilometres (231 mi) from the central plains to Bangkok and the Gulf of Thailand.


72. The most typical Australian tree which sheds its bark instead of leaves is
A. Eucalyptus
B. Teak
C. Birch
D. Oak

Answer

Answer: Option A
Explanation : The most typical Australian tree which sheds its bark instead of leaves is Eucalyptus. Shedding eucalyptus tree bark may help keep the tree healthy. As the tree sheds its bark, it also sheds any mosses, lichens, fungi and parasites that may live on the bark. Some peeling bark can perform photosynthesis, contributing to the rapid growth and overall health of the tree.


73. Barkhans are
A. Depositional features of wind action
B. Erosional features associated with rivers
C. Erosional features associated with wind action
D. Depositional features of glacial action

Answer

Answer: Option A
Explanation : Barchans are depositional features of wind action. They are a very common landform in sandy deserts all over the world and are arc-shaped, markedly asymmetrical in cross section, with a gentle slope facing toward the wind sand ridge, comprising well-sorted sand.


74. The Great Barrier Reef is ________
A. Conglomeration of corals in Australian waters
B. Mountains range in Utah U.S.
C.
D. Salt hills of Afghanistan
E. Sub-Oceanic mountain in South China Sea

Answer

Answer: Option A
Explanation : The Great Barrier Reef is Conglomeration of corals in Australian waters. It is the world’s largest coral reef system composed of over 2,900 individual reefs and 900 islands stretching for over 2,300 kilometres (1,400 mi) over an area of approximately 344,400 square kilometres (133,000 sq mi). The reef is located in the Coral Sea, off the coast of Queensland, Australia.


75. The most salty basin in the world is the
A. Caspian Sea
B. Dead Sea
C. Red Sea
D. Arabian Sea

Answer

Answer: Option B
Explanation : The most salty basin in the world is the Dead Sea. Its surface and shores are 427 metres below sea level, Earth’s lowest elevation on land. The Dead Sea is 306 m deep, the deepest hypersaline lake in the world.


76. Tropical deserts are rich in ________ resources.
A. Mineral
B. Agricultural
C. Soil
D. Animal

Answer

Answer: Option A
Explanation : Tropical deserts are rich in mineral resources. Rich mineral resources are found chiefly in deserts, and some are unique to deserts. Those minerals that occur naturally in dry saline inland bodies of water require sediments and near-surface brines for their formation. Minerals such as borax, sodium nitrate, sodium carbonate, bromine, iodine, calcium and strontium compounds are created when water in desert lakes (playa) evaporate.


77. Amongst the following, which is the largest island?
A. England
B. Japan
C. Borneo
D. New Guinea

Answer

Answer: Option D
Explanation : New Guinea is the largest island with a size of (2,941,517 sq miles).


78. Ferrel’s Law is associated with the
A. Velocity of winds
B. High pressure of winds
C. Low pressure of winds
D. Direction of winds

Answer

Answer: Option D
Explanation : Ferrel’s Law is associated with the Direction of winds. “If a body moves in any direction on the earth’s surface, there is a deflecting force arising from the earth’s rotation, which deflects it to the right in the northern hemisphere and to the left in the southern hemisphere.”


79. The Continents of Africa and Europe, at their close at point are separated from each other by the strait of
A. Messina
B. Bonifacio
C. Gibraltar
D. Otranto

Answer

Answer: Option C
Explanation : The Estrecho de Gibraltar which is also known as the Strait of Gibraltar is a channel connecting the Atlantic Ocean to the Mediterranean Sea. The channel lies between the northwesternmost part of Africa and the southernmost part of Europe.


80. Which is the correct sequence of the given American cities from West to East?
A. Salt Lake City – Pittsburgh – San Francisco – Boston
B. San Francisco – Salt Lake City Pittsburgh – Boston
C. Salt Lake City – San Francisco – Boston – Pittsburgh
D. San Pittsburgh – Salt Lake City – Boston

Answer

Answer: Option B
Explanation : Yhe correct sequence of the given American cities from West to East are San Francisco – Salt Lake City Pittsburgh – Boston.


81. The trans-continental railway in South America links
A. Moscow and Vladivostok
B. Sao Paulo and Trinidad
C. San Diego and Buenos Aires
D. Valparaiso and Buenos Aires

Answer

Answer: Option D
Explanation : The trans-continental railway in South America links Valparaiso and Buenos Aires.


82. The Shatt-al-Arab waterway is associated with the
A. Euphrates and Tigris
B. White Nile and Blue Nile
C. Jordan and Firat
D. Oxus and Kabul

Answer

Answer: Option A
Explanation : The Shatt-al-Arab waterway is associated with the Euphrates and Tigris. Shatt al-Arab is a river of some 200 km (120 mi) in length, formed by the confluence of the Euphrates and the Tigris in the town of al-Qurnah in the Basra Governorate of southern Iraq.


83. The formation of Extrusive rocks takes place ________
A. By solidification of magma above the earth’s surface
B. Due to excessive pressure at great depths
C. By deposition of magma at a great depth
D. At an intermediate depth by transformation

Answer

Answer: Option A
Explanation : The formation of Extrusive rocks takes place by solidification of magma above the earth’s surface. Extrusive igneous rocks cool and solidify quicker than intrusive igneous rocks. They are formed by the cooling of molten magma on the earth’s surface.


84. The busiest inland water-way of Europe is the Rhine. Its preeminence as a waterway rests on the fact that
A. It flows through the most industrialised part of Europe
B. It follows a rift valley
C. Its course is never frozen
D. It falls in the North Sea

Answer

Answer: Option C
Explanation : The busiest inland water-way of Europe is the Rhine. Its preeminence as a waterway rests on the fact that its course is never frozen.


85. Trans-continental railways have played an Important role in
A. International trade
B. Development of resources in the interior of continents
C. Providing employment to people
D. Deportation of prisoners to the interior

Answer

Answer: Option B
Explanation : Trans-continental railways have played an Important role in Development of resources in the interior of continents. The development of railroads was one of the most important phenomena of the Industrial Revolution. With their formation, construction and operation, they brought profound social, economic and political change to a country only 50 years old.


86. Which of the following is not a proof of Earth being an oblate spheroid?
A. When a ship goes out to sea first the hull disappears then the upper parts
B. The shadow of the Earth on the Sun during solar eclipse is always curved
C. Photographs of the Earth taken by artificial satellites have confirmed the fact
D. It has been found that polar circumference is lesser than equatorial circumference

Answer

Answer: Option B
Explanation : The shadow of the Earth on the Sun during solar eclipse is always curved is not a proof of Earth being an oblate spheroid.


87. Three European cities viz Vienna, Budapest and Belgrade lie on the river
A. Volga
B. Rhone
C. Rhine
D. Danube

Answer

Answer: Option D
Explanation : three european cities viz vienna, budapest and belgrade lie on the river Danube. Yhe river broadens into the famously smooth (and generally muddy brown, not blue) Danube as it passes by the capital cities of Vienna, Bratislava, and Budapest before turning south to cross the broad Hungarian Plain.


88. A large loop like bend in a river is called
A. A river cliff
B. A spur
C. A meander
D. A spit

Answer

Answer: Option C
Explanation : A large loop like bend in a river is called a meander. It is produced by a stream or river swinging from side to side as it flows across its floodplain or shifts its channel within a valley.


89. Black forest is found in
A. France
B. Germany
C. Czechoslovakia
D. Romania

Answer

Answer: Option B
Explanation : Black forest is found in Germany. It is a mountainous region with dense, evergreen forests in South West Germany bordering France and surrounded by Rhine Valley.


90. The capital of South Sudan is
A. Suva
B. Juba
C. Khartoum
D. Taichung

Answer

Answer: Option B
Explanation : Juba is capital and the largest city of South Sudan and a provincial capital. Juba is a commercial centre for agricultural products produced in the surrounding area.


91. ‘Land of Fishermen’ refers to
A. Europe
B. Former USSR
C. Norway
D. Japan

Answer

Answer: Option C
Explanation : ‘Land of Fishermen’ refers to Norway. The Norwegian coast is 21 000 km long and the prospect for expanding fisheries and marine aquaculture in the country is huge. Norway has 90 000 km² of sea within the baseline, which corresponds to approximately 1/3 of the total land area. Throughout its history, fishery has been a major industry in Norway.


92. A passenger, on his transcontinental railway journey in Europe comes across the cities. What is the sequence in which he touches these cities in his east to west journey? ,1. Belgrade,2. Vienna,3. Sophia,4. Prague
A. 1 3 4 2
B. 4 2 1 3
C. 2 3 4 1
D. 3 1 2 4

Answer

Answer: Option D
Explanation : A passenger, on his transcontinental railway journey in Europe comes across the cities. The sequence in which he touches these cities in his east to west journey are Sophia, Belgrade, Vienna and Prague.


93. Why is very little rice sold in world markets?
A. The farmers cultivating rice exist on subsistence cultivation
B. The main producers of rice are also the main consumers
C. The market price of rice is not very remunerative
D. Rice is not eaten throughout the world

Answer

Answer: Option B
Explanation : Very little rice sold in world markets because the main producers of rice are also the main consumers.


94. In hydro power station, the motion produced in turbines is due to
A. Flow of water
B. Burning of coal
C. Burning of diesel
D. Production of steam

Answer

Answer: Option A
Explanation : Hydro power Projects work on the principle of harnessing the potential energy of water stored at some height. As the water is released from the dam through narrow channel it falls on the tubine and produced motion in turbine to produce electricity.


95. ‘The Land of the Morning Calm’ is
A. Japan
B. Malaysia
C. Indonesia
D. Korea

Answer

Answer: Option D
Explanation : ‘The Land of the Morning Calm’ is Korea. The title was most suited to South Korea because of its spellbinding natural beauty of picturesque high mountains and clear waters and its splen-did tranquillity, particularly in the morning which further confirmed the title on South Korea as the ”Land of Morning Calm”.


96. Asia accounts for nearly ________ percent of world population.
A. 40
B. 45
C. 59
D. 70

Answer

Answer: Option C
Explanation : Asia accounts for nearly 59 percent of world population. Asia population is equivalent to 59.66% of the total world population. Asia ranks number 1 among regions of the world (roughly equivalent to “continents”), ordered by population.


97. Caucasian people account for ________ per cent of world population.
A. 33
B. 39
C. 29
D. 43

Answer

Answer: Option A
Explanation : Caucasian people account for 33 percent of world population.


98. The subject that deals with the regional distribution of human races and their history is called
A. Biogeography
B. Ethnography
C. Cartography
D. Paleontology

Answer

Answer: Option B
Explanation : The subject that deals with the regional distribution of human races and their history is called Ethnography. Ethnography is descriptive study of a particular human society or the process of making such a study. Contemporary ethnography is based almost entirely on fieldwork and requires the complete immersion of the anthropologist in the culture and everyday life of the people who are the subject of his study.


99. Adam’s bridge connects ________
A. Amman and Damascus
B. Dhanushkodi (Rameshwaram) and Talaimannar
C. Israel and Jerusalem
D. Persian Gulf and Gulf of Oman

Answer

Answer: Option B
Explanation : Adam’s bridge connects Dhanushkodi (Rameshwaram) and Talaimannar. Ram Setu, popularly known as Adam’s Bridge, lies between Dhanushkodi and Talaimannar in Sri Lanka. The 1964 cyclone washed away the 17 km-long railway line which remained unattended till date. Dhanushkodi was well connected by rail and road with Pamban and the mainland until the cyclone struck.


100. A short cut sea route between the European and-Asian countries was provided by
A. Cape of Good Hope route
B. Seuz Canal
C. Panama Canal
D. North Atlantic route

Answer

Answer: Option B
Explanation : A short cut sea route between the European and-Asian countries was provided by Seuz Canal. The Suez Canal is a sea-level waterway in Egypt, connecting the Mediterranean Sea to the Red Sea through the Isthmus of Suez.


101. In both the hemispheres the trade winds blow towards the
A. Equator
B. Horse latitudes
C. Sub polar low pressure belts
D. Poles

Answer

Answer: Option A
Explanation : In both the hemispheres the trade winds blow towards the Equator. The surface air that flows from these subtropical high-pressure belts toward the Equator is deflected toward the west in both hemispheres by the Coriolis effect.


102. Death Valley is in
A. The USA
B. Jordan
C. Israel
D. Peru

Answer

Answer: Option A
Explanation : Death Valley is a valley in the U.S. state of California.


103. The gigantic tidal waves produced by earthquakes are called
A. Breakers
B. Tsunamis
C. Currents
D. Drift

Answer

Answer: Option B
Explanation : The gigantic tidal waves produced by earthquakes are called Tsunamis. Any big surge of water from the oceans is called a tidal wave; the terms “Tsunami” and “Tidal Waves” mean the same and are interchangeable. Tsunamis are mistakenly called tidal waves because, when approaching land, they look as a tide which suddenly rushes away and crashes back in a form of a huge wave.


104. The broken hill famous for zinc and lead are located in
A. Turkey
B. France
C. Germany
D. Australia

Answer

Answer: Option D
Explanation : The broken hill famous for zinc and lead are located in Australia. The Broken Hill Ore Deposit is located underneath Broken Hill in western New South Wales, Australia, and is the namesake for the town. It is arguably the world’s richest and largest zinc-lead ore deposit.


105. Podzol is a type of soil which is characteristic of a region under the cover of ________ forest.
A. Deciduous
B. Coniferous
C. Evergreen
D. Alpine

Answer

Answer: Option B
Explanation : Podzol is a type of soil which is characteristic of a region under the cover of coniferous forest. Podzolic soils are forested soils found primarily on sandy deposits in ecozones (or parts of ecozones) where the mean annual precipitation is above 700 mm. Coniferous-dominated plant communities are the major vegetation type found on Podzolic soils.


106. Cuban economy is based on
A. Fishing
B. Sugarcane cultivation
C. Iron ore mining
D. Dairying

Answer

Answer: Option B
Explanation : Cuban economy is based on Sugarcane cultivation. Sugarcane and the growth of slavery. During the 18th century Cuba depended increasingly on the sugarcane crop and on the expansive, slave-based plantations that produced it. In 1740 the Havana Company was formed to stimulate agricultural development by increasing slave imports and regulating agricultural exports.


107. Schist is the metamorphic equivalent of ________.
A. Granite
B. Basalt
C. Shale
D. Dolerite

Answer

Answer: Option C
Explanation : Schist is the metamorphic equivalent of Shale. Schist is a still higher degree of metamorphism, characterized by coarse grained foliation and/or lineation, with mica crystals large enough to be easily identified with the unaided eye.


108. Fog is common around Newfoundland coast because ________
A. High tides occur causing storms
B. The rainfall is heavy
C. Warm and cold currents meet
D. Icebergs melt around the coast

Answer

Answer: Option C
Explanation : Fog is common around Newfoundland coast because Warm and cold currents meet. When cold and warm ocean current meet, the warm ocean current condenses after getting in contact with cold air and it results into fog. Example: Fog is found in Newfoundland Coast of North America due to meeting of warm Gulf Stream and cold Labrador Current.


109. ________ is the largest city in Canada and is the terminus of the Canadian Pacific Railway.
A. Toronto
B. Chicago
C. Quebec
D. Halifax

Answer

Answer: Option C
Explanation : Quebec is the largest city in Canada and is the terminus of the Canadian Pacific Railway.


110. Silk production in the world has declined because of
A. Lack of demand
B. Lack of remunerative prices
C. Growing competition from man-made fibres
D. Dearth of skilled labour

Answer

Answer: Option C
Explanation : Silk production in the world has declined because of Growing competition from man-made fibres.


111. Which of the following varieties of coal accounts for the maximum coal output of the world?
A. Peat
B. Lignite
C. Bituminous
D. Anthracite

Answer

Answer: Option C
Explanation : Bituminous varieties of coal accounts for the maximum coal output of the world. Bituminous coal or black coal is a relatively soft coal containing a tarlike substance called bitumen or asphalt. It is of higher quality than lignite coal but of poorer quality than anthracite. Formation is usually the result of high pressure being exerted on lignite.


112. Which of the following has to be crossed over to reach Colombo from Kanyakumari?
A. Palk Strait
B. Gulf of Palk
C. Gulf of Mannar
D. Cook Strait

Answer

Answer: Option C
Explanation : Colombo lies on the western side of Sri Lanka where lies the water body of Gulf of Mannar. So, Gulf of Mannar has to be crossed to reach Kanyakumari.


113. Condensation occurs when
A. The air is cold
B. The air is warm
C. The air is saturated
D. None of the above

Answer

Answer: Option A
Explanation : Condensation occurs when the air is cold. Cold air holds less water vapor than warm air. This is why warm climates are often more humid than cold ones: Water vapor remains in the air instead of condensing into rain. Cold climates are more likely to have rain, because water vapor condenses more easily there.


114. Where is Copacabana Beach located?
A. Buenos Aires
B. Hawaiian Islands
C. Rio de Janeiro
D. Valletta

Answer

Answer: Option C
Explanation : Copacabana Beach is located in Rio de Janeiro. Located in Zona Sul, Copacabana Beach is one of the most famous and most beautiful beaches in the world.


115. Which one of the following countries is land locked?
A. Bolivia
B. Peru
C. Suriname
D. Uruguay

Answer

Answer: Option A
Explanation : Bolivia Is Landlocked because this landlocked country doesn’t have access to one. At least, not anymore: During the War of the Pacific, a land fight with Chile that lasted from 1879 to 1883, Bolivia ceded all 250 miles of its coastline.


116. The temperature at which dew is formed is known as
A. Evaporation point
B. Dew point
C. Precipitation point
D. Critical point

Answer

Answer: Option B
Explanation : The temperature at which dew is formed is known as Dew point. The dew point is the temperature at which air is saturated with water vapor, which is the gaseous state of water.


117. A narrow passage of water that connects two large bodies or water is known as
A. Bay
B. Isthmus
C. Strait
D. Archipelago

Answer

Answer: Option C
Explanation : A narrow passage of water that connects two large bodies or water is known as Strait. A strait is a narrow channel of water that connects two larger bodies of water.


118. The correct descending order of the leading producers of milk is
A. China India Russia U.S.
B.
C. India U.S.
D. China Russia
E. U.S.

Answer

Answer: Option B
Explanation : The correct descending order of the leading producers of milk is India, U.S.A., China and Russia.


119. Doldrums is/are
A. Mount of sand blown and formed on beaches and in deserts
B. Low pressure belt of calm and light surface winds near the Equator
C. The north-east and south-east winds
D. Planetary winds

Answer

Answer: Option B
Explanation : Doldrums are Low pressure belt of calm and light surface winds near the Equator. The “doldrums” is a popular nautical term that refers to the belt around the Earth near the equator where sailing ships sometimes get stuck on windless waters.


120. Which of the following natural regions is inhabited by the Eskimos of Canada and Alaska, Lappas of Finland and the Samoyeds and Yakuts of Siberia?
A. High Mountain region
B. Polar Tundra
C. The Taiga region
D. Cool (Mid-Latitude) West margin region

Answer

Answer: Option B
Explanation : Polar Tundra natural regions is inhabited by the Eskimos of Canada and Alaska, Lappas of Finland and the Samoyeds and Yakuts of Siberia.


121. The Tropical Grasslands of Brazil are known as
A. Llanos
B. Campos
C. Selvas
D. Savanna

Answer

Answer: Option B
Explanation : The Tropical Grasslands of Brazil are known as Campos. The Campos, grassland with few trees or shrubs except near streams, lies between 24°S and 35°S; it includes parts of Brazil, Paraguay and Argentina, and all of Uruguay. Grassland -based livestock production is very important, based on the natural grassland that covers most of the area.


122. Mid-latitude grasslands of Canada are known as
A. Steppes
B. Prairies
C. Pampas
D. Downs

Answer

Answer: Option B
Explanation : Mid-latitude grasslands of Canada are known as Prairies. Prairies are ecosystems considered part of the temperate grasslands, savannas, and shrublands biome by ecologists, based on similar temperate climates, moderate rainfall, and a composition of grasses, herbs, and shrubs, rather than trees, as the dominant vegetation type.


123. The most important oil producing area of the USA is
A. The Applachian region
B. The Texas-Louisiana belt
C. The Ohio-Indiana belt
D. The California-Colorado belt

Answer

Answer: Option B
Explanation : The most important oil producing area of the USA is The Texas-Louisiana belt.


124. Fold mountains are formed due to forces born out of
A. Tension
B. Compression
C. Expansion
D. Volcanoes

Answer

Answer: Option B
Explanation : Fold mountains are formed due to forces born out of Compression. Fold mountains are created where two or more of Earth’s tectonic plates are pushed together. At these colliding, compressing boundaries, rocks and debris are warped and folded into rocky outcrops, hills, mountains, and entire mountain ranges.


125. Borobudur in Java is famous for
A. Lord Buddha’s tooth preserved for posterity
B. ABuddhist Temple
C. Its natural museum
D. Fisheries Research Station

Answer

Answer: Option B
Explanation : Borobudur in Java is famous for Buddhist Temple. The Borobudur Temple Compounds is one of the greatest Buddhist monuments in the world, and was built in the 8th and 9th centuries AD during the reign of the Syailendra Dynasty. The monument is located in the Kedu Valley, in the southern part of Central Java, at the centre of the island of Java, Indonesia.


126. Which of the following is incorrect?
A. Within the Arctic and Antarctic Circles there is at least one day in the year during which the sun does not set and at least one day on which it never rises
B. At the North Pole there is darkness for half the year
C. At the summer solstice the sun shines vertically over the Tropic of Capricorn
D. The sun shines vertically over the Equator twice the year

Answer

Answer: Option C
Explanation : At the summer solstice the sun shines vertically over the Tropic of Cancer. The moment the sun shines directly on the Tropic of Cancer, an imaginary line north of the equator. This is the first day of astronomical summer and also the longest day of the year for people in the Northern Hemisphere.


127. Which of the following grades of coal has the highest carbon content?
A. Anthracite
B. Bituminous coal
C. Lignite
D. Peat

Answer

Answer: Option A
Explanation : Anthracite is the highest rank of coal. Unlike other types of coal, it is usually considered to be a metamorphic rock. It has a carbon content of over 87% on a dry ash-free basis. Anthracite coal generally has the highest heating value per ton on a mineral-matter-free basis. It is often subdivided into semi-anthracite, anthracite, and meta-anthracite on the basis of carbon content. Anthracite is often referred to as “hard coal”; however, this is a layman’s term and has little to do with the hardness of the rock.


128. In population, high growth rate is a characteristics of
A. High birth and High mortality rate
B. High birth and Low mortality rate
C. Low birth and Low mortality rate
D. Low birth and High mortality rate

Answer

Answer: Option B
Explanation : High birth rate (annual no. of birth/ 1000 population), along with low mortality rate (annual no. of death/ 1000 population) is a condition responsible for huge population growth. It is mostly found in developing countries. e.g. Bangladesh and India.


129. A narrow strip of land connecting two large land masses is known as
A. Strait
B. Isthmus
C. Headland
D. Bay

Answer

Answer: Option B
Explanation : A narrow strip of land connecting two large land masses is known as Isthmus. It is generally surrounded by water bodies on two of its sides. (Irrelevant) Generally canals are built across isthmuses, as they provide the shortest route for travelling from one water body to the other by sea.


130. Which or the following is known as the lifelines of a country?
A. Natural growth rate of population
B. Food supplies
C. Energy resources
D. Modem means of transport and communication

Answer

Answer: Option D
Explanation : Modem means of transport and communication is known as the lifelines of a country. The means of transportation and communication are called the lifelines of a nation and its economy because they are the preconditions for progress and development.


131. What does the 17th parallel separate?
A. South and North America
B. North and South Korea
C. South and North Vietnam
D. South and North Yemen

Answer

Answer: Option C
Explanation : The Seventeenth parallel was the provisional military demarcation line between North and South Vietnam established by the Geneva Accords of 1954.


132. Cyclones are called as hurricanes in
A. Bangladesh
B. China
C. Gulf of Mexico and Caribbean Sea
D. Japan

Answer

Answer: Option C
Explanation : Cyclones are called as hurricanes in Gulf of Mexico and Caribbean Sea. Hurricanes originate in the Atlantic basin, which includes the Atlantic Ocean, Caribbean Sea, and Gulf of Mexico, the eastern North Pacific Ocean, and, less frequently, the central North Pacific Ocean.


133. What is the capital of East Timor ?
A. Darwin
B. DiIi
C. Semarang
D. Yogyakarta

Answer

Answer: Option B
Explanation : Dili, also spelled Dilly or Dilli, city and capital of East Timor. It lies on Ombai Strait on the northern coast of Timor island, the easternmost of the Lesser Sunda Islands.


134. Khartoum is situated at the confluence of ________
A. Murray and Darling
B. Porena and Prague
C. Tigris and Euphrates
D. White Nile and Blue Nile

Answer

Answer: Option D
Explanation : Khartoum is situated at the confluence of White Nile and Blue Nile. Khartoum is a city and the executive capital of Sudan, just south of the confluence of the Blue and White Nile rivers.


135. On the Equator the day and night are equal
A. On equinoxes
B. On summer solstice
C. On winter solstice
D. Throughout the year

Answer

Answer: Option D
Explanation : On the Equator the day and night are equal throughout the year because it does not tilt in relation to the sun’s location. However, throughout the entire orbit around the sun, the Earth’s equator remains nearly the same distance away at all times.


136. Country peopled almost by migration is
A. The USA
B. Brazil
C. Australia
D. Hawaii

Answer

Answer: Option C
Explanation : Country peopled almost by migration is Australia. Australia’s estimated resident population (ERP) reached 24 million. More than one quarter (28.2%) of Australia’s resident population was born overseas—a level that is considered very high compared to most other OECD countries.


137. European Turkey is separated from its Asian part by
A. Straits of Bosporus
B. Sea of Marmara
C. Straits of Dardanelles
D. All the above

Answer

Answer: Option D
Explanation : European Turkey is separated from its Asian part by Straits of Bosporus, Sea of Marmara and Straits of Dardanelles.


138. The actual amount or moisture contained in the air is known as
A. Relative humidity
B. Moisture content
C. Dew point
D. Absolute humidity

Answer

Answer: Option D
Explanation : The actual amount or moisture contained in the air is known as Absolute humidity. Absolute humidity (units are grams of water vapor per cubic meter volume of air) is a measure of the actual amount of water vapor in the air, regardless of the air’s temperature. The higher the amount of water vapor, the higher the absolute humidity.


139. The difference between the diameter of the Earth at the poles and at equator is
A. 27 miles
B. 36 miles
C. 48 miles
D. 57 miles

Answer

Answer: Option A
Explanation : The difference between the diameter of the Earth at the poles and at equator is 27 miles. The Earth’s equatorial bulge is about 43 kilometers (27 miles).


140. The only port in erstwhile Soviet Union that is ice free throughout the year is
A. Leningrad
B. Odessa
C. Murmansk
D. Vladivostok

Answer

Answer: Option C
Explanation : The only port in erstwhile Soviet Union that is ice free throughout the year is Murmansk. The port of Murmansk remains ice-free year round due to the warm North Atlantic Current and is an important fishing and shipping destination.


141. On which or the following African riven is the Kariba Dam one of the world’s largest hydel power systems located?
A. The Nile
B. The Niger
C. The Congo
D. The Zambezi

Answer

Answer: Option D
Explanation : On the Zambezi river is the Kariba Dam one of the world’s largest hydel power systems is located. The dam stands 128 metres (420 ft) tall and 579 metres (1,900 ft) long. The dam forms Lake Kariba, which extends for 280 kilometres (170 mi) and holds 185 cubic kilometres (150,000,000 acre⋅ft) of water.


142. The countries which are called the ‘Rice Bowl’ of Asia are
A. China Japan Sri Lanka India
B. Myanmar Thailand Kampuchea and Vietnam
C. India Pakistan Nepal Mongolia
D. Japan Indonesia Malaysia India

Answer

Answer: Option B
Explanation : The countries which are called the ‘Rice Bowl’ of Asia are Myanmar, Thailand, Kampuchea and Vietnam. The Mekong region is Asia’s rice bowl: in 2014 lower Mekong countries (Myanmar, Laos, Cambodia, Thailand and Vietnam) produced more than 100m tonnes of rice, around 15% of the world’s total.


143. The major factors responsible for changes in the sizes of population are
A. Births
B. Deaths
C. Migrations
D. All the above

Answer

Answer: Option D
Explanation : The major factors responsible for changes in the sizes of population are Births, Deaths and Migrations.


144. Hydro-electric power provides more than 90% of the total power consumed in
A. Norway
B. Sweden
C. India
D. UK

Answer

Answer: Option A
Explanation : Hydro-electric power provides more than 90% of the total power consumed in Norway. The electricity sector in Norway relies predominantly on hydroelectricity. A significant share of the total electrical production is consumed by national industry.


145. Which of the following are reasons for the chief fishing grounds being located in continental shelf areas where cold and warm currents meet?,I. Warm currents are rich in phosphates and nitrates,II. Plankton – the chief fish feed – thrives in these regions,III. The meeting of cold and warm currents allows the water to be well aerated,IV. The epicontinental seas allow sunlight to reach almost to the bottom
A. I and III
B. II and IV
C. I II and III
D. II III and IV

Answer

Answer: Option D
Explanation : The reasons for the chief fishing grounds being located in continental shelf areas where cold and warm currents meet are Plankton – the chief fish feed – thrives in these regions, The meeting of cold and warm currents allows the water to be well aerated and The epicontinental seas allow sunlight to reach almost to the bottom.


146. Kiel Canal connects the North Sea with the ________ sea.
A. Atlantic
B. Baltic
C. Arctic
D. None of these

Answer

Answer: Option B
Explanation : Kiel Canal connects the North Sea with the Baltic sea. The canal was finished in 1895, but later widened, and links the North Sea at Brunsbüttel to the Baltic Sea at Kiel-Holtenau.


147. The Great Barrier Reef is found
A. Close to Greenland
B. Near the Suez Canal
C. In the Bay of Bengal
D. Of the coast of Queensland

Answer

Answer: Option D
Explanation : The Great Barrier Reef is found of the coast of Queensland. The Great Barrier Reef is the world’s largest coral reef. It is near the coast of Queensland, Australia. It is made up of nearly 2900 coral reefs and over 600 islands. It is 327,800 km² big and 2600 km long.


148. Where is the Great Barrier Reef located?
A. Pacific Ocean
B. Indian Ocean
C. Atlantic Ocean
D. Arctic Ocean

Answer

Answer: Option A
Explanation : Great Barrier Reef is located on the off shore region of Eastern Australia in the Pacific Ocean. It runs parallel to eastern coast of Australia.


149. Which of the following are related with “Red Indian”?
A. New Zealand
B. Srilanka
C. North America
D. Kenya

Answer

Answer: Option C
Explanation : Red Indian are the Native American tribes of U.S.A. Native Americans who were living in North America when Europeans arrived there used to be called Red Indians.


150. Scandinavia is a suitable country for exploiting hydroelectricity because of the presence of
A. Lakes
B. Steep slopes
C. Great rivers
D. Waterfalls

Answer

Answer: Option A
Explanation : Scandinavia is a suitable country for exploiting hydroelectricity because of the presence of Lakes.


151. Which of the following is taken into consideration while deciding a shipping route?
A. Depth of the sea
B. Availability of ports and harbours
C. Climate of the region
D. All the above

Answer

Answer: Option D
Explanation : The following that is taken into consideration while deciding a shipping route are Depth of the sea, Availability of ports and harbours and Climate of the region.


152. Lowest temperature is recorded in
A. USA
B. Mexico
C. Verkhoyansk
D. Alaska

Answer

Answer: Option C
Explanation : Lowest temperature is recorded in Verkhoyansk. In January the average temperature is minus 56 degrees Fahrenheit (minus 48.9 degrees Celsius) and mean monthly temperatures stay below freezing from October through April. In 1892, residents recorded the still all-time low of minus 90 degrees Fahrenheit (minus 67.8 degrees Celsius).


153. Which of the following is a correct sequence of increasing depth?
A. Continental shelf Continental slope Deep sea plains Ocean depths
B. Continental slope Continental shelf Deep sea plains Ocean depths
C. Continental shelf Continental slope Ocean depths Deep sea plains
D. Continental slope Continental shelf Ocean depths Deep sea plains

Answer

Answer: Option C
Explanation : Continental shelf, Continental slope, Ocean depths and then Deep sea plains is a correct sequence of increasing depth.


154. What is the similarity between Milwaukee Deep, Java Trench and Challenger Deep?
A. They all are trenches in the Pacific Ocean
B. They are the deepest points of the Atlantic Indian and Pacific Oceans respectively
C. They all are trenches in the Indian Ocean
D. They all are deeps of the Atlantic Ocean

Answer

Answer: Option B
Explanation : Milwaukee Deep, Java Trench and Challenger Deep are the deepest points of the Atlantic, Indian and Pacific Oceans, respectively.


155. The term “epicentre” is associated with
A. The inner core of the earth
B. The outer core of the Earth
C. Earthquakes
D. Volcanoes

Answer

Answer: Option C
Explanation : The term “epicentre” is associated with Earthquakes. The epicenter is the point on the earth’s surface vertically above the hypocenter (or focus), point in the crust where a seismic rupture begins.


156. The largest desert in the world is the
A. Sahara
B. Gobi
C. Kalahari
D. Thar

Answer

Answer: Option A
Explanation : The largest desert in the world is the Sahara. The largest is the Sahara Desert, a subtropical desert in northern Africa. It covers a surface area of about 3.5 million square miles.


157. Eskimos belong to
A. Mongoloid race
B. White race
C. Negroid race
D. None of these

Answer

Answer: Option A
Explanation : Eskimos belong to Mongoloid race. Eskimos are racially distinct from American Indians, and are not, as previously believed, merely “Indians transformed.” In fact, the Eskimos are most closely related to the Mongolian peoples of eastern Asia. Eskimos consider themselves to be “Inuit” (The People). The Eskimo-Aleut languages are unrelated to any American Indian language groups.


158. Hemp of Mesta, Ramie and Kapok are
A. Food crops
B. Fodder crops
C. Fibre crops
D. Plantation crops

Answer

Answer: Option C
Explanation : Hemp of Mesta, Ramie and Kapok are Fibre crops. Fiber crops are field crops grown for their fibers, which are traditionally used to make paper, cloth, or rope. They are organized into 3 main groups—textile fibers, cordage fibers, and filling fibers. They are a type of natural fiber.


159. Mid-latitude or temperate forests include the trees of
A. Rosewood Mahogany Rubber
B. Teak Sal Bamboo
C. Pine Fir Spruce
D. Olive Mapple Walnut

Answer

Answer: Option D
Explanation : Mid-latitude or temperate forests include the trees of Olive, Mapple, Walnut. The mid-latitude regions of the Earth’s surface, those that generally lie between the 10°C and 18°C isotherms, are characterized by having distinct summer and winter seasons, and are designated as the temperate zones [Bailey, 1996].


160. Which of the following set of countries are completely surrounded by land?
A. Libya Zambia Czech Republic
B. Mongolia Zambia Bolivia
C. Paraguay Mongolia Syria
D. Syria Czech Republic Paraguay

Answer

Answer: Option B
Explanation : Mongolia, Zambia and Bolivia are completely surrounded by land. They are Landlocked countries. A landlocked country is defined as a country that is completely surrounded by land.


161. The ________ mountains crossing the Great Plains and the ________ Sea further south separate Europe from Asia.
A. Caucasus Black
B. Ural Caspian
C. Carpathian Mediterranean
D. Balkan Sea of Azov

Answer

Answer: Option B
Explanation : The Ural mountains crossing the Great Plains and the Caspian Sea further south separate Europe from Asia.


162. Which of the following separates Asia and Africa?
A. Panama Canal
B. Suez Canal
C. Nile River
D. Buckingham River

Answer

Answer: Option B
Explanation : Suez Canal connects Mediterranean Sea with Red Sea & separates African continent from Asia continent.


163. The river Mahaweli flows through
A. India
B. Bangladesh
C. Sri Lanka
D. Pakistan

Answer

Answer: Option C
Explanation : The river Mahaweli flows through Sri Lanka. Mahaweli Ganga, (Sinhalese: “Great Sandy River”), river, central and eastern Sri Lanka. At 208 mi (335 km) in length, it is Sri Lanka’s longest river.


164. Which one of the following can be considered as an initial warning of an approaching Tsunami?
A. Thundering noise
B. Squally winds and rainfall
C. Rapid withdrawal of water away from the beach
D. Rapid landward movement of water

Answer

Answer: Option C
Explanation : Rapid withdrawal of water away from the beach can be considered as an initial warning of an approaching Tsunami. People who are near the seashore during a strong earthquake are at the highest risk. Rapid changes in the water level are an indication of an approaching tsunami.


165. The number of women per thousand men is called
A. Men-women ratio
B. Sex ratio
C. Relative density
D. Manage ratio

Answer

Answer: Option B
Explanation : The number of women per thousand men is called Sex ratio. In sheer numbers, males outnumber females by 35 million in population. Sex ratio is defined as the number of females per thousand males.


166. The place of origin of an earthquake is called
A. Seismographic zone
B. Perimeter
C. Epicentre
D. Seismic focus

Answer

Answer: Option D
Explanation : The place of origin of an earthquake is called Seismic focus. Earth’s surface directly above a hypocenter or focus, the point where an earthquake or an underground explosion originates.


167. The “cradles of civilization” have been
A. Valleys
B. Plains
C. Plateaux
D. Hills

Answer

Answer: Option B
Explanation : The “cradles of civilization” have been Plains. In Mesopotamia, the convergence of the Tigris and Euphrates rivers produced rich fertile soil and a supply of water for irrigation.It is because of this that the fertile crescent region, and Mesopotamia in particular, are often referred to as the “cradle of civilization”.


168. The river transports material in
A. Suspension
B. Solution
C. Traction
D. All of these

Answer

Answer: Option D
Explanation : The river transports material in Suspension, Solution and Traction.


169. Krivoy Rog, Kerch Peninsula, Magnitogorsk are major producers of ________ in former Soviet Union.
A. Coal
B. Manganese
C. Copper
D. Iron ore

Answer

Answer: Option D
Explanation : Krivoy Rog, Kerch Peninsula, Magnitogorsk are major producers of Iron ore in former Soviet Union. The Krivoy Rog Basin and the Kerch’ district, on the Crimean peninsula, are the iron ore producing areas in the southern Ukraine.


170. Forest soils are
A. Rich in potash
B. Deficient in potash
C. Rich in salt
D. Rich in organic matter

Answer

Answer: Option D
Explanation : Forest soils are Rich in organic matter. Forest soils, where soil formation has been influenced by forest vegetation, are generally characterized by deeply rooted trees, significant ‘litter layers’ or O horizons, recycling of organic matter and nutrients, including wood, and wide varieties of soil-dwelling organisms.


171. A continuous belt of urban population progressively linking several cities is known as
A. Metropolis
B. Megalopolis
C. Conurbation
D. City sprawl

Answer

Answer: Option C
Explanation : A continuous belt of urban population progressively linking several cities is known as Conurbation. Conurbation is the result of several central cities whose urban areas have merged.


172. Why does the mercury column in the barometer fall rapidly before a severe storm? It is due to
A. Increase in the humidity of air
B. Fall in the atmospheric pressure
C. Severe heat energy from the sun
D. Rise in the atmospheric pressure

Answer

Answer: Option B
Explanation : The mercury column in the barometer fall rapidly before a severe storm, It is due to Fall in the atmospheric pressure. In general, a falling barometer indicates the approach of a storm. If the mercury is over 30.20 inches but falling quickly, warmer, cloudier weather is coming.


173. Which of the following statements about New Zealand is incorrect ?
A. It is a part of Australian continent
B. It consists of two islands separated by Cook Strait
C. Tasman Sea is a land locked sea in the South Island
D. Mt. Cook is the highest peak of New Zealand

Answer

Answer: Option C
Explanation : Tasman Sea, section of the southwestern Pacific Ocean, between the southeastern coast of Australia and Tasmania on the west and New Zealand on the east; it merges with the Coral Sea to the north and encloses a body of water about 1,400 miles (2,250 km) wide and 900,000 square miles (2,300,000 square km) in area. It is not landlocked.


174. Most of the people in the world live in lowland areas except those in ________
A. Central and Western Africa
B. Southern and South – eastern Europe
C. Southern and South – eastern Asia
D. Central America and North – western South America

Answer

Answer: Option A
Explanation : Most of the people in the world live in lowland areas except those in Central and Western Africa.


175. The length of the Suez Canal is approximately
A. 80 km
B. 110 km
C. 140 km
D. 190 km

Answer

Answer: Option D
Explanation : The length of the Suez Canal is approximately 190 km. It extends from the northern terminus of Port Said to the southern terminus of Port Tewfik at the city of Suez. Its length is 193.30 km (120.11 mi), including its northern and southern access channels.


176. Which of the following tribe performs animal husbandry?
A. Bodo
B. Maasai
C. Pygmy
D. Eskimo

Answer

Answer: Option B
Explanation : Maasai tribe is found in Africa on the border region of Kenya & Tanzania. This tribe performs animal husbandry. They build circular huts with sticks & bushes & keep zebu cattle.


177. The pygmies are found in ________.
A. The Congo Basin Africa
B. Bihar India
C. Queenslands Australia
D. Finland Northern Europe

Answer

Answer: Option A
Explanation : The pygmies are found in The Congo Basin, Africa. Various hunter-gatherer pygmy groups inhabit the forests of the Congo basin. Most of these groups live in close association with the agriculturists, trading foods, meats, and other products.


178. The 90º East Ridge lies in the ________ Ocean.
A. Pacific
B. Atlantic
C. Indian
D. Arctic

Answer

Answer: Option C
Explanation : The 90º East Ridge lies in the Indian Ocean. The Ninety East Ridge is a sea mount in the Indian Ocean which runs approximately South to North along the 90 degree line of longitude.


179. Which parallel separates North and South Korea?
A. 38th
B. 50th
C. 42nd
D. 40th

Answer

Answer: Option A
Explanation : 38th parallel, popular name given to latitude 38° N that in East Asia roughly demarcates North Korea and South Korea.


180. The least densely populated country among the following countries of Europe is ________
A. Poland
B. Norway
C. Germany
D. Belgium

Answer

Answer: Option B
Explanation : The least densely populated country among the following countries of Europe is Norway. Norway, with its population of just over 5 million, is one of the three Scandinavian countries. It is ranked as one of the best countries to live in and has one of the lowest crime rates in the world.


181. Which one of the following statements is not true?
A. Gulfs with narrow fronts and wider rears experience high tides
B. Tidal currents take place when a gulf is connected with the open sea by a narrow channel
C. Tidal bore occurs when a tide enters the narrow and shallow estuary of a river
D. The tidal nature of the mouth of the river Hooghly is of crucial importance to Kolkata as port

Answer

Answer: Option A
Explanation : Gulfs with narrow fronts and wider rears experience high tides is incorrect. The gulfs with wide fronts and narrow rears experience higher tides. The in and out movement of water into a gulf through a channels called a tidal current.


182. Inland waterways have many disadvantages. Among them the most common is
A. Presence of water falls and rapids
B. Seasonal freezing and drying up of rivers
C. International controls on traffic
D. Fluctuating volume of traffic

Answer

Answer: Option B
Explanation : Inland waterways have many disadvantages. Among them the most common is Seasonal freezing and drying up of rivers.


183. Which among these is a cold current
A. Florida current
B. Canaries current
C. Brazilian current
D. Norwegian current

Answer

Answer: Option B
Explanation : Canaries current is a cold current. Cold currents flow toward the equator on the eastern side of ocean basins.


184. The Cape Route has recently assumed some importance in the shipping world because ________
A. The trade through this route has greatly increased with the development of South African States
B. The canal tolls on the Suez Canal are heavy
C. Large oil tankers cannot be accommodated in the narrow and comparatively shallow Suez Canal
D. All the above

Answer

Answer: Option D
Explanation : The Cape Route has recently assumed some importance in the shipping world because The trade through this route has greatly increased with the development of South African States, The canal tolls on the Suez Canal are heavy and Large oil tankers cannot be accommodated in the narrow and comparatively shallow Suez Canal.


185. The navigable European rivers are
A. Mississippi Missourie Tennesee St Lawrence
B. Ob Yenisey Volga Rhine
C. Danube Rhone Seine and Elbe
D. Yangtze Sikiang Hwang Ho Chientang

Answer

Answer: Option C
Explanation : The navigable European rivers are Danube, Rhone, Seine and Elbe.


186. The Persian Gulf region is rich in ________.
A. Crude oil reserves
B. Iron ore reserves
C. Bauxite ore reserves
D. Uranium ore reserves

Answer

Answer: Option A
Explanation : The Persian Gulf region is rich in Crude oil reserves. The Persian Gulf countries contain a significant percentage of the world’s oil reserves and production capacity: Proven oil reserves of 728 billion barrels, representing 55 percent of the world’s oil reserves at the end of 2006.


187. Diego Garcia is in ________
A. Arabian Sea
B. Bay of Bengal
C. Indian Ocean
D. Gulf of Aden

Answer

Answer: Option C
Explanation : Diego Garcia is in Indian Ocean. Diego Garcia. Diego Garcia, coral atoll, largest and southernmost member of the Chagos Archipelago, in the central Indian Ocean, part of the British Indian Ocean Territory.


188. You are asked to import Ostrich, Platypus and Kaola Bear. To which country will you opt to go, to get all the three?
A. Australia
B. Japan
C. Canada
D. New Zealand

Answer

Answer: Option A
Explanation : Australia will be opted to get all the three Ostrich, Platypus and Kaola Bear.


189. The canal that connects the upper Hudson at Albany with the Great Lakes at Buffalo is
A. Erie canal
B. Kiel canal
C. North Sea canal
D. Trent and Mersey canal

Answer

Answer: Option A
Explanation : Erie Canal, historic waterway of the United States, connecting the Great Lakes with New York City via the Hudson River at Albany. Taking advantage of the Mohawk River gap in the Appalachian Mountains, the Erie Canal, 363 miles (584 km) long, was the first canal in the United States to connect western waterways with the Atlantic Ocean.


190. The “Horn of Africa” includes ________
A. Ethiopia Djibouti Somalia
B. Namibia Botswana Republic of South Africa
C. Senegal Guinea Somalia
D. Libya Egypt Sudan

Answer

Answer: Option A
Explanation : The “Horn of Africa” includes Ethiopia, Djibouti, Somalia. Horn of Africa, region of eastern Africa. It is the easternmost extension of African land and for the purposes of this article is defined as the region that is home to the countries of Djibouti, Eritrea, Ethiopia, and Somalia, whose cultures have been linked throughout their long history.


191. The population of India has of late been growing very fast because
A. Not enough attention has been paid to methods of population control
B. The death rate has been dropping on account of better medical and health facilities
C. The birth rate has continuously been increasing
D. Industrialization has provided a fillip to growth of population

Answer

Answer: Option B
Explanation : The population of India has of late been growing very fast because the death rate has been dropping on account of better medical and health facilities. Better health, hygiene and sanitation facilities in the town are expected to have lowered the death rate.


192. The smallest amongst the following continents is ________.
A. Africa
B. Antarctica
C. North America
D. South America

Answer

Answer: Option B
Explanation : The smallest amongst the following continents is Antarctica. Below is the list of continents starting from smallest
Continent Area (km2) Australia 8600000 Europe 10180000 Antarctica 14000000 South America 17840000 North America 24709000 Africa 30370000 Asia 44579000


193. The highest coal-producing country in the world is
A. India
B. USA
C. China
D. Russia

Answer

Answer: Option C
Explanation : The highest coal-producing country in the world is China. China has been the biggest coal producer in the past three decades. China produced nearly 3.7 billion tons of coal in 2013 representing 47% of global total coal yield. It is estimated that China uses half of its coal for power generation, which accounts for over 80% of the country’s electricity production.


194. Which one of the following does not border Panama?
A. Costa Rica
B. Pacific Ocean
C. Columbia
D. Venezuela

Answer

Answer: Option D
Explanation : Venezuela does not border Panama. Panama is a country located in Central America, bordering both the Caribbean Sea and the Pacific Ocean, between Colombia and Costa Rica. Panama is located on the narrow and low Isthmus of Panama.


195. The age of the Earth is approximately
A. 1000 million years
B. 4600 million years
C. 3000 million years
D. 100 million years

Answer

Answer: Option B
Explanation : The age of the Earth is approximately 4600 million years. By dating the rocks in Earth’s ever-changing crust, as well as the rocks in Earth’s neighbors, such as the moon and visiting meteorites, scientists have calculated that Earth is 4.54 billion years old, with an error range of 50 million years.


196. Which one of the following is not a plant product?
A. Lac
B. Hemp
C. Linen
D. Flax

Answer

Answer: Option A
Explanation : Lac is not a plant product. Lac is the scarlet resinous secretion of a number of species of lac insects, of which the most commonly cultivated is Kerria lacca.


197. About 50% of the world population is concentrated between the latitudes of ________
A. 5°N and 20°N
B. 20°N and 40°N
C. 40°N and 60°N
D. 20°S and 40°S

Answer

Answer: Option B
Explanation : About 50% of the world population is concentrated between latitude of 20°N and 40°N, because most populous countries and areas like Asia, U.S.A, Europe and India lies with in this latitude.


198. What is the modern name of “Siam”?
A. Myanmar
B. Thailand
C. Philippines
D. Cambodia

Answer

Answer: Option B
Explanation : The south eastern Asian country of Thailand was earlier known by the name of Siam.


199. The general direction or movement or a tropical cyclone in the northern hemisphere is
A. Eastward then poleward
B. Eastward then towards the equator
C. Westward then poleward
D. Westward then towards the equator

Answer

Answer: Option C
Explanation : The general direction or movement or a tropical cyclone in the northern hemisphere is Westward then poleward.


200. Sun’s halo is produced by the refraction of light in
A. Water vapour in Stratus Clouds
B. Ice crystals in Cirro-cumulus clouds
C. Dust particles in stratus clouds
D. Ice crystals in Cirrus clouds

Answer

Answer: Option D
Explanation : Sun’s halo is produced by the refraction of light in Ice crystals in Cirrus clouds. A halo is a ring or light that forms around the sun or moon as the sun or moon light refracts off ice crystals present in a thin veil of cirrus clouds.


201. Which one of the following scholars suggests the earth’s origin is from gases and dust particles?
A. James Jeans
B. H Alfven
C. F Hoyle
D. O Schmidt

Answer

Answer: Option D
Explanation : O Schmidt suggests the earth’s origin is from gases and dust particles. O. Schmidt in 1943 gave Inter-Slellar Dust Hypothesis for the origin of the earth and solar system.


202. Which of the following is called the “Gateway to the Pacific” ?
A. Suez Canal
B. Panama Canal
C. Bering Sea
D. Gulf of Alaska

Answer

Answer: Option B
Explanation : Panama Canal is called the “Gateway to the Pacific”. The Panama Canal is a short cut between the Atlantic Ocean and the Pacific Ocean. Using the Panama Canal enables shipping to avoid the longer and stormy passage round Cape Horn (the southern tip of South America).


203. Which is the largest island of the world?
A. New Guinea
B. Medagascar
C. Greenland
D. Iceland

Answer

Answer: Option C
Explanation : Greenland is the worlds largest island. Greenland has an area of 2,166,086 square km, but a meager population of 56,452. The populations is over 85% Inuit.


204. Which of the following regions are regarded as areas of high density of population?
A. East Asia Central and Southern Europe Tropical deserts
B. Amazon and Congo Basins South-East Asia European Russia
C. Congo Basin and Indonesia Central and Southern Europe European Russia
D. East Asia Southern Asia North Western Europe

Answer

Answer: Option D
Explanation : East Asia, Southern Asia, North Western Europe are regarded as areas of high density of population.


205. Which one of the following can one come across if one travels through the Strait of Malacca?
A. Bali
B. Brunei
C. Java
D. Singapore

Answer

Answer: Option D
Explanation : One can one come across to Singapore if one travels through the Strait of Malacca. Malacca strait is between the Indonesian island of Sumatra and the Malaya Peninsula. Singapore lies on the tip of the Malay Peninsular.


206. The river that runs along the USA-Mexico boundary is
A. Rio-Grande
B. Gila
C. Mississippi
D. Colorado

Answer

Answer: Option A
Explanation : The river that runs along the USA-Mexico boundary is Rio-Grande. It is fifth longest river of North America, and the 20th longest in the world, forming the border between the U.S. state of Texas and Mexico.


207. Which set of two river given below together form the world’s largest delta?
A. The Ganges and the Brahmpautra
B. The Mississippi and the Missouri
C. The Tigris and the Euphrates
D. The Seine and the Rhine

Answer

Answer: Option A
Explanation : The Ganges and the Brahmpautra together form the world’s largest delta. The delta is shared by both India and Bangladesh. It has an area of more than 105,000 sq. km and is one of the most fertile regions of the world. The river is drained by both the Ganges and Brahmaputra, alongwith their tributaries and distributaries.


208. Which is the world’s largest river?
A. The Nile
B. The Amazon
C. The Brahmaputra
D. The Mississippi and Missouri

Answer

Answer: Option B
Explanation : The Amazon is the world’s largest river by volume, but scientists have believed it is slightly shorter than Africa’s Nile. The Brazilian scientists’ 14-day expedition extended the Amazon’s length by about 176 miles (284 kilometers), making it 65 miles (105 kilometers) longer than the Nile.


209. The country with the highest density is ________
A. China
B. Bangladesh
C. India
D. Singapore

Answer

Answer: Option D
Explanation : The country with the highest density is Singapore. Population density of Singapore is highest among countries having population over 1 million.


210. One will not have to pass through the Suez Canal while going from Mumbai to
A. Alexandria
B. Suez
C. Port Said
D. Benghazi

Answer

Answer: Option B
Explanation : One will not have to pass through the Suez Canal while going from Mumbai to Suez.


211. From east to west, which one of the following sequences of cities of Canada is correct?
A. Quebec – Montreal – Ottawa -Toronto
B. Montreal – Quebec – Ottawa -Toronto
C. Quebec – Montreal – Toronto – Ottawa
D. Montreal – Quebec – Toronto – Ottawa

Answer

Answer: Option A
Explanation : From east to west, Quebec – Montreal – Ottawa -Toronto sequences of cities of Canada is correct.


212. The biggest freshwater lake is
A. Titicaca
B. Baikal
C. Superior
D. Victoria

Answer

Answer: Option C
Explanation : The biggest freshwater lake is Superior. Lake Superior is, by surface area, the world’s largest freshwater lake. The surface area of Lake Superior (31,700 square miles or 82,170 square kilometers) is greater than the combined areas of Vermont, Massachusetts, Rhode Island, Connecticut, and New Hampshire.


213. The ________ railway is the longest railway route in the world.
A. Trans-Siberian
B. Canadian-Pacific
C. Trans-Andes
D. Trans-Continental Australia

Answer

Answer: Option A
Explanation : The Trans-Siberian Railway, built 1891-1916, is the longest railway in the world. The longest train service in the world is from Moscow to Pyongyang along the Trans-Siberian route to Ussuriysk near Vladivostok, where it branches south into North Korea.


214. Limestone is an example of
A. Igneous rocks
B. Sedimentary rocks
C. Metamorphic rocks
D. Crystalline rocks

Answer

Answer: Option B
Explanation : Limestone is an example of Sedimentary rocks. Limestone is a carbonate sedimentary rock that is often composed of the skeletal fragments of marine organisms such as coral, foraminifera, and molluscs. Its major materials are the minerals calcite and aragonite, which are different crystal forms of calcium carbonate (CaCO3).


215. The leading producer of coal in former Soviet Union is
A. Donetz basin
B. Kuzentsk basin
C. Karaganda Basin
D. Pechora basin

Answer

Answer: Option A
Explanation : The leading producer of coal in former Soviet Union is Donetz basin. Donets Basin, Ukrainian Donetskyy Baseyn, Russian Donetsky Basseyn, byname Donbas or Donbass, large mining and industrial region of southeastern Europe, notable for its large coal reserves.


216. Which is the largest fishing ground in the world?
A. Japan Sea
B. North Sea
C. North-East Pacific
D. Newfoundland Water

Answer

Answer: Option B
Explanation : North Sea is the largest fishing ground in the world. It stretches from the Bering Sea to East China Sea and the world’s largest as well as greatest fishing ground.


217. Red Indians called this local wind Ice Eater?
A. Mistral
B. Chinook
C. Foehn
D. Haramattan

Answer

Answer: Option B
Explanation : Red Indians called Chinook wind Ice Eater. As these winds move east, they rise over the western slopes of the mountains then descend, becoming extremely dry and much warmer. Air naturally warms as it sinks, but dry air warms almost twice as fast as moist air. The winds can also spread fires easily because of the extremely dry air.


218. Muscovite, Phlogopite and Biotite are
A. Iron ore varieties
B. Mica varieties
C. Coal varieties
D. Manganese varieties

Answer

Answer: Option B
Explanation : Muscovite, Phlogopite and Biotite are Mica varieties. Muscovite, often called ‘white mica’, is the lightest colored mica mineral. Usually colorless, white or silver, occasionally yellow or brown. Phlogopite is a yellow, greenish, or reddish-brown member of the mica family of phyllosilicates. It is also known as magnesium mica or amber mica. Biotite is a common phyllosilicate mineral within the mica group. The high content of iron ions gives it a dark hue. It is sometimes called “iron mica” because it is more iron-rich than phlogopite.


219. The duration of the day and night is always equal at the
A. Equator
B. Greenwhich Line
C. Poles
D. Tropics

Answer

Answer: Option A
Explanation : The duration of the day and night is always equal at the Equator. The equator receives equal day and night throughout the year because it does not tilt in relation to the sun’s location. Since the equator stays in the same place relative to the sun, the level of sunlight received throughout the year is nearly constant.


220. A narrow inlet of sea between cliffs or steep slopes, especially associated with Scandinavia is
A. Bay
B. Gulf Stream
C. Fjord
D. None of these

Answer

Answer: Option C
Explanation : A narrow inlet of sea between cliffs or steep slopes, especially associated with Scandinavia is Fjord. Fjords usually occur where ocean water flows into valleys formed near the coast by glaciers.


221. The largest fish exporting region in the world is ________
A. The South – East Asian Region
B. The North – West Pacific Region
C. The North – East Pacific Region
D. The North – East Atlantic Region

Answer

Answer: Option B
Explanation : The largest fish exporting region in the world is The North – West Pacific Region.


222. Numerically the largest human race in the world is ________
A. Caucasoid
B. Mongoloid
C. Negroid
D. Austroloid

Answer

Answer: Option A
Explanation : Numerically the largest human race in the world is Caucasoid. The Caucasian race (also Caucasoid or Europid) is a grouping of human beings historically regarded as a biological taxon, which, depending on which of the historical race classifications is used, has usually included some or all of the ancient and modern populations of Europe, Western Asia, Central Asia, South Asia, North Africa, and the Horn of Africa.


223. The sea with one of the highest salinities in the world is ________.
A. The Dead Sea
B. The Red Sea
C. The Arabian Sea
D. The Baltic Sea

Answer

Answer: Option A
Explanation : The sea with one of the highest salinities in the world is The Dead Sea. With 33.7% salinity, the Dead Sea is one of the world’s saltiest bodies of water. Although Lake Assal (Djibouti), Garabogazköl and some hypersaline lakes of the McMurdo Dry Valleys in Antarctica (such as Don Juan Pond) have reported higher salinities.


224. In USA rice is grown
A. In Texas
B. In the West
C. In Colorado
D. Nowhere

Answer

Answer: Option A
Explanation : In USA rice is grown In Texas. Primary classification of rice grown are the long, medium or short grain type.


225. The Indian sub-continent lies
A. Mostly in the southern hemisphere and partly in the northern hemisphere
B. Mostly in the northern hemisphere and partly in the southern hemisphere
C. Wholly in the northern hemisphere
D. Wholly in the southern hemisphere

Answer

Answer: Option C
Explanation : The Indian sub-continent lies Wholly in the northern hemisphere. India is considered part of the South Asia region, and it is the seventh-largest country in the world by geographical size. The Northern Hemisphere includes all of North America, the northern reaches of South America, about two-thirds of Africa, all of Asia excluding (parts of Indonesia) and all of Europe.


226. A person of mixed European and Indian blood in Latin America is called a ________
A. Mulatto
B. Mestizo
C. Meiji
D. Mau Mau

Answer

Answer: Option B
Explanation : A person of mixed European and Indian blood in Latin America is called a Mestizo. The term mestizo means mixed in Spanish, and is generally used throughout Latin America to describe people of mixed ancestry with a white European and an indigenous background.


227. A source of renewable minimum cost is power which can be developed with
A. Geothermal power
B. Tidal power
C. Wind mills
D. Biomass power

Answer

Answer: Option D
Explanation : A source of renewable minimum cost is power which can be developed with Biomass power. Biomass is a renewable energy resource derived from the carbonaceous waste of various human and natural activities. It is derived from numerous sources, including the by-products from the timber industry, agricultural crops, raw material from the forest, major parts of household waste and wood.


228. ________ stands at the head of Chesapeake Bay which is known for oysters.
A. Baltimore
B. Boston
C. New Orleans
D. Philadelphia

Answer

Answer: Option A
Explanation : Baltimore stands at the head of Chesapeake Bay which is known for oysters. Known for both its beauty and bounty, the Bay has become “emptier”, with fewer crabs, oysters and watermen in past years.


229. Lava solidifies to form ________.
A. Pumice
B. Basalt
C. Marble
D. Sandstone

Answer

Answer: Option B
Explanation : Lava solidifies to form Basalt. Basalt is an extrusive igneous rock which is formed by rapid cooling of basaltic lava which is exposed near the surface of Earth. Basalt is a fine-grained, hard rock which forms when bits of lava shoot out of volcanoes. It forms when lava reaches the Earth’s surface near an active volcano.


230. The phenomenon known as the “Midnight Sun” is caused by
A. The rotation of the Earth on its axis
B. Sun spots
C. Inclination of the Earth’s axis
D. Nuclear explosions

Answer

Answer: Option C
Explanation : The phenomenon known as the “Midnight Sun” is caused by Inclination of the Earth’s axis. The midnight sun phenomenon, where the Sun does not set for 24 hours or more, occurs only in the polar region. The Earth is tilted at an angle when it revolves around the Sun, which is known as axial tilt.


231. The Savannas or Orinoco Basin are termed as
A. Pampas
B. Steppes
C. Prairies
D. Llanos

Answer

Answer: Option D
Explanation : The Savannas or Orinoco Basin are termed as Llanos. llanos is a tropical grassland while the remaining three are temperature grasslands.


232. Biome represents ________.
A. A desert vegetation
B. A major ecological group of plants
C. A large ecological region characterised by similar vegetation and climate
D. A major ecological group of animals

Answer

Answer: Option C
Explanation : Biome represents a large ecological region characterised by similar vegetation and climate. Biomes are large-scale environments that are distinguished by characteristic temperature ranges and amounts of precipitation. These two variables affect the types of vegetation and animal life that can exist in those areas. Because each biome is defined by climate, the same biome can occur in geographically distinct areas with similar climates


233. A heavenly body which takes nearly the same time to complete one rotation and one revolution is
A. An asteroid
B. A comet
C. Moon
D. Mars

Answer

Answer: Option C
Explanation : A heavenly body which takes nearly the same time to complete one rotation and one revolution is Moon.


234. River Rhine of Europe drains into
A. North Sea
B. Black Sea
C. Caspian Sea
D. Adriatic Sea

Answer

Answer: Option A
Explanation : River Rhine of Europe drains into North Sea. It flows from two small headways in the Alps of east- central Switzerland north and west to the North Sea, into which it drains through the Netherlands.


235. In Britain the Grand Union Canal is running from
A. Liverpool to Hull
B. Cardiff to Birmingham
C. Glasgow to Edinburgh
D. West Midlands to the London area

Answer

Answer: Option D
Explanation : In Britain the Grand Union Canal is running from West Midlands to the London area. The Grand Union Canal Race (GUCR) is run from the centre of Birmingham to the centre of London along the Grand Union Canal.


236. The ‘land of eternal spring’ are certain parts of
A. Tundra region
B. Highland region
C. Mediterranean region
D. Hot Wet Equatorial region

Answer

Answer: Option D
Explanation : The ‘land of eternal spring’ are certain parts of Hot Wet Equatorial region. Wet equatorial climate, major climate type of the Köppen classification characterized by consistently high temperatures (around 30 °C [86 °F]), with plentiful precipitation (150–1,000 cm [59–394 inches]), heavy cloud cover, and high humidity, with very little annual temperature variation.


237. Volcanic eruption does not occur in the
A. Black Sea
B. Caspian Sea
C. Baltic Sea
D. Caribbean Sea

Answer

Answer: Option C
Explanation : Volcanic eruption does not occur in the Baltic Sea. This Baltic Sea anomaly looks like a harbor with a pair of stairways and a couple of slot- like boat launches that lead to deeper water.


238. Which one of the following is correctly matched?
A. Canary current – Mediterranean Sea
B. Falkland current – Arabian Sea
C. Gulf Stream – Pacific Ocean
D. Labrador Current – North Atlantic Ocean

Answer

Answer: Option D
Explanation : The Labrador Current is a cold current in the North Atlantic Ocean which flows from the Arctic Ocean south along the coast of Labrador and passes around Newfoundland, continuing south along the east coast of Nova Scotia.


239. Which of the following is correctly matched?
A. Eskimo – Canada
B. Oran – Sweden
C. Pygmies – Pampas
D. Gonds – Africa

Answer

Answer: Option A
Explanation : Eskimo, any member of a group of peoples who, with the closely related Aleuts, constitute the chief element in the indigenous population of the Arctic and subarctic regions of Greenland, Canada, the United States, and far eastern Russia (Siberia).


240. Which calendar is being universally followed?
A. Christian Era
B. Julian Calendar
C. Gregorian Calendar
D. Saka Era

Answer

Answer: Option C
Explanation : Gregorian Calendar is being universally followed. The Gregorian calendar is a solar calendar with 12 months of 28–31 days each. A regular Gregorian year consists of 365 days, but in certain years known as leap years, a leap day is added to February.


241. Which of the following is India’s north station?
A. Dakshin Gangotri
B. Maitri
C. Himadri
D. None of These

Answer

Answer: Option C
Explanation : Himadri is India’s north station. Inaugurated on 1st July 2008 by Ministry of Earth Sciences, India, Himadri is first Arctic station of India. It is located at a distance of 1,200 kilometres from the North Pole.


242. The Density of population in any region is measured by numbers of
A. People
B. Children
C. Families
D. Houses

Answer

Answer: Option A
Explanation : The Density of population in any region is measured by numbers of People. It is expressed as the number of people divided by the total area. Population density
= No. of people/ Unit Area


243. Which of the following pair is not correctly matched?
A. Hevea tree – Brazil
B. Sumatra storm – Malaysia
C. River kayan – Borneo
D. Dekke toba fish – Brazil

Answer

Answer: Option D
Explanation : Dekke toba fish is a fish found in Lake Toba in Sumatra, Indonesia.


244. Which of the followings is not correctly matched?
A. Indonesia – Jakarta
B. Maldives – male
C. North Korea – Seoul
D. Zimbabwe – Harare

Answer

Answer: Option C
Explanation : In all other 3 options countries are correctly matched with their capitals. Seoul is the capital of South Korea whereas Pyongyang is the capital of North Korea.


245. Which one of the following pairs is not correctly matched?
A. Abuja : Nigeria
B. Ashkhabad : Turkmenistan
C. Pretoria : South Africa
D. Rabat : Yemen

Answer

Answer: Option C
Explanation : Pretoria, city in Gauteng province and administrative capital of the Republic of South Africa.


246. What is the capital of Rwanda?
A. Libreville
B. Kigali
C. Copenhagen
D. Bogota

Answer

Answer: Option B
Explanation : The East African country Rwanda is the land locked country with capital Kigali.


247. The largest fresh water body (volume-wise) in the world is lake
A. Baikal
B. Titicaca
C. Superior
D. Victoria

Answer

Answer: Option C
Explanation : The largest fresh water body (volume-wise) in the world is lake Superior (82,100 km2).


248. Beaufort scale is an internationally recognised scale for describing
A. Earthquake intensity
B. Noise level
C. Wind speed
D. Radioactive intensity

Answer

Answer: Option C
Explanation : Beaufort scale is an internationally recognised scale for describing Wind speed. Beaufort scale, in full Beaufort wind force scale, scale devised in 1805 by Commander (later Admiral and Knight Commander of the Bath) Francis Beaufort of the British navy for observing and classifying wind force at sea. Originally based on the effect of the wind on a full-rigged man-of-war, in 1838 it became mandatory for log entries in all ships in the Royal Navy.


249. Greenland is the largest island of the world. It is an integral part of
A. Norway
B. North America
C. Denmark
D. Canada

Answer

Answer: Option C
Explanation : Greenland is a territory of Denmark lying between North Atlantic Ocean & Arctic Ocean in Northern Hemisphere.


250. Quartzite is metamorphosed from
A. Limestone
B. Obsidian
C. Sandstone
D. Shale

Answer

Answer: Option C
Explanation : Quartzite is metamorphosed from Sandstone. Quartzite is a nonfoliated metamorphic rock composed almost entirely of quartz. It forms when a quartz-rich sandstone is altered by the heat, pressure, and chemical activity of metamorphism. These conditions recrystallize the sand grains and the silica cement that binds them together.


251. The process which leads to the formation of clouds is known as
A. Evaporation
B. Condensation
C. Cloud-transpiration
D. Precipitation

Answer

Answer: Option B
Explanation : The process which leads to the formation of clouds is known as Condensation. Clouds are an interesting natural phenomenon which one can observe everyday. Interestingly, there are a variety of clouds and they are all made up of the same substances, water or ice or both, in a condensed form.


252. ________ is a land locked country.
A. Bangladesh
B. Myanmar
C. Nepal
D. Sri Lanka

Answer

Answer: Option C
Explanation : Nepal is a land locked country. Nepal is a landlocked country (between India and China), approximately the size of Illinois. However, it has an extremely diverse topography, with the Himalayas in the north, the plains in the south and everything else in between.


253. Which one of the following is a land-locked country?
A. Angola
B. Gabon
C. Tanzania
D. Zimbabwe

Answer

Answer: Option D
Explanation : Zimbabwe is also a landlocked country in Southern Africa. It is located between Zambezi and Limpopo rivers and bordered by Mozambique, South Africa, Botswana, and Zambia.


254. Trincomalee is a port situated on the
A. East coast Tamil Nadu
B. East coast of Sri Lanka
C. East coast Andhra Pradesh
D. West coast of Sri Lanka

Answer

Answer: Option B
Explanation : Trincomalee is a port situated on the East coast of Sri Lanka. Trincomalee is a port city on the northeast coast of Sri Lanka. Located by Trincomalee, Sri Lanka, in the heart of the Indian Ocean, its strategic importance has shaped its history.


255. The Australian state which is an island is ________
A. Queensland
B. Java
C. Tasmania
D. New Guinea

Answer

Answer: Option C
Explanation : The Australian state which is an island is Tasmania. It is located 240 km (150 mi) to the south of the Australian mainland, separated by Bass Strait.


256. Canals serve as passage of communication in the city of ________ in Italy.
A. Rome
B. Venice
C. Turin
D. Genoa

Answer

Answer: Option B
Explanation : Canals serve as passage of communication in the city of Venice in Italy. Venice is almost defined by its canals, with more than 150 waterways meandering through the city and traversed by over 400 bridges.


257. The capital city of Kenya is ________.
A. Nairobi
B. Entebbe
C. Cairo
D. Port Louis

Answer

Answer: Option A
Explanation : The capital city of Kenya is Nairobi. It is situated in the south-central part of the country, in the highlands at an elevation of about 5,500 feet (1,680 metres). The city lies 300 miles (480 km) northwest of Mombasa, Kenya’s major port on the Indian Ocean.


258. Which city is called the ‘City of Perpetual Spring’?
A. Colombo
B. Quito
C. Lisbon
D. Berlin

Answer

Answer: Option B
Explanation : Quito city is called the ‘City of Perpetual Spring’. It is called the city of eternal spring due to the combination of its location–just a few south to the equator and it is 9000 mts altitude. The result is a subtropical highland climate which is neither too cold nor too warm.


259. The velocity of wind is related to
A. Revolution of the Earth
B. Rotation of the Earth
C. Temperature
D. Pressure gradient

Answer

Answer: Option D
Explanation : The velocity of wind is related to Pressure gradient. The pressure difference between two locations is called a pressure gradient, and the force that actually moves air as wind is called the pressure gradient force.


260. Which of the following is a low-density area?,I. Equatorial forest,II. Tropical deserts,III. Eastern Asia,IV. North-eastern America
A. I II and IV
B. I and II
C. II and III
D. I III and IV

Answer

Answer: Option B
Explanation : Equatorial forest and Tropical deserts are low-density area.


261. Albert Canal links ________ and ________
A. Manchester and Antwerp
B. Antwerp and Frankfurt
C. Antwerp and Liege
D. Manchester and Liege

Answer

Answer: Option C
Explanation : Albert Canal links Antwerp and Liege. Albert Canal, waterway connecting the cities of Antwerp and Liège in Belgium. The Albert Canal is about 130 km (80 miles) long.


262. What is the usual altitude of the plains?
A. Sea level
B. Less than 500 metres above sea level
C. Less than 600 metres above sea level
D. Less than 800 metres above sea level

Answer

Answer: Option B
Explanation : The usual altitude of the plains less than 500 metres above sea level.


263. Outwash plain is a depositional feature associated with
A. Rivers
B. Wind
C. Glacier
D. Wave

Answer

Answer: Option C
Explanation : Outwash plain is a depositional feature associated with Glacier. It is a plain formed of glacial sediments deposited by meltwater outwash at the terminus of a glacier. As it flows, the glacier grinds the underlying rock surface and carries the debris along.


264. Strait of Malacca separates ________
A. Sumatra and Malaysia
B. Java and Brunei
C. Sumatra and Java
D. Malaysia and Brunei

Answer

Answer: Option A
Explanation : Strait of Malacca separates Sumatra and Malaysia. The Strait of Malacca (also called the Straits of Malacca, and in Malay Selat Melaka) is a narrow stretch of water between Peninsular Malaysia (West Malaysia) and the Indonesian island of Sumatra.


265. Ladang refers to
A. Shifting cultivation
B. Plantation agriculture
C. Subsistence type of agriculture
D. Dry farming

Answer

Answer: Option A
Explanation : Ladang refers to Shifting cultivation. Shifting cultivation is an agricultural system in which areas of land are cultivated for a short time. Then they are left to grow back their natural vegetation, while the farmer moves to another area.


266. Amur, one of the longest rivers in the worlds is in
A. Africa
B. South America
C. Asia
D. Europe

Answer

Answer: Option C
Explanation : Amur, one of the longest rivers in the worlds is in Asia. Amur – Argun, Asia – 4,444km (2,763 miles). The river forms the border between the Russian Far East and Northeastern China.


267. Fjords abound the coast of
A. Sweden
B. France
C. Norway
D. Germany

Answer

Answer: Option C
Explanation : Fjords abound the coast of Norway. Fjord Norway has stunning scenery with deep blue fjords, flowing waterfalls, and sharp, snow-capped mountains that tower high above the water.


268. Jute requires
A. Hot and humid climate
B. Cold climate
C. Hot and dry climate
D. Mediterranean climate

Answer

Answer: Option A
Explanation : Jute requires Hot and humid climate. Jute is a tropical plant and needs a hot and humid climate. It requires high temperature ranging between 25 °C and 35 °C. It grows well in regions of heavy rainfall, say about 120 cm to 150 cm. About 85 per cent relative humidity is needed during the period of its growth.


269. Takla Makan desert is situated in
A. Mongolia
B. Tibet
C. Sinkiang
D. South West China

Answer

Answer: Option C
Explanation : Takla Makan desert is situated in Sinkiang. It is a great desert of Central Asia and one of the largest sandy deserts in the world. The Takla Makan occupies the central part of the Tarim Basin in the Uygur Autonomous Region of Xinjiang, western China.


270. Kolkhoz and Sovkhoz are
A. Farming organization on agriculture
B. Fertile alluvial lands
C. High yielding variety seeds
D. Breeds of cattle

Answer

Answer: Option A
Explanation : Kolkhoz and Sovkhoz are farming organization on agriculture. They are collective farms. All farms are organized as either collective (kolkhoz) or state (sovkhoz) farms.


271. Lake Manasarovar is in
A. Sikkim
B. Tibet
C. Kashmir
D. Himachal Pradesh

Answer

Answer: Option B
Explanation : Lake Manasarovar is in Tibet. Lake Manasarovar is Tibet’s holiest lake. It is located in Ngari prefecture over 1200 kilometers from Lhasa.


272. The Earth’s axis is tilted at an angle of ________ degrees from the perpendicular to its orbital plane.
A. 17.5°
B. 23.5°
C. 36°
D. 45°

Answer

Answer: Option B
Explanation : Earth has seasons because our planet’s axis of rotation is tilted at an angle of 23.5 degrees relative to our orbital plane – the plane of Earth’s orbit around the sun. The tilt in the axis of the Earth is called its obliquity by scientists.


273. The largest Inland Sea is the
A. Black Sea
B. Dead Sea
C. Caspian Sea
D. Aral Sea

Answer

Answer: Option C
Explanation : The Caspian Sea is the world’s largest inland water body. The sea covers roughly 143,200 square miles (371,000 square kilometers) and borders five countries: Iran, Russia, Kazakhstan, Turkmenistan and Azerbaijan.


274. The world’s largest port is ________.
A. Mumbai
B. Sydney
C. New York/New Jersey
D. Singapore

Answer

Answer: Option C
Explanation : The world’s largest port is New York/New Jersey. The Port of New York and New Jersey is the busiest container port on the East Coast of the United States. Its terminals are located throughout the New York metropolitan area including Newark, New Jersey. The largest terminal in the port is operated by Maher Terminals.


275. Tibet, the highest plateau in the world, is a
A. Piedmont plateau
B. Intermontane plateau
C. Continental plateau
D. Volcanic plateau

Answer

Answer: Option B
Explanation : Tibet, the highest plateau in the world, is a Intermontane plateau. Tibetan plateau is the highest plateau in the world at the height of 4599-5000 Meters.


276. The most valuable cash crop of Brazil is
A. Tea
B. Coffee
C. Cardamom
D. Spices

Answer

Answer: Option B
Explanation : The most valuable cash crop of Brazil is Coffee. Brazil eventually became the world’s largest exporter of coffee, soybeans, beef, and crop-based ethanol. Brazil exported 37 thousand tons of processed cashew nuts valued at 187.7 thousand USD in 2012.


277. The North Pole is always in the light
A. From September 23 to March 21
B. From March 21 to September 23
C. From June 21 to December 22
D. On June 21

Answer

Answer: Option B
Explanation : The North Pole is always in the light from March 21 to September 23.


278. Lesotho is ________
A. An island in Mediterranean Sea
B. An important seaport in Tanzania
C. A country completely surrounded by South Africa
D. A mountain peak in Zambia

Answer

Answer: Option C
Explanation : Lesotho is a country completely surrounded by South Africa. Lesotho is a small land-locked country, completely surrounded by its neighbor South Africa.


279. In which country will you find the river Po?
A. China
B. Italy
C. Sri Lanka
D. Zambia

Answer

Answer: Option B
Explanation : In Italy river Po is found. Po River. Po River, Latin Padus, longest river in Italy, rising in the Monte Viso group of the Cottian Alps on Italy’s western frontier and emptying into the Adriatic Sea in the east after a course of 405 miles (652 km).


280. Which or the following is a rift valley?
A. The Amazon Valley
B. The Thames Valley
C. The Rhine Valley
D. The Mississippi Valley

Answer

Answer: Option C
Explanation : The Rhine is a rift valley. The Rhine Rift valley shape is the result of the rifting which begins some 60 million years ago. It is known as a hot spot due to the eruption of the hot mantle into the crust, which caused the crust uplift and the mantle overflow with the development of the Black Forest, the Vosges and the Rhine Rift valley.


281. Latex is a name for
A. A milk product
B. Latent heat
C. Crude rubber
D. Refined honey

Answer

Answer: Option C
Explanation : Latex is a name for Crude rubber. Natural rubber, also called India rubber or caoutchouc, as initially produced, consists of polymers of the organic compound isoprene, with minor impurities of other organic compounds, plus water. Currently, rubber is harvested mainly in the form of the latex from the rubber tree or others.


282. Simla is cooler than Amritsar although they are on the same latitude. This is because
A. Shimla is further north
B. Shimla is at greater height above sea level than Amritsar
C. Shimla is farther from the Equator
D. Their longitudes differ

Answer

Answer: Option B
Explanation : Simla is cooler than Amritsar although they are on the same latitude. This is because Shimla is at greater height above sea level than Amritsar.


283. The continents of the world are mainly composed of
A. Core
B. Mantal
C. Sial
D. Sima

Answer

Answer: Option C
Explanation : The continents of the world are mainly composed of Sial. The main mineral constituents of the continental mass are silica and alumina.It is thus called sial (si-silica and al-alumina).


284. First railway line in the world, was laid in ________
A. Western U.S.
B.
C. North-East England
D. Federal Republic of Germany
E. Russia

Answer

Answer: Option B
Explanation : First railway line in the world, was laid in North-East England. The Stockton and Darlington Railway (S&DR) was the first railway company that operated in north-east England from 1825 to 1863. Its first line connected collieries near Shildon with Stockton-on-Tees and Darlington, and was officially opened on 27 September 1825.


285. The country which is commonly known as ‘The Land of Golden Fleece’ is ________
A. Australia
B. Britain
C. France
D. Japan

Answer

Answer: Option A
Explanation : The country which is commonly known as ‘The Land of Golden Fleece’ is Australia. Australia has been called “The Land of the Golden Fleece,” because of the quantity of wool produced there. Georgia, one of the Caucasian nations is also called “The Land of the Golden Fleece.”


286. Which of the following is not an island?
A. Cuba
B. Greenland
C. Ireland
D. Sweden

Answer

Answer: Option D
Explanation : Sweden is not an island. Sweden, country located on the Scandinavian Peninsula in northern Europe. The name Sweden was derived from the Svear, or Suiones, a people mentioned as early as 98 CE by the Roman author Tacitus.


287. The largest amongst the following seas is ________.
A. The Red Sea
B. The South China Sea
C. The Caribbean Sea
D. The Andaman Sea

Answer

Answer: Option B
Explanation : The largest amongst the following seas is The South China Sea. The South China Sea is a marginal sea that is part of the Pacific Ocean, encompassing an area from the Karimata and Malacca Straits to the Strait of Taiwan of around 3,500,000 square kilometres.


288. People living in South India generally have more features of
A. Caucasoid
B. Mongoloid
C. Negroid
D. None of these

Answer

Answer: Option C
Explanation : People living in South India generally have more features of Negroid. Negroid (Black) race. They ar mainly black people.


289. Panama Canal has locks alone its path in order to
A. Regulate traffic
B. Permit check by customs staff
C. Control the level of water
D. Collect canal dues

Answer

Answer: Option C
Explanation : Panama Canal has locks alone its path in order to Control the level of water. The locks have been called the structural triumph of the Panama Canal and are a unique aspect of the waterway. At the time of their construction, their overall mass, dimensions and innovative design surpassed any similar existing structures, and they are still considered to be an engineering wonder of the world.


290. Hurricanes or typhoons (tropical cyclones) develop and mature
A. Over land bodies only
B. In a belt 35°-65° N and S latitudes
C. Anywhere in the tropical zone
D. Over water bodies only

Answer

Answer: Option D
Explanation : Hurricanes or typhoons (tropical cyclones) develop and mature over water bodies only. Hurricane and typhoon is defined as a tropical cyclone in the western Pacific. Hurricanes and Typhoons generally track in a westward or northern direction.


291. Pygmies are people living in
A. Congo forests
B. Namibia
C. Kenya
D. Cape Province

Answer

Answer: Option A
Explanation : Pygmies are people living in Congo forests. The African rainforest is home to some of the most celebrated tribal people, the so-called “Pygmies” of the Ituri forest in northern Congo. The tallest of these people, known as the Mbuti, rarely exceed five feet (1.5 m).


292. The deflection of the wind to the right in the northern hemisphere and to Its left in the southern hemisphere is caused by the
A. Rotation of the Earth
B. Revolution of the Earth
C. Uneven heating of the Earth
D. All the above

Answer

Answer: Option A
Explanation : The deflection of the wind to the right in the northern hemisphere and to Its left in the southern hemisphere is caused by the Rotation of the Earth. Because the Earth rotates on its axis, circulating air is deflected toward the right in the Northern Hemisphere and toward the left in the Southern Hemisphere. This deflection is called the Coriolis effect.


293. Mixed farming involves
A. Both crops and livestock
B. Growing a series of different crops
C. Specialised cultivation of vegetables and fruits
D. Pooling land voluntarily and managing it jointly under a democratic constitution

Answer

Answer: Option A
Explanation : Mixed farming involves Both crops and livestock. Mixed farming is an agricultural system in which a farmer conducts different agricultural practice together, such as cash crops and livestock. The aim is to increase income through different sources and to complement land and labour demands across the year.


294. Moraine is
A. A tribe inhabiting the equatorial region
B. The debris of fragments of rock material brought down with the movement of a glacier
C. A salt water lake in Arabia
D. A nocturnal animal found in Amazon forests

Answer

Answer: Option B
Explanation : Moraine is the debris of fragments of rock material brought down with the movement of a glacier. Moraines are made up of the pieces of rock that are shattered by frost action, imbedded in the glaciers & brought down the valley.


295. Mount Kailas is in
A. Nepal
B. Tibet
C. Sikkim
D. Bhutan

Answer

Answer: Option B
Explanation : Mount Kailas is in Tibet. Mount Kailash is called Gangs Rinpoche in Tibetan language meaning “precious jewel of snows”. The name originates from the year-round snow on its peak and its historical religious connections. Mt. Kailash is the highest peak in the massive Gangdise Mountain Range with an altitude over 6,600m (21,654 ft).


296. The construction of Panama Canal in 1913 eliminated the long and hazardous voyage
A. Round the stormy Cape Horn
B. Round the Cape of Good Hope
C. Between North and South America
D. In the stormy Atlantic Ocean

Answer

Answer: Option A
Explanation : The construction of Panama Canal in 1913 eliminated the long and hazardous voyage Round the stormy Cape Horn. Cape Horn has long been known as a sailor’s graveyard due to the strong winds and currents, large waves and icebergs.


297. ________ and ________ are called the ‘Low countries’ of Europe.
A. Spain and Portugal
B. Holland and Belgium
C. England and Ireland
D. Denmark and Germany

Answer

Answer: Option B
Explanation : Holland and Belgium are called the ‘Low countries’ of Europe because they are globally Europe’s flattest part.


298. Which of the following is a Great Circle?
A. The Equator
B. The 0º longitude
C. 180º longitude
D. Antarctic region

Answer

Answer: Option A
Explanation : The Equator is a Great Circle. Most importantly, the equator is the largest circle of all the lines of latitudes. this is why equator is the most important line of the latitude.


299. Doldrum is an area of ________.
A. Low rainfall
B. Low temperature
C. Low pressure
D. High winds

Answer

Answer: Option C
Explanation : Doldrum is an area of Low pressure. Doldrums are the the regions surrounding the earth, north of the equator. They are areas where there is absolutely no air movement and an intense low pressure exists.


300. Railways are best suited for
A. Passenger traffic for distances upto 1000 km
B. Freight traffic for bulky commodities
C. Urban commuters
D. Fast passenger movement

Answer

Answer: Option B
Explanation : Railways are best suited for Freight traffic for bulky commodities. There are a number of advantages when using this transportation method. First and foremost is the fuel cost reduction. Trains are a far more fuel efficient method of transportation compared to trucks, considering the amount of cargo a single train can transport. Consequently the use of a highway train is also less damaging for the environment.


301. Sunspots generally wax and wane over a/an
A. 11-year cycle
B. 25-year cycle
C. 3-year cycle
D. 15-year cycle

Answer

Answer: Option A
Explanation : Sunspots generally wax and wane over an 11-year cycle. Centuries of observations have shown that the number of sunspots waxes and wanes over a roughly 11-year period. Sunspots exhibit other predictable behavior.


302. Malaspine Glacier is in
A. Karakoram Range
B. Alps
C. Alaska
D. Hindukush

Answer

Answer: Option C
Explanation : Malaspine Glacier is in Alaska. Malaspina Glacier in southeastern Alaska is considered the classic example of a piedmont glacier. Piedmont glaciers occur where valley glaciers exit a mountain range onto broad lowlands, are no longer laterally confined, and spread to become wide lobes. Malaspina Glacier is actually a compound glacier, formed by the merger of several valley glaciers, the most prominent of which seen here are Agassiz Glacier (left) and Seward Glacier (right). In total, Malaspina Glacier is up to 65 kilometers (40 miles) wide and extends up to 45 kilometers (28 miles) from the mountain front nearly to the sea.


303. Which of the following is the smallest in terms of area?
A. Vatican City
B. Monaco
C. Nauru
D. San Marino

Answer

Answer: Option A
Explanation : Vatican City is the smallest in terms of area. Vatican City is the smallest country in the world, measuring just 0.2 square miles, almost 120 times smaller than the island of Manhattan. Situated on the western bank of the Tiber River, Vatican City’s 2-mile border is landlocked by Italy.


304. Which one of the following is present in the largest amount in terms of per cent by man In the earth’s crust?
A. Silicon
B. Oxygen
C. Carbon
D. Calcium

Answer

Answer: Option B
Explanation : 46.6% Oxygen is the most abundant element in the Earth’s crust. Oxygen makes up 467,100 ppm (parts per million) of the Earth’s crust, or 46.6%. It exists as a major compound of the silicate minerals where it combines with other elements.


305. Which is the smallest (in area) member of United Nations?
A. Papua New Guinea
B. Cuba
C. Vatican
D. Seychelles

Answer

Answer: Option D
Explanation : With an archipelago of 115 islands in Indian Ocean in East of African continent lies Seychelles which is the smallest among the other given options.


306. The biggest railway Junction in the United States is
A. New York
B. Chicago
C. St Louis
D. Washington DC

Answer

Answer: Option B
Explanation : The biggest railway Junction in the United States is Chicago. Chicago Union Station is a major railroad station that opened in 1925 in Chicago, Illinois, replacing an earlier station built in 1881.


307. The meeting or warm and cold currents in the ocean gives rise to
A. Frost
B. Rain
C. Fog
D. Cloud-burst

Answer

Answer: Option C
Explanation : The meeting or warm and cold currents in the ocean gives rise to Fog. The air above warm ocean currents is warm and air above cold ocean currents is cool. When cold and warm ocean current meet, the warm ocean current condenses after getting in contact with cold air and it results into fog.


308. Durand line is the border common to which two countries?
A. India and China
B. China and Afghanistan
C. India and Tibet
D. Pakistan and Afghanistan

Answer

Answer: Option D
Explanation : Durand line is the border common to Pakistan and Afghanistan. It’s the result of an agreement between Sir Mortimer Durand, a secretary of the British Indian government, and Abdur Rahman Khan, the emir, or ruler, of Afghanistan.


309. Vernal equinox occurs on the
A. 23rd September
B. 21st March
C. 21st June
D. 22nd December

Answer

Answer: Option B
Explanation : Vernal equinox occurs on the 21st March. The March equinox is known as the vernal equinox in the Northern Hemisphere and as the autumnal equinox in the Southern. On the Gregorian calendar, the Northward equinox can occur as early as 19 March or as late as 21 March at Greenwich.


310. The ‘Old Faithful’ in Yellowstone National Park, Wyoming is a/an
A. Hot spring
B. Extinct volcano
C. Geyser
D. Oak tree

Answer

Answer: Option C
Explanation : The ‘Old Faithful’ in Yellowstone National Park, Wyoming is a Geyser. Old Faithful is a cone geyser located in Yellowstone National Park in Wyoming, United States. It was named in 1870 during the Washburn-Langford-Doane Expedition and was the first geyser in the park to receive a name. It is a highly predictable geothermal feature, and has erupted every 44 to 125 minutes since 2000.


311. Mount Kilimanjaro is in
A. Alaska
B. Argentina
C. Chile
D. Tanzania

Answer

Answer: Option D
Explanation : Mount Kilimanjaro is in Tanzania. Mount Kilimanjaro is located in Tanzania, a country in East Africa. The mountain is one of the seven summits (the highest peaks on each of the seven continents), and rises 5,895 meters or 19,340 feet above sea level. It is the tallest freestanding mountain (not part of a mountain range) in the world.


312. In which of the following country, the first iron bridge was constructed?
A. England
B. Italy
C. Greece
D. France

Answer

Answer: Option A
Explanation : Industrial Revolution first occured in England. Then iron was started to be used on wide scale for building bridges.


313. The north flowing rivers of Siberia are not of much use for navigation mainly because
A. They are too shallow in the dry season
B. The lower courses remain frozen for many months
C. They flow through uninhabited areas
D. They lack navigation facilities to ensure safety

Answer

Answer: Option B
Explanation : The north flowing rivers of Siberia are not of much use for navigation mainly because The lower courses remain frozen for many months.


314. The shallow waters of continental shelves are rich fishing grounds. An example of such area is
A. North Sea
B. Baltic Sea
C. Mediterranean Sea
D. Red Sea

Answer

Answer: Option A
Explanation : The shallow waters of continental shelves are rich fishing grounds. An example of such area is North Sea. Fishing in the North Sea is concentrated in the southern part of the coastal waters.


315. The world’s major fishing grounds are located along the eastern shores of continents between 40º N and 50º N latitudes where cool and warm currents converge. An example or such area is
A. The Grand Banks off Newfoundland
B. The Dogger Bank in the North Sea
C. The coastal region of Norway
D. Fisheries off the coast of Peru

Answer

Answer: Option A
Explanation : The world’s major fishing grounds are located along the eastern shores of continents between 40º N and 50º N latitudes where cool and warm currents converge. An example or such area is The Grand Banks off Newfoundland.


316. Shortest air route from New Delhi to Vancouver will be ________
A. New Delhi – Bangkok – Tokyo – Vancouver
B. New Delhi – London – New York – Vancouver
C. New Delhi – Moscow – London – Vancouver
D. New Delhi – Paris – New York – Vancouver

Answer

Answer: Option A
Explanation : Shortest air route from New Delhi to Vancouver will be New Delhi – Bangkok – Tokyo – Vancouver.


317. Wind erosion is dominant in
A. Tropical regions
B. Temperate regions
C. Deserts
D. Mediterranean regions

Answer

Answer: Option C
Explanation : Wind erosion is dominant in Deserts. Wind is a powerful force that has the ability to shape a landscape. In deserts, wind action changes the landscape through wind erosion in the form of abrasion and deflation.


318. Why does temperature increase with increase in height in Stratosphere?
A. Closeness of the sun
B. Decrease in atmospheric pressure
C. Concentration of ozone
D. None of these

Answer

Answer: Option C
Explanation : Concentration of ozone cause temperature increase with increase in height in Stratosphere. The reason is that the direct heat source for the stratosphere is the Sun. A layer of ozone molecules absorbs solar radiation, which heats the stratosphere.


319. Isolines are lines Joining places having
A. Equal temperature
B. Equal diurnal range of temperature
C. Equal salinity
D. Equal cloud cover

Answer

Answer: Option C
Explanation : Isolines are lines Joining places having Equal salinity. Isolines are lines drawn to link different places that share a common value. The prefix ‘iso’ is a greek word meaning equal, so an isoline must be a line joining equal points.


320. Where is ‘Ninety East Ridge’situated?
A. Pacific Ocean
B. Indian Ocean
C. Atlantic Ocean
D. Arctic Ocean

Answer

Answer: Option B
Explanation : ‘Ninety East Ridge’ is situated in Indian Ocean. The Ninetyeast Ridge is thought to have originated from hot spot volcanic activity now located at the Kerguelen Islands near Antarctica.


321. Quinine is obtained from the
A. Mulberry tree
B. Eucalyptus tree
C. Cinchona plant
D. Cocoa plant

Answer

Answer: Option C
Explanation : Quinine is obtained from the Cinchona plant. Quinine, drug obtained from cinchona bark that is used chiefly in the treatment of malaria, an infection caused by the protozoan parasite Plasmodium, which is transmitted to humans by the bite of various species of mosquitoes.


322. Through which one of the following Straits does a tunnel connect the United Kingdom and France?
A. Davis Strait
B. Denmark Strait
C. Strait of Dover
D. Strait of Gibraltar

Answer

Answer: Option C
Explanation : Through Strait of Dover a tunnel connect the United Kingdom and France. Strait of Dover connects United Kingdom and France. It is about 32 km in length.


323. Which among the following is the greatest inland waterway in Europe?
A. Thames
B. Elbe
C. Rhine
D. Volga

Answer

Answer: Option C
Explanation : Rhine is the greatest inland waterway in Europe. The Main-Danube waterway connecting the Rhine with the Black Sea was completed in 1992 and provides a route for traffic between eastern and western Europe through Germany, accommodating craft of 1,350 tons throughout its length.


324. Atlantic and Pacific Oceans are connected by ________ canal
A. Suez
B. Panama
C. Kiel
D. Soo

Answer

Answer: Option B
Explanation : Atlantic and Pacific Oceans are connected by Panama canal. The canal cuts across the Isthmus of Panama and is a key conduit for international maritime trade.


325. The hair or Nordic races is generally
A. Blond
B. Woolly
C. Straight
D. All of these

Answer

Answer: Option A
Explanation : The hair or Nordic races is generally Blond. There are two theories as to why many Scandinavians have blonde hair. One popular theory is it was caused by genetic mutations as a result of the lack of sunlight once humans began to spread north.


326. Which one of the following is the correct sequence of the given towns of Pakistan while moving from the North towards the South ?
A. Islamabad – Gujranwala – Peshawar – Multan
B. Peshawar – Gujranwala – Multan – Islamabad
C. Peshawar – Islamabad – Gujranwala – Multan
D. Islamabad – Multan – Peshawar – Gujranwala

Answer

Answer: Option C
Explanation : The correct sequence of the given towns of Pakistan while moving from the North towards the South is Peshawar – Islamabad – Gujranwala – Multan.


327. By what name the Third World Countries are now commonly referred to?
A. Least developed countries
B. Underdeveloped countries
C. Developing countries
D. Underdeveloped countries

Answer

Answer: Option C
Explanation : The Third World Countries are now commonly referred to as Developing countries. During the Cold War, the term Third World referred to the developing countries of Asia, Africa, and Latin America, the nations not aligned with either the First World or the Second World.


328. ________ grasses are known as elephant grasses.
A. Prairie
B. Steppe
C. Savannah
D. Llanos

Answer

Answer: Option C
Explanation : Savannah grasses are known as elephant grasses. Elephant grass is a tall grass that originally came from Africa in 1913. It grows in dense clumps of up to 10 feet tall. In the savannas of Africa it grows along lake beds and rivers where the soil is rich. Local farmers cut the grass for their animals, carrying it home in huge piles on their backs or on carts.


329. The flat temperate grasslands of the Hungarian plains are known as ________.
A. Steppe
B. Prairies
C. Pampas
D. Pusztas

Answer

Answer: Option D
Explanation : The flat temperate grasslands of the Hungarian plains are known as Pusztas. Puszta is Hungarian name for steppe with official name Pannonian (Pannonic) steppe. Its not only in Hungary (but probably more than 90% of steppe is there, and in Hungary is known as Puszta) and Burgenland but also in Slovakia and Serbia.


330. What was Norway’s rank in 2016 Human Development Index published by the United Nations Development Programme?
A. 1
B. 10
C. 100
D. 200

Answer

Answer: Option A
Explanation : Norway’s rank in 2016 Human Development Index published by the United Nations Development Programme was 1. Human Development Index is published by United Nations development programme (UNDP). It was developed by combined effort of Amartya Sen and Mehboob-ul-Haq in 1990. India in 2016 was placed at 131 rank.


331. The South African River draining into Atlantic Ocean is
A. Orange
B. Zambezi
C. Limpopo
D. Nile

Answer

Answer: Option A
Explanation : The South African River draining into Atlantic Ocean is Orange. The Orange River is South Africa’s longest river. It originates from the Drakensberg in neighboring Lesotho where it is also known as the Senqu River and flows into Atlantic Ocean through South Africa.


332. Which among the following is a renewable source?
A. Forest
B. Coal
C. Limestone
D. Sand

Answer

Answer: Option A
Explanation : Forest is a renewable source. Biomass is considered one of the renewable sources of energy. Trees are also very important for us as a renewable resource. Trees are a natural resource that can be renewed – by the planting of trees – replacing the trees that are harvested for use by people.


333. Which of the following is the largest island?
A. Sumatra
B. Madagascar
C. Honshu
D. Cuba

Answer

Answer: Option B
Explanation : Madagascar is the largest island. Area: 587,713 sq km (226,917 sq miles), Population: 22,005,222, Population density: 33/km2 (85/sq miles), Location: Africa. Madagascar is the fourth largest island in the world, the largest island in the Indian Ocean, and an incredibly diverse island with huge swathes of virtually uninhabited land.


334. Which of the following countries has the greatest length of inland waterways?
A. The United States
B. The Former USSR
C. Germany
D. France

Answer

Answer: Option B
Explanation : The Former USSR countries has the greatest length of inland waterways.


335. Which of the following tribes and their area of inhabitance is incorrect ?
A. Samoyeds – Asiatic Tundra
B. Gaucho – Uruguay
C. Bindibus – Algeria
D. Tartars – Siberia

Answer

Answer: Option C
Explanation : Bindibus tribes are from Australia.


336. ‘Recycling’ is a method for conserving the resources of
A. Coal
B. Oil
C. Copper
D. Limestone

Answer

Answer: Option C
Explanation : ‘Recycling’ is a method for conserving the resources of Copper. To date only about 12% of known copper resources have been mined. However copper ore is a finite resource and it makes sense to conserve ore by recycling.


337. Horizontal transfer of heat is known as
A. Advection
B. Convection
C. Conduction
D. Elliptical line

Answer

Answer: Option A
Explanation : Horizontal transfer of heat is known as Advection. This type of heat transfer is not powered by variations in density, but rather requires an outside force, such as wind or currents, to displace the particles of the medium.


338. The country worst affected by cyclones and accompanying floods is ________.
A. Indonesia
B. Sri Lanka
C. Pakistan
D. Bangladesh

Answer

Answer: Option D
Explanation : Bangladesh is considered to be among the countries most vulnerable to a variety of natural disasters. The major natural hazards which Bangladesh has to confront regularly include floods, cyclones and accompanying storm surge. Damages due to large floods have increased considerably with time.


339. The Trans-Siberian railway links
A. The Baltic coast with the North Sea
B. Leningrad with Oldiotsk
C. Buanos Aires with Valparaiso
D. Leningrad with Vladivostok

Answer

Answer: Option D
Explanation : The Trans-Siberian railway links Leningrad with Vladivostok. The Trans-Siberian rail tour is one of the most interesting adventures anyone can take and Russian Trains.


340. When humidity is expressed in terms of percentage then it is known as
A. Absolute humidity
B. Specific humidity
C. Relative humidity
D. Ratio humidity

Answer

Answer: Option C
Explanation : When humidity is expressed in terms of percentage then it is known as Relative humidity. Relative humidity is expressed as a percentage. The relative humidity is 100% if the air is saturated with water vapor and 0% if no water vapor is present in the air at all.


341. Valparaiso is an important port of
A. Argentina
B. Uruguay
C. Chile
D. Paraguay

Answer

Answer: Option C
Explanation : Valparaiso is an important port of Chile. Starting in 1544 with Pedro de Valdivia´s proclamation as the “official port of the capital city of the Kingdom of Chile”, to its 1811 free trade declaration, to its 2003 designation as a UNESCO World Heritage Site.


342. The snowline is the line on mountain or hill slope which
A. Is the highest limit of perpetual snow .
B. Is the lowest limit of perpetual snow
C. Represents the highest amount of snowfall received
D. Represents the lowest amount of snowfall received

Answer

Answer: Option B
Explanation : The snowline is the line on mountain or hill slope which is the lowest limit of perpetual snow. The Imaginary line on a mountain or a hill which forms the lower limit of perpetual snow is called snow line. Below this line the snow melts away and above this line snow never melts. The snow line does not exist at the same level everywhere on earth. The average altitude of snow line is 5500m in the equatorial region, 4000-5000m in the Himalayan region, 2700m in the Alps and 0m in the polar regions.


343. The extension of the landmass into the sea is shown as the
A. Continental slope
B. Continental shelf
C. Continental mass
D. Continental territory

Answer

Answer: Option B
Explanation : The extension of the landmass into the sea is shown as the Continental shelf. Continental shelves were formed in between glacial periods as the ocean flowed over the continents forming shallow areas along the coasts. About 18,000 years ago, during the height of the Pleistocene ice ages, much of what is now a continental shelf was actually above water.


344. Peshawar is nearer to which of the following?
A. Karakoram Pass
B. Zojila Pass
C. Namika La Pass
D. Khybar Pass

Answer

Answer: Option D
Explanation : Peshawar is nearer to Khybar Pass. With the meagre distance of 50km, Khyber Pass stands closer to Peshawar as comparred to other passes.


345. In the transformation of vegetative matter to coal the first product is.
A. Peat
B. Lignite
C. Bituminous coal
D. Anthracite

Answer

Answer: Option A
Explanation : In the transformation of vegetative matter to coal the first product is Peat. As plants and trees died, their remains sank to the bottom of the swampy areas, making layers and layers of plant material and eventually forming a soggy, thick material called PEAT. The pressure from sedimentary rocks squeezed the water from the peat.


346. Consider the following statements :,1. Where fold mountains run parallel and close to the coast, continental shelf is narrow or absent,2. The average depth of continental shelf is 100 meters,3. Submarine canyons are mostly found in continental slope,4. The submarine flat topped mountains are called guyots.
A. 1 2 3 and 4
B. 1 2 and 4
C. 1 3 and 4
D. 2 and 3

Answer

Answer: Option B
Explanation : The following statements that are correct are: Where fold mountains run parallel and close to the coast, continental shelf is narrow or absent, Submarine canyons are mostly found in continental slope and The submarine flat topped mountains are called guyots.


347. The International Date Line is
A. 180 degrees longitude
B. The equator
C. 90 degrees east longitude
D. 0 degree longitude

Answer

Answer: Option A
Explanation : The International Date Line is 180 degrees longitude. The 180 degree line on the globe is the basis for the International Date Line. The 180 degrees line of longitude, which runs from west to east, on the globe indicates the meridian which divides the Earth to north and south parts.


348. The ports which function as the collection centres where the goods are brought from different countries to be sent to others besides their own country are called ________
A. Out ports
B. Packet stations
C. Entreport ports
D. Port of call

Answer

Answer: Option C
Explanation : Entrepôt refers to a port, city or warehouse where goods are imported to be stored or traded for re-export. The ports which function as the collection centres where the goods are brought from different countries to be sent to others besides their own country are called Entreport ports.


349. Which of the following natural regions is known as the “big game country” a land of “Safari”?
A. Equatorial
B. Tropical deserts
C. Savanna or Tropical Grassland region
D. Temperate Grassland region

Answer

Answer: Option C
Explanation : Savanna or Tropical Grassland region is known as the “big game country” a land of “Safari”.


350. The Gutenburg discontinuity separates ________.
A. Sial and Sima
B. Lithosphere and outer mantle
C. Outer mantle and inner mantle
D. Inner mantle and the core

Answer

Answer: Option D
Explanation : The Gutenburg discontinuity separates Inner mantle and the core. It separates and divides the lower mantle from the outer core. The lower mantle above the Gutenberg line is solid, but the outer core below the line is liquid molten.


351. Oases are examples of
A. Wet point settlements
B. Dry point settlements
C. Foothill settlements
D. None of these

Answer

Answer: Option C
Explanation : Oases are examples of Foothill settlements. Oasis, fertile tract of land that occurs in a desert wherever a perennial supply of fresh water is available. Oases also provide habitat for animals and spontaneous plants.


352. Which of the following are known as coastlines of submergence?
A. Ria coasts
B. Uplifted lowland
C. Emergent upland
D. Concordant coasts

Answer

Answer: Option A
Explanation : Ria coasts are known as coastlines of submergence. A ria coast is a coastline having several parallel rias separated by prominent ridges, extending a distance inland. The sea level change that caused the submergence of a river valley may be either eustatic (where global sea levels rise), or isostatic(where the local land sinks).


353. Ocean currents rotate
A. Clockwise in the northern hemisphere and anti-clockwise in the southern hemisphere
B. Anti-clockwise in the northern hemisphere and clockwise in the southern hemisphere
C. Clockwise in both the hemispheres
D. Anti-clockwise in both the hemispheres

Answer

Answer: Option A
Explanation : Ocean currents rotate Anti-clockwise in the northern hemisphere and clockwise in the southern hemisphere. The rotation of the Earth causes a clockwise shift in the direction of wind and ocean current flow in the Northern Hemisphere and a counterclockwise shift in the Southern Hemisphere known as the “Coriolis effect”.


354. The ________ route is the busiest ocean route of the world.
A. Suez
B. Panama
C. North Atlantic
D. South Atlantic

Answer

Answer: Option C
Explanation : The North Atlantic route is the busiest ocean route of the world because it includes the Suez Canal, which connects Europe to the Mediterranean Sea.


355. Spot the odd item in the following
A. Mt Fuji
B. Mt Everest
C. Mt Vesuvius
D. Mt Pele

Answer

Answer: Option B
Explanation : Mount Everest is a mountain. There have never been any volcanic eruptions in the Himalayas, and it is highly unlikely there will be in the near future. There is no active lava source within the mountain range.


356. Which European country has over 200 volcanoes, many of them active?
A. Norway
B. Finland
C. Ireland
D. Iceland

Answer

Answer: Option D
Explanation : Iceland has over 200 volcanoes, many of them active. There are approximately 130 volcanoes in Iceland, active and inactive. About 30 active volcanic systems can be found under the island, in all parts of the country other than the Westfjords.


357. The route is connecting Western Europe, Africa and Australia by way of the Cape of Good Hope.
A. Panama route
B. Cape route
C. North Pacific route
D. South Atlantic route

Answer

Answer: Option B
Explanation : Cape route is connecting Western Europe, Africa and Australia by way of the Cape of Good Hope. The European-Asian sea route, also known as the sea route to India or the Cape Route is a shipping route from European coast of the Atlantic Ocean to Asia’s coast of the Indian Ocean passing by the Cape of Good Hope and Cape Agulhas at the southern edge of Africa.


358. Leningrad region and Kola Peninsula in former Soviet Union are major producers of
A. Lead
B. Zinc
C. Bauxite
D. Copper

Answer

Answer: Option C
Explanation : Leningrad region and Kola Peninsula in former Soviet Union are major producers of Bauxite.


359. Heligoland is an archipelago of which of the following nations?
A. Britain
B. Germany
C. USA
D. Indonesia

Answer

Answer: Option B
Explanation : Heligoland is an archipelago of Germany. At present is administered by Germany in North Sea. Earlier these were occupied by Denmark & later by Britain.


360. Which one of the following countries makes maximum use of the geothermal energy?
A. New Zealand
B. Japan
C. Iceland
D. Russia

Answer

Answer: Option C
Explanation : Iceland countries makes maximum use of the geothermal energy. Iceland is a pioneer in the use of geothermal energy for space heating. Generating electricity with geothermal energy has increased significantly in recent years. Geothermal power facilities currently generate 25% of the country’s total electricity production.


361. Israel has common borders with ________
A. Lebanon Syria Jordan and Egypt
B. Lebanon Syria Turkey and Jordan
C. Cyprus Turkey Jordan and Egypt
D. Turkey Syria Iraq and Yemen

Answer

Answer: Option B
Explanation : Israel is located in the Middle East along the eastern end of the Mediterranean Sea. It is bound by the Mediterranean Sea to the west, Lebanon to the north, Syria to the northeast, Jordan to the east and Egypt to the southwest. The country is divided into three topographical regions.


362. Which, among the following countries, occupies the largest chunk of the Sunderban’s Delta?
A. Bangladesh
B. Myanmar
C. India
D. Thailand

Answer

Answer: Option A
Explanation : Bangladesh, occupies the largest chunk of the Sunderban’s Delta. Sundarbans, The largest single block of tidal halophytic mangrove forest in the world, located in the southwestern part of Bangladesh.


363. The country with the highest density of population in Europe is ________
A. France
B. Italy
C. Belgium
D. Netherlands

Answer

Answer: Option D
Explanation : The country with the highest density of population in Europe is Netherlands. With over 16.5 million people and a population density of 488 people per km2, the Netherlands is the most densely populated country of the European Union and one of the mostly densely populated countries in the world.


364. Three European cities viz, Belgrade, Budapest and Vienna, are located on the banks of the river
A. Danube
B. Rhine
C. Rhone
D. Volga

Answer

Answer: Option A
Explanation : Three European cities viz, Belgrade, Budapest and Vienna, are located on the banks of the river Danube.


365. The largest railway station of the world is ________
A. Trans-Siberian railway station Russia
B. Grand Central Terminal Chicago
C. Kharagpur railway station India
D. None of these

Answer

Answer: Option B
Explanation : The largest railway station of the world is Grand Central Terminal, Chicago. The world’s largest station by number of platforms is Grand Central Terminal, Park Avenue and 42nd Street, New York City, USA, built from 1903–13 which has 44 platforms. They are situated on two underground levels with 41 tracks on the upper level and 26 on the lower.


366. Cold currents have cooling effect on the shore of ________
A. Peru
B. Japan
C. Western Europe
D. Alaska

Answer

Answer: Option A
Explanation : Cold currents have cooling effect on the shore of Peru. The Peruvian Current affects Peru year round, and moderates the climate of Chile in spring and summer, when it displaces a subtropical center of high pressure. The air that accompanies the current is dry, keeping the coast arid.


367. Which one of the following pairs is not correctly matched? States of USA Geography Division
A. Iowa West North Central
B. Texas West South Central
C. California Pacific
D. New Jersey South Atlantic

Answer

Answer: Option D
Explanation : New Jersey is a northeastern U.S. state with some 130 miles of Atlantic coast.


368. Fog can be classified as a
A. Nimbus cloud
B. Cirrus cloud
C. Cumulus cloud
D. Low Stratus cloud

Answer

Answer: Option D
Explanation : Fog can be classified as a Low Stratus cloud. Stratus is used to describe flat, featureless clouds of low altitude, typically less than 300 metres. On the other hand, fog is a stratus cloud in contact with the ground. Horizontal visibility in fog is less than 1 km.


369. Crude oil is generally found in
A. Igneous rocks
B. Sedimentary rocks
C. Metamorphic rocks
D. Sandy desert soils

Answer

Answer: Option B
Explanation : Crude oil is generally found in Sedimentary rocks. Crude oil is found in the pore spaces within non-solid rocks such as compacted sediments.


370. Which of the following absorbs the insolation from the sun and preserves the earth’s radiated heat, thus acting like a blanket allowing the earth neither to become too cold nor too hot?
A. Dust
B. Water vapour
C. Ozone
D. Nitrogen

Answer

Answer: Option B
Explanation : Water vapour absorbs parts of the insolation from sun and thus reduces its amount. It also preserves the earth’s radiated heat. It thus acts like a blanket allowing the earth neither to become too cold nor too hot.


371. ________ is famous for its rubber production.
A. Nigeria
B. Ghana
C. Malaysia
D. Germany

Answer

Answer: Option C
Explanation : Malaysia is famous for its rubber production. About 46% of the total world’s rubber is produced in Malaysia. They are the leader in the production and export of rubber products.


372. Which of the following layers of the Earth is believed to account for the Earth’s magnetism?
A. Crust
B. Mantle
C. Core
D. Lithosphere

Answer

Answer: Option C
Explanation : Core of the Earth is believed to account for the Earth’s magnetism. The Earth’s magnetic field is believed to be generated by electric currents in the conductive iron alloys of its core, created by convection currents due to heat escaping from the core.


373. Bara (kral) is used for which of the following
A. House of Maasai shepherd
B. Village of Maasai shepherd
C. Stockyard of kikuyu
D. Tent of Kirgiz

Answer

Answer: Option B
Explanation : Maasai tribe of African origin is found in the border region of Kenya and Tanzania. It lives in patchy settlement termed as Bara (Village).


374. The temperature at the core of the earth is about
A. 20000º C
B. 5200º C
C. 200000º C
D. 26000º C

Answer

Answer: Option B
Explanation : The temperature at the core of the earth is about 5200º C. Earth’s core is the very hot, very dense center of our planet. Temperature in the inner core is about 5,200° Celsius (9,392° Fahrenheit) .


375. Which one among the following is not correct regarding Tsunami?
A. Tsunamis have a very long wavelength
B. Tsunamis have a slight swell of about twelve inches about the normal sea surface
C. Tsunamis grow in height when they reach shallower water
D. Tide also play an important role in the generation of Tsunamis

Answer

Answer: Option D
Explanation : The tsunami waves are completely unrelated to the astronomical tides – which are caused by the extraterrestrial, gravitational influences of the moon, sun, and the planets. Thus, the Japanese word “tsunami”, meaning “harbor wave” is the correct, official and all-inclusive term. It has been internationally adopted because it covers all forms of impulsive wave generation.


376. Suez Canal has reduced the distance between London and Mumbai by about
A. 1000 km
B. 5000 km
C. 6000 km
D. 7000 km

Answer

Answer: Option D
Explanation : India is connected with Europe, North America and South America through both the routes-the Cape of Good Hope and the Suez Canal. Since the opening of the Suez Canal in 1869, distance between India and Europe has been reduced by 7000 kms.


377. The division of the world into major natural regions is primarily based on
A. Atmosphere
B. Rainfall
C. Climate types
D. Annual range of temperature

Answer

Answer: Option C
Explanation : The division of the world into major natural regions is primarily based on Climate types. The natural regions of the world are divided on the basis of climate. The climate includes both the temperature and rainfall in a given region.


378. The cold Labrador current brings nine months winter to ________
A. USA
B. Eastern Newfoundland
C. Western Norway
D. Northern Sweden

Answer

Answer: Option B
Explanation : The cold Labrador Current brings nine months winter to Eastern Newfoundland. The Labrador Current is a cold current in the North Atlantic Ocean. It flows from the Arctic Ocean along the coast of Labrador. It passes around Newfoundland and continues south along the east coast of Nova Scotia.


379. Contours are lines showing
A. Places on earth of equal rainfall
B. Places of same temperature
C. Places on earth at the same altitude
D. None of these

Answer

Answer: Option C
Explanation : Contours are lines showing Places on earth at the same altitude. Contour lines are isolines joining places that have the same height value.


380. A country through which both Equator and Tropic of Capricorn pass is
A. Brazil
B. Argentina
C. Congo
D. Indonesia

Answer

Answer: Option A
Explanation : A country through which both Equator and Tropic of Capricorn pass is Brazil. The Tropic of Capricorn lies at 23.5 degrees south of the equator and runs through Australia, Chile, southern Brazil (Brazil is the only country that passes through both the equator and a tropic), and northern South Africa.


381. Where is the Port City of Cork located?
A. Italy
B. France
C. Greece
D. Ireland

Answer

Answer: Option D
Explanation : The Port City of Cork is located in Ireland. The Port of Cork (Irish: Port Chorcaí) is the main port serving the South of Ireland, County Cork and Cork City.


382. People of North Africa are generally
A. Negroid
B. Berbers
C. Mongoloid
D. None of these

Answer

Answer: Option B
Explanation : People of north Africa are generally Berbers. The inhabitants of Northern Africa are generally divided in a manner roughly corresponding to the principal geographic regions: The Maghreb, the Nile Valley, and the Sahara. Northwest Africa on the whole is believed to have been inhabited by Berbers since the beginning of recorded history, while the eastern part of Northern Africa has been home to the Egyptians, Abyssinians (Ethiopians), and Nubians (Sudanic descent), although ancient Egyptians record extensive contact in their Western desert with peoples that appear to have been Berber or proto-Berber.


383. The Strait which separates Asia from North America is ________
A. The Bering Strait
B. The Palk Strait
C. The Strait of Gibraltar
D. The Strait of Malacca

Answer

Answer: Option A
Explanation : The northwestern tip of North America is separated from northeast Asia by the Bering Strait. This stretch of water is just 55 miles (88 km) wide, leading from the Bering Sea to the Arctic Ocean.


384. The shortest air route from Moscow to San Francisco is
A. Over the Atlantic Ocean
B. Over the Pacific via Siberia
C. Over the North Pole
D. Through the latitude which passes through them

Answer

Answer: Option A
Explanation : The shortest air route from Moscow to San Francisco is over the Atlantic Ocean.


385. Which of the following does not characterless ‘selva’?
A. Evergreen forest
B. They have suffered the greatest destruction at the hands of man
C. Climbers hang from the roof formed by the foliage
D. Commercial utilisation of these forests is difficult

Answer

Answer: Option B
Explanation : They have suffered the greatest destruction at the hands of man.


386. The smallest country in South America is ________
A. Uruguay
B. Guyana
C. Surinam
D. Ecuador

Answer

Answer: Option C
Explanation : The smallest country in South America is Surinam. Suriname, country located on the northern coast of South America. Suriname is one of the smallest countries in South America, yet its population is one of the most ethnically diverse in the region.


387. The tip of South America is ________
A. Cape Verde
B. Cape York
C. Cape Horn
D. Cape Cod

Answer

Answer: Option C
Explanation : The tip of South America is Cape Horn. Cape Horn, the southernmost tip of South America, remain a maritime legend to this day, as sailing around this remote point and then through the Drake Passage was (and is) one of the most challenging nautical routes on the planet.


388. ________ is called the ‘low country’ of the earth.
A. Spain
B. Holland (Netherlands)
C. Italy
D. Hungary

Answer

Answer: Option B
Explanation : Holland (Netherlands) is called the ‘low country’ of the earth because much of their land surface is at or below sea level.


389. Which of the following is a hardwood tree?
A. Shisham
B. Pine
C. Spruce
D. Deodar

Answer

Answer: Option A
Explanation : Shisham is a hardwood tree. Sheesham wood also known as Indian Rosewood or Dalbergia sisso, is a deciduous tree. After teak, it’s most significant cultivated timber tree in Asian country.


390. The smallest amongst the following oceans is ________.
A. The Arctic Ocean
B. The Indian Ocean
C. The Atlantic Ocean
D. The Pacific Ocean

Answer

Answer: Option A
Explanation : The Arctic Ocean is the smallest of the world’s five ocean basins.


391. Palk Strait separates
A. Malaysia and Sumatra
B. India and Sri Lanka
C. Sunda and Sumatra
D. Borneo and Celebes

Answer

Answer: Option B
Explanation : Palk Strait separates India and Sri Lanka. Palk Strait, inlet of the Bay of Bengal between southeastern India and northern Sri Lanka. It is bounded on the south by Pamban Island (India), Adam’s (Rama’s) Bridge (a chain of shoals), the Gulf of Mannar, and Mannar Island (Sri Lanka).


392. Name the mountain that separates France and Spain.
A. Alps
B. Pyrenees
C. Jura
D. Vosges

Answer

Answer: Option B
Explanation : Pyrenees separates France and Spain. It is located right on the border between the two countries.


393. Which of the following is not helpful for delta formation?
A. Fast movement of river in mountains to cut sediments
B. Slow speed
C. Long river course
D. Tidal nature of waves

Answer

Answer: Option D
Explanation : Tidal nature of waves is not helpful for delta formation. A tidal delta is the sand bar or shoaling area left at the mouth of a river by the movement of bottom mud and sand by a diurnal tide and the currents that result from that tide.


394. In which of the following units is atmospheric pressure measured?
A. Grams per square km
B. Grams per centimetre
C. Kilograms per square metre
D. Pounds per cubic mile

Answer

Answer: Option C
Explanation : Atmospheric pressure is measured in Kilograms per square metre.


395. The continent through which the Equator, Tropic of Cancer and the Tropic of Capricorn pass, is
A. Asia
B. South America
C. Africa
D. Australia

Answer

Answer: Option C
Explanation : The Tropic of Capricorn lies at 23.5 degrees south of the equator and runs through Australia, Chile, southern Brazil (Brazil is the only country that passes through both the equator and a tropic), and northern South Africa.


396. Ozone layer is present in
A. Stratosphere
B. Troposphere
C. Ionosphere
D. Exosphere

Answer

Answer: Option A
Explanation : Ozone layer is present in Stratosphere. Ozone is mainly found in two regions of the Earth’s atmosphere. Most ozone (about 90%) resides in a layer that begins between 6 and 10 miles (10 and 17 kilometers) above the Earth’s surface and extends up to about 30 miles (50 kilometers). This region of the atmosphere is called the stratosphere.


397. Ozone layers absorb
A. Infrared rays
B. Ultraviolet rays coming from the sun
C. All radiations from the sun
D. Cosmic rays

Answer

Answer: Option B
Explanation : Ozone layers absorb Ultraviolet rays coming from the sun. Although the concentration of the ozone in the ozone layer is very small, it is vitally important to life because it absorbs biologically harmful ultraviolet (UV) radiation coming from the sun.


398. Pakistan is bordered by
A. Afghanistan Iraq India and China
B. Afghanistan Iran and India
C. Afghanistan CIS India and China
D. Afghanistan Turkey India and China

Answer

Answer: Option B
Explanation : Pakistan is bordered by Afghanistan, Iran and India.


399. Arrange the following countries of southAsia in their descending order of literacy of people-, 1. Bangladesh, ,2. India, ,3. Pakistan, ,4. Srilanka
A. 1432
B. 3241
C. 4231
D. 4321

Answer

Answer: Option C
Explanation : Descending order of countries in literacy rate:- Sri Lanka > India > Pakistan > Bangladesh (in 2006). At present, Sri Lanka (92%)> India (72%)> Bangladesh (61%)> Pakistan(56%).


400. World’s principal producer of silver is
A. India
B. Mexico
C. The USA
D. Canada

Answer

Answer: Option B
Explanation : World’s principal producer of silver is Mexico. Fresnillo Company from Mexico is the world’s leading producers of silver.


401. Po river flows through
A. Austria
B. Italy
C. Yugoslavia
D. France

Answer

Answer: Option B
Explanation : Po river flows through Italy. Po River, Latin Padus, longest river in Italy, rising in the Monte Viso group of the Cottian Alps on Italy’s western frontier and emptying into the Adriatic Sea in the east after a course of 405 miles (652 km).


402. Prognathiam rerers to
A. Slanting forehead
B. Protruding jaw
C. Slit eye
D. None of these

Answer

Answer: Option B
Explanation : Prognathiam refers to Protruding jaw. Prognathism refers to a protruding jaw. It is also called an extended chin or Habsburg jaw. The condition is usually a sign of an underlying condition. Talk to your doctor if you suspect your jaw is protruding.


403. Cut off loops of rivers are known as
A. Bajada
B. Meanders
C. Oxbow lake
D. Levee

Answer

Answer: Option C
Explanation : Cut off loops of rivers are known as Oxbow lake. Cut off, in a river, shortcut across a meander (q.v.). loop that shortens and straightens the course of the stream.


404. Copper alloys with tin to give
A. Brass
B. Bronze
C. German silver
D. Zinc

Answer

Answer: Option B
Explanation : Copper alloys with tin to give Bronze. Bronze is an alloy that consists primarily of copper with the addition of other ingredients. In most cases the ingredient added is typically tin, but arsenic, phosphorus, aluminum, manganese, and silicon can also be used to produce different properties in the material.


405. Which among the following is the most populous city in the world?
A. Kolkata
B. Mexico City
C. Sao Paulo
D. Tokyo

Answer

Answer: Option D
Explanation : Tokyo is the most populous city in the world. Tokyo is still the world’s largest city with 37 million inhabitants, but it’s growth has plateaued and is projected to begin declining around 2020.


406. Oceans occupy per cent of the earth.
A. 21
B. 71
C. 29
D. 79

Answer

Answer: Option B
Explanation : The oceans cover 71 percent of the Earth’s surface and contain 97 percent of the Earth’s water.


407. Which one the following condition is not favourable for the formation of delta?
A. Active vertical and lateral erosion in the lower course of the rivers
B. Coast should be preferably tideless
C. Sea adjoining the delta should be shallow
D. There should be no strong current running at right angles to the river mouths

Answer

Answer: Option A
Explanation : Active vertical and lateral erosion in the lower course of the rivers is not favourable for the formation of delta. Favourable conditions are Active vertical & lateral erosion in upper course of the river to provide extensive sediments. Coast should be sheltered preferably tideless & no strong current at right angle to the mouth of the river washing away the sediments. Sea adjoining the delta should be shallow or else the load will disappear in deep waters.


408. In the Himalayas the melting or the snow is more on the
A. Southern slopes
B. Northern slopes
C. Eastern slopes
D. Western slopes

Answer

Answer: Option A
Explanation : In the Himalayas the melting or the snow is more on the Southern slopes. In winter, the sun is always towards the South and the northern facing parts of any mountain are in perpetual shadow. However, in summer, the sun will not migrate as much to the North as it migrated to the South in winter.


409. Which of the following gives the correct decreasing order in numbers in the food chain pyramid?,1. Primary Producers,2. Herbivores,3. Carnivores,4. Man
A. 1 2 3 4
B. 1 3 2 4
C. 4 3 2 1
D. 1 2 4 3

Answer

Answer: Option A
Explanation : The correct decreasing order in numbers in the food chain pyramid are Primary Producers, Herbivores, Carnivores and Man.


410. The trees of spruce, fir, pine, etc are found in which of the following natural regions of the Earth?
A. Siberian Type
B. Tundra Type
C. Taiga Type
D. Laurentian Type

Answer

Answer: Option C
Explanation : The trees of spruce, fir, pine, etc are found in Taiga Type natural regions of the Earth. Taigas are thick forests. Coniferous trees, such as spruce, pine, and fir, are common. Coniferous trees have needles instead of broad leaves, and their seeds grow inside protective, woody cones.


411. Danube, Dneiper, Don, Volga and Ural rivers are flowing in
A. North America
B. Europe
C. Africa
D. Australia

Answer

Answer: Option B
Explanation : Danube, Dneiper, Don, Volga and Ural rivers are flowing in Europe.


412. The sea that separates Greece and Italy is
A. Mediterranean
B. The Tyrrhenian
C. The Adriatic
D. The Aegean

Answer

Answer: Option C
Explanation : The sea that separates Greece and Italy is The Adriatic. The Adriatic Sea is part of the Mediterranean Sea. It separates the Italian Peninsula from the Balkan peninsula and the Apennine Mountains from the Dinaric Alps and adjacent ranges.


413. The layers of the earth’s crust are distinguished by their
A. Thickness and depth
B. Density and temperature
C. Metallic content and rocks
D. All the above

Answer

Answer: Option D
Explanation : The layers of the earth’s crust are distinguished by their Thickness and depth, Density and temperature and Metallic content and rocks.


414. The coral reefs are most characteristic of
A. Indian Ocean
B. Pacific Ocean
C. Atlantic Ocean
D. Mediterranean Sea

Answer

Answer: Option B
Explanation : The coral reefs are most characteristic of Pacific Ocean. Coral reefs are the most diverse of all marine ecosystems. They teem with life, with perhaps one-quarter of all ocean species depending on reefs for food and shelter.


415. The regions with the highest and lowest densities are respectively ________
A. Europe and Africa
B. Asia and Oceania
C. North America and Africa
D. Europe and Oceania

Answer

Answer: Option D
Explanation : The regions with the highest and lowest densities are respectively Europe and Oceania.


416. Rice crop needs
A. Low temperature and heavy rainfall
B. High temperature and low rainfall
C. Annual rainfall of about 150 cm high temperature and humidity
D. Low temperature and low rainfall

Answer

Answer: Option C
Explanation : Rice crop needs annual rainfall of about 150 cm, high temperature and humidity. Rice requires high temperature above 200C-350C and high rainfall between 150 to 300 cm. Well drained Alluvial and loamy soil is suitable for the growth of wheat.


417. Pale colour, medium height, epicanthic eye are the characteristics of
A. Australoid
B. Negroid
C. Mongoloid
D. Cancosoid

Answer

Answer: Option C
Explanation : Mongoloids are Indigenous people of East, Central South East & North Asia. They are also found in Arctic, Americas and Pacific. They are of medium height & pale colour.


418. “Golan Heights” belonging to a country A were captured by a country B. The countries A and B are ________
A. Tibet and China
B. Russia and Japan
C. Syria and Israel
D. Iran and Iraq

Answer

Answer: Option C
Explanation : “Golan Heights” belonging to a country A were captured by a country B. The countries A and B are Syria and Israel.


419. Pyranees form a the boundary between
A. Spain and Portugal
B. France and Spain
C. Poland and Russia
D. Hungary and Rumania

Answer

Answer: Option B
Explanation : The Pyrenees Mountains form the natural border between France and Spain, and it completely engulfs the nation of Andorra. The mountains extend for about 270 miles from the Bay of Biscay to the Mediterranean Sea. The highest point is Pico de Aneto at 11,168 ft. (3,404m).


420. Consider the following countries:,1. Algeria,2. Morocco,3. Mauritania,Which of these borders is/are the Mediterranean Sea?
A. 1 only
B. 1 and 2
C. 2 and 3
D. 1 2 and 3

Answer

Answer: Option B
Explanation : Algeria and Morocco Mauritania borders the Mediterranean Sea.


421. The difference between the equatorial and polar diametres of the earth is about
A. 22 km
B. 32 km
C. 42 km
D. 62 km

Answer

Answer: Option C
Explanation : The difference between the equatorial and polar diametres of the earth is about 42 km. Due to spinning motion the Earth’s equator area bulges out. There is difference of about 42 Kilometers between diameter at equator and at poles.


422. ‘Roaring Forties’ are
A. Waterfalls in Canada
B. Cyclonic winds
C. The turbulent years in world History from 1940 to 1944
D. Westerly winds blowing through out the year over the oceans of the Southern Hemisphere between 40 º and 60 º South

Answer

Answer: Option D
Explanation : ‘Roaring Forties’ are Westerly winds blowing through out the year over the oceans of the Southern Hemisphere between 40 º and 60 º South. The Roaring Forties refers to the belt of ripping westerly winds, aided by the Earth’s rotation, between roughly 40 and 60 degrees latitude in the southern hemisphere.


423. Which of the following is not concerned with increased production of a food item?
A. Green Revolution
B. Blue Revolution
C. White Revolution
D. Orange Revolution

Answer

Answer: Option D
Explanation : Orange Revolution is not concerned with increased production of a food item.


424. A narrow strip of land connecting two large land masses is called
A. Ridge
B. Peninsula
C. Isthmus
D. Neck

Answer

Answer: Option C
Explanation : An isthmus is a land bridge, a narrow strip of land that stretches across a body of water to connect two larger land masses. The Greek isthmos means “neck,” so you can see how isthmus came to mean the connecting strip between land masses.


425. If I wanted to reach the South Atlantic from the South Pacific while touching South America, I would have to use
A. Florida Straits
B. Dardanelles
C. Magellan Straits
D. Bosporus

Answer

Answer: Option C
Explanation : If I wanted to reach the South Atlantic from the South Pacific while touching South America, I would have to use Magellan Straits.


426. The inequality between day and night becomes greater or more marked as one travels from ________ to ________.
A. Poles Equator
B. Equator Poles
C. Tropic of Cancer Tropic of Capricorn
D. Tropic of Capricorn Tropic of Cancer

Answer

Answer: Option B
Explanation : The inequality between day and night becomes greater or more marked as one travels from Equator to Poles.


427. Which of the following countries have achieved a negative growth rate of population?
A. Australia Canada and Sweden
B. U.K. Sweden and Germany
C. France Italy and Ireland
D. Japan U.S.
E. and Russia

Answer

Answer: Option A
Explanation : Australia, Canada and Sweden have achieved a negative growth rate of population. This negative or zero natural population growth means that these countries have more deaths than births or an even number of deaths and births; this figure does not include the effects of immigration or emigration.


428. Which one of the following statements correctly defines the term ‘Karroo’ of Southern Africa?
A. Rift valley formed due to faulting
B. Dome shaped landforms caused by earth’s movements
C. Steppes from the coastal lowland on to the high plateau
D. None of the above

Answer

Answer: Option C
Explanation : Steppes from the coastal lowland on to the high plateau defines the term ‘Karroo’ of Southern Africa. The Karoo is a semi desert natural region of South Africa.


429. Infant mortality rate is ________
A. Number of deaths of children below 1 year of age per 1000 of the population
B. Number of deaths of children below 5 years of age per 1000 of the population
C. Number of deaths of children below 1 year of age per 100 of the population
D. Number of deaths of children below 5 years of age per 100 of the population

Answer

Answer: Option A
Explanation : Infant mortality rate is number of deaths of children below 1 year of age per 1000 of the population. Infant mortality rate, measure of human infant deaths in a group younger than one year of age. It is an important indicator of the overall physical health of a community.


430. Strait of Florida runs in between ________
A. Atlantic Ocean and Caribbean Sea
B. Gulf of Mexico and Caribbean Sea
C. Atlantic Ocean and Gulf of Mexico
D. Caribbean Sea and Pacific Ocean

Answer

Answer: Option C
Explanation : Strait of Florida runs in between Atlantic Ocean and Gulf of Mexico. It is a strait located south-southeast of the North American mainland, generally accepted to be between the Gulf of Mexico and the Atlantic Ocean, and between the Florida Keys (U.S.) and Cuba.


431. Llanos and Campos are grasslands found in
A. Siberia
B. North America
C. South America
D. India

Answer

Answer: Option C
Explanation : llanos and campos are grasslands found in South America. Ilanos is a vast tropical grassland plain situated to the east of the Andes in Colombia and Venezuela, in northwestern South America. It is an ecoregion of the flooded grasslands and savannas biome.The Campos, grassland with few trees or shrubs except near streams, lies between 24°S and 35°S; it includes parts of Brazil, Paraguay and Argentina, and all of Uruguay.


432. St Andreas Fault passes close to the US city of
A. Los Angeles
B. New Orleans
C. Hollywood
D. San Francisco

Answer

Answer: Option D
Explanation : St Andreas Fault passes close to the US city of San Francisco. The San Andreas Fault is a continental transform fault that extends roughly 1,200 kilometers (750 mi) through California. It forms the tectonic boundary between the Pacific Plate and the North American Plate, and its motion is right-lateral strike-slip (horizontal).


433. Ear rot is a disease which Infects
A. Wheat
B. Sugarcane
C. Corn
D. Coffee

Answer

Answer: Option C
Explanation : Ear rot is a disease which Infects Corn. Variable weather conditions are causing ear rots to appear in corn.


434. Main hunting ground for blue whales is located in ________
A. Antarctic region
B. Arctic Ocean
C. Indian Ocean
D. Southern Atlantic Ocean

Answer

Answer: Option A
Explanation : Main hunting ground for blue whales is located in Antarctic Ocean. It is estimated that more than 330,000 blue whales had been caught in the Antarctic alone, and the original population had been reduced to just 360 individuals.


435. The correct decreasing order of the population of the various continents is ________
A. Asia Europe America Africa Australia
B. Asia America Europe Africa Australia
C. Asia Africa Europe America Australia
D. Asia America Africa Europe Australia

Answer

Answer: Option C
Explanation : The correct decreasing order of the population of the various continents is
Sr. No.Continent 1Asia 2Africa 3Europe 4North America 5South America 6Oceania 7Antarctica


436. Addis Ababa is the capital of
A. Ethiopia
B. Kenya
C. Zambia
D. Nigeria

Answer

Answer: Option A
Explanation : Addis Ababa is the capital of Ethiopia. Addis Ababa, also spelled Addis Abeba, capital and largest city of Ethiopia. It is located on a well-watered plateau surrounded by hills and mountains, in the geographic centre of the country. Only since the late 19th century has Addis Ababa been the capital of the Ethiopian state.


437. The deepest or the Oceanic trenches is the ________ trench.
A. Mariana
B. Puerto Rico
C. Japanese
D. Sunda

Answer

Answer: Option A
Explanation : The deepest of the Oceanic trenches is the Mariana trench. The Challenger Deep in the Mariana Trench is the deepest known point in Earth’s oceans.


438. ________ canal is gateway to the Pacific.
A. Suez
B. Panama
C. Kiel
D. Grand Union

Answer

Answer: Option B
Explanation : Panama Canal is called the Gateway to the Pacific. The Panama Canal combines the Pacific Ocean and (by the Caribbean Sea) the Atlantic Ocean.


439. Some riven and canals have locks across them in order to
A. Collect taxes
B. Ensure safety
C. Control the level of water
D. Load or unload freight

Answer

Answer: Option C
Explanation : Some rivers and canals have locks across them in order to Control the level of water. A lock is a device used for raising and lowering boats, ships and other watercraft between stretches of water of different levels on river and canal waterways.


440. What is meant by ‘brain drain’?
A. Increase in the population of mentally retarded or brainless people
B. The increasing medical facilities which have resulted in decrease in the number of mentally retarded people
C. The marriage of highly intellectual people so as to add more intelligent progency to the population
D. The migration of highly qualified or skilled people to better places of employment

Answer

Answer: Option D
Explanation : Brain Drain is the departure of educated or professional people from one country, economic sector, or field for another usually for better pay or living conditions.


441. Lines Joining places having equal height are
A. Contours
B. Isobars
C. Isohalines
D. Isohyet

Answer

Answer: Option A
Explanation : A line joining the places with equal heights is called counter lines. The countor lines join places with equal heights. Countor lines are also called as isolines.


442. The dwarf camels, known as Alpacas, Vicunas and Llamas are found in
A. Tibet
B. Afghanistan
C. Myanmar
D. Cyprus

Answer

Answer: Option A
Explanation : The dwarf camels, known as Alpacas, Vicunas and Llamas are found in Tibet. The llama, its wild cousin , the Vicuna and hump-less Alpaca are also found in the high pasture lands of the Andes region of Bolivia and peru, where they are useful as beasts of burden and for the production of fleece and meat.


443. Which part of Europe is most densely populated?
A. Eastern Europe
B. Western Europe
C. Northern Europe
D. Area around the Black Sea

Answer

Answer: Option B
Explanation : Western Europe is most densely populated because they have flatter land, good soils and a mild climate.


444. The world’s most humid continent is
A. Asia
B. Europe
C. North America
D. South America

Answer

Answer: Option D
Explanation : As most of the South America lies in humid temperate zone & also the fact that equator passes through South America, the climate is mostly humid.


445. The area or the Indian Ocean is approximately ________.
A. 24 million sq km
B. 44 million sq km
C. 64 million sq km
D. 74 million sq km

Answer

Answer: Option D
Explanation : The area or the Indian Ocean is approximately 74 million sq km. Indian Ocean, body of salt water covering approximately one-fifth of the total ocean area of the world. It is the smallest, geologically youngest, and physically most complex of the world’s three major oceans. It stretches for more than 6,200 miles (10,000 km) between the southern tips of Africa and Australia and, without its marginal seas, has an area of about 28,360,000 square miles (73,440,000 square km).


446. Which of the following regions of the world is most thickly populated?
A. East Asia
B. South Asia
C. North-West Europe
D. North and South America

Answer

Answer: Option A
Explanation : East Asia is most thickly populated. Asia is the most densely populated continent in the world. Eastern Asia population is equivalent to 21.51% of the total world population.


447. Which one of these places lies nearest to the Equator?
A. Singapore
B. Lagos
C. Colombo
D. Jakarta

Answer

Answer: Option A
Explanation : Singapore lies nearest to the Equator. Singapore is located 6,125.89 mi (9,858.67 km) south of the North Pole.


448. Which of the following is not a tropical cyclone?
A. Typhoon
B. Tornado
C. Hurricane
D. Willy-willy

Answer

Answer: Option B
Explanation : Tornado is not a tropical cyclone. It is important not to confuse a tornado with a hurricane or tropical cyclone because tornadoes and hurricanes are very different phenomena. The only similarity between them is that they both contain strong rotating winds that can cause damage.


449. At which particular place on earth are day and nights of equal length always?
A. Prime Meridian
B. Poles
C. Equator
D. Nowhere

Answer

Answer: Option C
Explanation : At Equator days and night are of equal length.


450. Which one of the following is not an African country?
A. Cameroon
B. Gabon
C. Mauritania
D. Yemen

Answer

Answer: Option D
Explanation : No, Yemen is not in Africa. It is a country that is located on the Arabian peninsula in Asia, across the Red Sea and the Gulf of Aden.


451. In the context of exports, which one of the following pairs is NOT correctly matched ?
A. Cape Town : Wool and wine
B. Adelaide : Wheat and wool
C. Perth : Rice and corn
D. San Francisco : Fruits and wine

Answer

Answer: Option C
Explanation : Perth is famous for fruits and vegetables.


452. What is the shape of the Earth?
A. Geoid
B. Circle
C. Sphere
D. Hemi-sphere

Answer

Answer: Option A
Explanation : Geoid is the shape of the Earth. There are many reasons why the geoid isn’t a sphere. First and foremost, the Earth itself isn’t a sphere. It is closer to an ellipsoid, being flattened at the poles by the centrifugal force of its own rotation. But the planet isn’t an ellipsoid either, because of topography.


453. Strait of Gibraltar connects which of the following?
A. Red Sea – Mediterranean Sea
B. Red Sea – Arabian Sea
C. Atlantic Ocean – Mediterranean Sea
D. Mediterranean Sea – Black Sea

Answer

Answer: Option C
Explanation : Situated on Iberian Peninsula (Spain) Strait of Gibraltar separates Europe from Africa & connects two large water bodies, Atlantic Ocean & Mediterranean Sea.


454. The Trans-Andean railway connects
A. Mexico City with Santiago
B. Panama City with Santiago
C. Buenos Aires with Valparaiso
D. Rio de Janerio with Bogota

Answer

Answer: Option C
Explanation : The Trans-Andean railway connects Buenos Aires with Valparaiso.


455. A stony desert is termed as ________ and sandy desert as ________.
A. Reg erg
B. Reg loess
C. Erg loess
D. Erg reg

Answer

Answer: Option A
Explanation : A stony desert is termed as Reg and sandy desert as erg.


456. Savanna is a
A. Desert
B. Forest
C. Grassland
D. Snowclad mountain

Answer

Answer: Option C
Explanation : Savanna is a Grassland. The savanna is a type of grasslands biome. The savanna is sometimes called the tropical grasslands.


457. Which or the following is mainly responsible for causing discomfort to mountaineers?
A. Steep height
B. Extreme cold
C. Lack of oxygen
D. Lack of shelter

Answer

Answer: Option C
Explanation : Lack of oxygen is mainly responsible for causing discomfort to mountaineers.


458. Along which one of the following meridians did India experience the first light of the sunrise of the new millennium?
A. 20°30’W
B. 82°30’E
C. 92°30’W
D. 92°30’E

Answer

Answer: Option D
Explanation : Along 92°30’E meridians India experience the first light of the sunrise of the new millennium. The first light of the sunrise of new millenium in India experienced at Kathal island of Andaman and Nicobar, which is located at 92°30 E meridian.


459. The winter snow-house or Eskimos is known as
A. Yurt
B. Tupic
C. Igloo
D. Tent

Answer

Answer: Option C
Explanation : The winter snow-house or Eskimos is known as Igloo. Eskimo houses are called “igloos” which are made by blocks of ice cemented together with snow. They live in Igloo during winter to protect themselves from the severe cold outside since the heat is locked up inside and make it habitable.


460. Selvas is a word denoting
A. Long narrow depression formed in the land sunk between two parallel faults
B. A southerly region of the oceans located between 45°S and N
C. The equatorial rain forest of the Amazon basin
D. The grasslands of North America

Answer

Answer: Option C
Explanation : Selvas is a word denoting the equatorial rain forest of the Amazon basin. It is a tropical rain forest, as that in the Amazon basin of South America.


461. Which of the following countries share borders with Moldova?,1. Ukraine,2. Romania,3. Belarus,Select the correct answer using the code given below
A. 1 and 2 only
B. 2 and 3 only
C. 1 and 3 only
D. 1 2 and 3

Answer

Answer: Option A
Explanation : Ukraine and Romania share borders with Moldova. The country shares its international border with its two neighboring countries; Romania and Ukraine. Ukraine’s border with the country is the longest of the two, as it is 759 miles long compared to the 423.3-mile long of the Romania-Moldova border.


462. Which of the following is/are the chief Characteristics of commercial grain farming of the middle latitude grasslands?,1. The size of farms are generally large.,2. Cultivation is highly mechanized.,3. It is a type of extensive farming.,Select the correct answer using the code given below
A. 1 and 2 only
B. 2 only
C. 1 2 and 3
D. 1 and 3 only

Answer

Answer: Option C
Explanation : The chief Characteristics of commercial grain farming of the middle latitude grasslands are The size of farms are generally large, Cultivation is highly mechanized and It is a type of extensive farming.


463. Chicago, one of world’s largest steel centres, is located on the southern tip of the lake.
A. Superior
B. Michigan
C. Huron
D. Erie

Answer

Answer: Option B
Explanation : Chicago, one of world’s largest steel centres, is located on the southern tip of Michigan lake.


464. Spot the odd item.
A. Mt Fuji
B. Mt Godwin Austin
C. Mt Vesuvius
D. Mt Etna

Answer

Answer: Option B
Explanation : Mount K2, located in Jammu and Kashmir and also known as Godwin- Austen, is the highest peak of India.


465. Cloudy nights are warmer compared to clear cloudless nights, because clouds
A. Prevent cold waves from the sky from descending on earth
B. Reflect back the heat given off by earth
C. Produce heat and radiate it towards earth
D. Absorb heat from the atmosphere and send it towards earth

Answer

Answer: Option B
Explanation : Cloudy nights are warmer compared to clear cloudless nights, because clouds Reflect back the heat given off by earth. At night cloud cover has the opposite effect. If skies are clear, heat emitted from the earth’s surface freely escapes into space, resulting in colder temperatures. However, if clouds are present, some of the heat emitted from the earth’s surface is trapped by the clouds and reemitted back towards the earth.


466. Tonle Sap is the biggest fresh water lake in South East Asia. It is located In
A. Thailand
B. Kampuchea
C. Laos
D. Vietnam

Answer

Answer: Option B
Explanation : Tonle Sap is the biggest fresh water lake in South East Asia. It is located In Kampuchea. This lake is the largest freshwater lake in South East Asia and a great source of fish for the local fisherman.


467. Which one of the following continents has the lowest birth and death rates?
A. Europe
B. Australia
C. North America
D. South America

Answer

Answer: Option B
Explanation : Australia has the lowest birth and death rates. Australia’s population growth rate has reached its lowest point in almost 10 years, largely due to a decrease in immigration and births.


468. Which is the largest country of the Africa?
A. South Africa
B. Sudan
C. Algeria
D. Egypt

Answer

Answer: Option C
Explanation : Algeria is the largest country of the Africa. Algeria is the largest country in Africa and the 10th largest country in the world.


469. Light from the nearest star reaches the earth in
A. 43 minutes
B. 4.3 seconds
C. 4.3 minutes
D. 4.3 years

Answer

Answer: Option D
Explanation : Light from the nearest star reaches the earth in 4.3 years. The closest star to Earth are three stars in the Alpha Centauri system. The two main stars are Alpha Centauri A and Alpha Centauri B, which form a binary pair. They are an average of 4.3 light-years from Earth. The third star is Proxima Centauri. It is about 4.22 light-years from Earth and is the closest star other than the sun.


470. Which continent has the highest density of population?
A. Asia
B. Africa
C. Europe
D. North America

Answer

Answer: Option C
Explanation : Europe has the highest density of population. Monaco is the most densely populated country of Europe while the region of Iceland is the least densely populated place in the continent.


471. Which of the following is the busiest of ocean trade routes?
A. Suez Canal
B. Cape of Good Hope
C. North Atlantic
D. Panama Canal

Answer

Answer: Option C
Explanation : North Atlantic is the busiest of ocean trade routes. The Atlantic Ocean is between many trading ports from different countries and that is why it it the busiest ocean.


472. Suez Canal connects
A. Red Sea and Arabian Sea
B. Red Sea and Mediterranean Sea
C. Arabian Sea and Mediterranean Sea
D. North Sea and Baltic Sea

Answer

Answer: Option B
Explanation : An artificial sea canal built by English in 19th century to shorten the trading route between Asia & Europe connects Red Sea with Mediterranean Sea.


473. Suez Canal links
A. Mediterranean Sea and Caribbean Sea
B. Black Sea and Baltic Sea
C. Red Sea and Mediterranean Sea
D. Arabian Sea and Red Sea

Answer

Answer: Option C
Explanation : Suez Canal links Red Sea and Mediterranean Sea. Suez Canal, sea-level waterway running north-south across the Isthmus of Suez in Egypt to connect the Mediterranean and the Red seas.


474. Japan’s total land is about ________ that of India.
A. One-sixth
B. One-fifth
C. One-ninth
D. One-tenth

Answer

Answer: Option C
Explanation : Japan’s total land is about One-ninth that of India.


475. Which one of the following is not an astronomical object?
A. Pulsar
B. Brittle Star
C. Black Hole
D. Quasar

Answer

Answer: Option B
Explanation : Brittle Star is not an astronomical object. Brittle star, also called serpent star, any of the 2,100 living species of marine invertebrates constituting the subclass Ophiuroidea (phylum Echinodermata).


476. The moat abundant element in sea water is
A. Sodium
B. Chlorine
C. Magnesium
D. Sulphur

Answer

Answer: Option B
Explanation : The moat abundant element in sea water is Chlorine. Seawater contains 19,000 mg of chlorine per liter.


477. Pakistan’s nuclear power plant is located at ________
A. Islamabad
B. Peshawar
C. Kahuta
D. Karachi

Answer

Answer: Option C
Explanation : Pakistan’s nuclear power plant is located at Kahuta. The primary Pakistani fissile-material production facility is located at Kahuta, employing gas centrifuge enrichment technology to produce Highly Enriched Uranium [HEU].


478. What is the average salt content in a litre of seawater?
A. 39 gm.
B. 37 gm.
C. 35 gm.
D. 32 gm.

Answer

Answer: Option C
Explanation : The average salt content is 35 gm. in a litre of seawater. This means that for every 1 litre (1000 mL) of seawater there are 35 grams of salts (mostly, but not entirely, sodium chloride) dissolved in it.


479. The USA is deficient in domestic supplies of
A. Copper
B. Lead
C. Zinc
D. Manganese

Answer

Answer: Option D
Explanation : The USA is deficient in domestic supplies of Manganese.


480. World’s largest solar power plant is located in ________
A. Kalahari Desert Africa
B. Sahara Desert Libya
C. Thar Desert India
D. Mojava Desert USA

Answer

Answer: Option D
Explanation : World’s largest solar power plant is located in Mojava Desert USA. Ivanpah, the world’s largest solar thermal plant, is to begin generating power this summer. Challenges included relocating a population of endangered desert tortoises.


481. The waterfall with the greatest meterage in the fall is
A. The Jog Falls India
B. The Victoria Falls Africa
C. The Niagara Falls North America
D. The Salto Angel Falls South America

Answer

Answer: Option D
Explanation : The waterfall with the greatest meterage in the fall is The Salto Angel Falls, South America. Angel Falls is the tallest waterfall in the world. The measured height of falls is 979 m and tallest single uninterrupted drop – 807 m high.


482. The world’s largest inland sea is ________.
A. The Black Sea
B. The Dead Sea
C. The Caspian Sea
D. The Aral Sea

Answer

Answer: Option C
Explanation : The Caspian Sea is the world’s largest inland water body. The sea covers roughly 143,200 square miles (371,000 square kilometers) and borders five countries: Iran, Russia, Kazakhstan, Turkmenistan and Azerbaijan.


483. A geographical mile is a measure of length equal to one sixtieth of a degree or one minute (1′) of latitude. It varies with latitudes, but is approximately equals
A. 7200 feet
B. 6080 feet
C. 4500 feet
D. 6000 feet

Answer

Answer: Option B
Explanation : A geographical mile is a measure of length equal to one sixtieth of a degree or one minute (1′) of latitude. It varies with latitudes, but is approximately equals to 6080 feet.


484. The Balkan Peninsula is separated from the rest of Europe by the river.
A. Danube
B. Rhine
C. Elbe
D. Weser

Answer

Answer: Option A
Explanation : The Balkan Peninsula is separated from the rest of Europe by the river Danube. Balkans, the the countries occupying the part of SE Europe lying south of the Danube and Sava Rivers, forming a peninsula bounded by the Adriatic and Ionian Seas in the west, the Aegean and Black Seas in the east, and the Mediterranean in the south.


485. The highest plateau in the world is
A. Plateau of Tibet
B. Deccan Plateau
C. Plateau of Iran
D. Spanish Plateau

Answer

Answer: Option A
Explanation : The highest plateau in the world is Plateau of Tibet. It is the highest plateau in the world with an average altitude of 4,500m, and was reputed as “the roof of the world” and “the world’s third pole”.


486. The approximate radius of the Earth is
A. 25000 km
B. 12500 km
C. 6400 km
D. 4500 km

Answer

Answer: Option C
Explanation : The approximate radius of the Earth is 6400 km.


487. Everyday the moon rises about ________ minutes later than the previous day
A. 20
B. 30
C. 60
D. 50

Answer

Answer: Option D
Explanation : Everyday the moon rises about 50 minutes later than the previous day. The Moon rises on an average 50 minutes later each day in Earth’s skies due to the difference in Earth’s rotation and Moon’s revolution.


488. The term ‘transhumance’ means
A. The practice of moving to a new clearing in the forest every few years
B. The seasonal movement of people along with their cattle in the mountains
C. Moving over to another neighbouring country
D. Moving from place to place in search of work

Answer

Answer: Option B
Explanation : The term ‘transhumance’ means the seasonal movement of people along with their cattle in the mountains. It is the seasonal movement of livestock (such as sheep) between mountain and lowland pastures either under the care of herders or in company with the owners.


489. The first steam boat was built by
A. Stevenson
B. Newton
C. James Watt
D. Robert Fulton

Answer

Answer: Option D
Explanation : The first steam boat was built by Robert Fulton. Robert Fulton (1765–1815) was an American engineer and inventor who is widely known for developing a commercially successful steamboat called Clermont. In 1807, that steamboat took passengers from New York City to Albany and back again, a round trip of 300 miles, in 62 hours.


490. The shortest air-route from Perth to London is ________
A. Perth Mumbai Rome London
B. Perth Ankara Paris London
C. Perth Aden Paris London
D. Perth Mombasa Rome London

Answer

Answer: Option C
Explanation : The shortest air-route from Perth to London is Perth, Aden, Paris and London.


491. Pearl Harbour, (the target of Japanese attack on the American fleet) is in
A. Japan
B. Philippines
C. Hawaiian Islands
D. Singapore

Answer

Answer: Option C
Explanation : Pearl Harbour, (the target of Japanese attack on the American fleet) is in Hawaiian Islands. The Japanese, concluding that war with the United States is inevitable, attempt to knock out the U.S. Pacific fleet based in the Hawaiian Islands at Pearl Harbor.


492. The coast of which of the following country are not touched by Arabian sea?
A. Saudi Arabia
B. Oman
C. Kenya
D. Iran

Answer

Answer: Option A
Explanation : Saudi Arabia has Persian Gulf at its Northern margin and Red Sea at its Southern margin.


493. In demographic features, India is sorted out in which of the following stage of population cycle?
A. Early expanding stage
B. High stationary
C. Late expanding stage
D. Declining stage

Answer

Answer: Option C
Explanation : In this stage birth rate falls & death rate declines further but population still grows as birth rate exceeds death rate e.g. India, China.


494. The Tagus river flows through
A. Damascus
B. Khartoum
C. Lisbon
D. Kabul

Answer

Answer: Option C
Explanation : The Tagus river flows through Lisbon. The longest river of the Iberian Peninsula extends for 650 miles through Spain and Portugal. The Tagus River flowing through Lisbon, Portugal at night, shortly before emptying into the Atlantic Ocean.


495. The capital of the Ontario province of Canada situated on the Lake Ontario is the port city of
A. Montreal
B. Uruguay
C. Chile
D. Paraguay

Answer

Answer: Option B
Explanation : The capital of the Ontario province of Canada situated on the Lake Ontario is the port city of Uruguay.


496. The busiest sea route is
A. The Mediterranean Red-Sea Route
B. The South Atlantic Route
C. The North Atlantic Route
D. The Pacific Route

Answer

Answer: Option C
Explanation : The busiest sea route is The North Atlantic Route. Known as the busiest shipping lane in the world, the English Channel separates England from France, and connects the North Sea and the Atlantic Ocean. The channel is 350 miles long, 20-150 miles wide, and 150-400 feet deep.


497. Which of the following is a reason for the increase in population of the developing countries?
A. Increase in birth rate due to reluctance to accept family planning measures
B. Increase in life expectancy and decrease of death rate due to medical facilities
C. Both of these
D. None of these

Answer

Answer: Option B
Explanation : Increase in life expectancy and decrease of death rate due to medical facilities is a reason for the increase in population of the developing countries.


498. The approximate distance between the Earth and the Moon is
A. 380000 km
B. 180000 km
C. 280000 km
D. 50000 km

Answer

Answer: Option A
Explanation : The approximate distance between the Earth and the Moon is 380000 km.


499. The home of spices is
A. Middle East
B. The USA
C. France
D. India

Answer

Answer: Option D
Explanation : The home of spices is India. India, known as the home of spices, boasts a long history of trading with the ancient civilisations of Rome and China. Today, Indian spices are the most sought-after globally, given their exquisite aroma, texture, taste and medicinal value. India has the largest domestic market for spices in the world.


500. Which of the statements as regards the consequences of the movement of the earth is not correct?
A. Revolution of the earth is the cause of the change of seasons
B. Rotation of the earth is the cause of days and nights
C. Rotation of the earth cause variation in the duration of days and nights
D. Rotation of the earth affects the movement of winds and ocean currents

Answer

Answer: Option C
Explanation : The change between day and night is caused by the rotation of the Earth on its axis. If the Earth did not rotate as it does, the day/night cycle would be very different or possibly even nonexistent. The changing lengths of days and nights depends on where you are on Earth and the time of year.


501. In winter the Eskimos of the Tundra region live in
A. Grass Huts
B. Trees
C. Igloos
D. Tents

Answer

Answer: Option C
Explanation : In winter the Eskimos of the Tundra region live in Igloos. Igloo is simply the Eskimo word for “house.” So, technically, most eskimos DO live in igloos, as they define them, but not as most people think of them. Mainly through the use of cartoons, people have long thought that Eskimos live in the rounded icehouses in order to keep warm in the cold climates in which they live.


502. The evergreen trees are those which remain green through out the year. This is because
A. Their leaves do not fall at all
B. Of the effect of the climatic conditions in which they have evolved
C. Their leaves fall in small numbers at intervals
D. None of the above

Answer

Answer: Option C
Explanation : The evergreen trees are those which remain green through out the year. This is because their leaves fall in small numbers at intervals. Deciduous trees are those that lose their leaves each fall and enter a stage of dormancy for the winter or summer or rainy months. The leaves that fell in the autumn or spring chill grow a new in the other season on deciduous trees looks like dead wood, repeating this cycle of rebirth and death for the lifespan of the tree.


503. Which of the following yield teak?
A. Coniferous forests
B. Tropical forests
C. Deciduous forests
D. Himalayan forests

Answer

Answer: Option C
Explanation : Deciduous forests yield teak. Teak is a tropical hardwood tree species placed in the flowering plant family Lamiaceae.


504. The urban population of the world is growing faster than Its rural population. This is mainly because of
A. High birth rate in towns
B. Rural-urban migration
C. High death rate in rural areas
D. Low death rate in towns

Answer

Answer: Option B
Explanation : The urban population of the world is growing faster than Its rural population. This is mainly because of Rural-urban migration. Rural-urban migration is the movement of people from the countryside to the city. This causes two things to happen: 1. Urban growth – towns and cities are expanding, covering a greater area of land. 2. Urbanisation – an increasing proportion of people living in towns and cities.


505. The difference in mean temperature between the hottest and coldest months in a year is known as
A. Average annual temperature
B. Mean temperature
C. Annual range of temperature
D. Isothermic range of temperature

Answer

Answer: Option C
Explanation : The difference in mean temperature between the hottest and coldest months in a year is known as Annual range of temperature. The annual range is defined as the difference between the hottest and coldest months, taking monthly mean temperatures in each case. It is given approximately by the difference between the average of the January maximum and minimum temperatures, and the corresponding average for July (6).


506. East African Valleys stretching from East Africa through Red Sea to Syria is an example of
A. River valley
B. Rift valley system
C. Glaciated valley
D. A canyon

Answer

Answer: Option B
Explanation : East African Valleys stretching from East Africa through Red Sea to Syria is an example of Rift valley system.


507. The major cause of the decrease in population of rural areas as compared to the towns is ________
A. Increased death rate due to lack of amenities
B. Lack of transport facilities
C. Low birth rates
D. Migration of people from villages to the urban areas in search of better facilities and employment

Answer

Answer: Option D
Explanation : The major cause of the decrease in population of rural areas as compared to the towns is Migration of people from villages to the urban areas in search of better facilities and employment.


508. When total land is divided by total population, we get?
A. Population Density
B. Nutritional Density
C. Agriculture Density
D. Industrial Density

Answer

Answer: Option A
Explanation : When total land is divided by total population, we get Population Density. Population density is given by population per unit area.


509. The Pacific terminal of Trans-Siberian Railway is
A. Okhotsk
B. Vladivostok
C. Leningrad
D. Moscow

Answer

Answer: Option B
Explanation : The Pacific terminal of Trans-Siberian Railway is Vladivostok. The Trans-Siberian Railway is the longest railway in the world. It was built between 1891 and 1916 to connect Moscow with the Far-East city of Vladivostok.


510. Which one of the following countries is not land-locked?
A. Afghanistan
B. Myanmar
C. Nepal
D. Switzerland

Answer

Answer: Option B
Explanation : Myanmar is located in Southeast Asia. Also called Burma, Myanmar is comprised of over 100 different nationalities. Myanmar is not a landlocked country, though it does share borders with five other Asian countries. These include Thailand, China, Bangladesh, India, and Laos.


511. Which one of the following countries is situated, geographically in America’ but politically a part of Europe?
A. Iceland
B. Canary Islands
C. Green Land
D. Cuba

Answer

Answer: Option C
Explanation : Green Land is situated, geographically in America’ but politically a part of Europe. The reason behind this classification is because geographically, Greenland is classified under North America as it lies on the North American tectonic plate while politically, the country is recognized as part of Europe.


512. ________ canal lies on one of the world’s largest of trade routes connecting Europe and Far East.
A. The Panama
B. The Suez
C. The Kiel
D. The Goth

Answer

Answer: Option B
Explanation : The Suez canal lies on one of the world’s largest of trade routes connecting Europe and Far East. The canal separates the African continent from Asia, and it provides the shortest maritime route between Europe and the lands lying around the Indian and western Pacific oceans. It is one of the world’s most heavily used shipping lanes.


513. The Famous Kruger National Park is located in
A. Saudi Arabia
B. South Africa
C. Sudan
D. Tanzania

Answer

Answer: Option B
Explanation : Kruger National Park is in South Africa. Mainly lions leopards, rhinos, elephants & buffalos are found in abundance. It is situated in N.E. South Africa.


514. Mackeral, tuna, herrings, and anchovies belong to ________ type of fish.
A. Pelagic
B. Demersal
C. Anadromous
D. Fresh water

Answer

Answer: Option A
Explanation : Mackeral, tuna, herrings, and anchovies belong to Pelagic type of fish. Pelagic fish spend the majority of their life cycle in the pelagic zone.


515. A landform composed of sediments brought by a river and found where it enters a main valley or emerges from a mountain on to a plain is called a/an
A. Meander
B. Alluvial fan
C. Oxbow lake
D. Levee

Answer

Answer: Option B
Explanation : A landform composed of sediments brought by a river and found where it enters a main valley or emerges from a mountain on to a plain is called an Alluvial fan. An alluvial fan is a triangle-shaped deposit of gravel, sand, and even smaller pieces of sediment, such as silt. This sediment is called alluvium. Alluvial fans are usually created as flowing water interacts with mountains, hills, or the steep walls of canyons.


516. What is the country through which both Equator and Tropic of Capricorn pass?
A. Brazil
B. Uruguay
C. Colombia
D. Ecuador

Answer

Answer: Option A
Explanation : Brazil is the country through which both Equator and Tropic of Capricorn pass. The Tropic of Capricorn lies at 23.5 degrees south of the equator and runs through Australia, Chile, southern Brazil (Brazil is the only country that passes through both the equator and a tropic), and northern South Africa.


517. The land-locked country in South East Asia is
A. Thailand
B. Malayasia
C. Laos
D. Vietnam

Answer

Answer: Option C
Explanation : The land-locked country in south east asia is Laos. Laos is an independent republic, and a landlocked nation in Southeast Asia, northeast of Thailand, west of Vietnam.


518. The only land-locked country in South-East Asia is
A. Laos
B. Thailand
C. Malaysia
D. Kampuchea

Answer

Answer: Option A
Explanation : The land-locked country in south east asia is Laos. Laos is an independent republic, and a landlocked nation in Southeast Asia, northeast of Thailand, west of Vietnam.


519. Panama Canal connects ________ and ________.
A. San Jose and Colon
B. Panama and San Jose
C. Panama and Colon
D. Cartagena and Colon

Answer

Answer: Option C
Explanation : Panama Canal connects Panama and Colon. The Panama Canal is a 50-mile long engineering wonder connecting the Caribbean Sea and the Pacific Ocean. Completed by the United States in 1914, it runs southeastward from Colon, through the man-made Gatun Lake, to Panama City on the Pacific side of the Isthmus of Panama.


520. Sues Canal connects ________ and ________.
A. Port Said and Suez
B. Cairo and Dumyat
C. Dumyat and Port Said
D. Suez and Dumyat

Answer

Answer: Option A
Explanation : Sues Canal connects Port Said and Suez. The canal also connects the Port Said in northeast Egypt with Port Tewfik at the city of Suez in the south. The construction of the Suez Canal was carried out between 1859 and 1869 by the Suez Canal Company, and the Suez Canal Authority owns and maintains the waterway.


521. Plains formed by the deposits brought by glacier are
A. Lacustrine plains
B. Flood plains
C. Drift plains
D. Coastal plains

Answer

Answer: Option C
Explanation : Plains formed by the deposits brought by glacier are Drift plains. The drift plains are forms when a sheet of ice becomes detached from the main body of glacier and melts in place depositing the sediments it carried.


522. ‘Ursa Major’ is the scientific name for one of the constellations of stars. Its common English name is
A. The Northern Crown
B. The Chained Lady
C. The Milky Way
D. The Great Bear

Answer

Answer: Option D
Explanation : ‘Ursa Major’ is the scientific name for one of the constellations of stars. Its common English name is The Great Bear. The constellation Ursa Major contains the group of stars commonly called the Big Dipper. The handle of the Dipper is the Great Bear’s tail and the Dipper’s cup is the Bear’s flank. The Big Dipper is not a constellation itself, but an asterism, which is a distinctive group of stars.


523. Gulf Stream, the most important current of the Atlantic Ocean, takes Its birth from the Gulf of Mexico. Its importance lies in its
A. Cooling effect
B. Warming effect
C. Area of influence
D. Influence on travel

Answer

Answer: Option B
Explanation : Gulf Stream, the most important current of the Atlantic Ocean, takes Its birth from the Gulf of Mexico. Its importance lies in its warming effect.


524. Which of these is not true of the effects of the ocean currents?
A. They affect commerce because they keep the harbours open even during winter
B. Water of the ocean is kept pure
C. Warm equatorial currents raise the temperature while the cold polar currents lower the temperature of the place through which they pass
D. They do not affect rainfall over the coast

Answer

Answer: Option D
Explanation : Ocean currents affect rainfall over the coast. Ocean currents act as conveyer belts of warm and cold water, sending heat toward the polar regions and helping tropical areas cool off.


525. The famous Matterhorn Peak is found in
A. Austria
B. Switzerland
C. France
D. Sweden

Answer

Answer: Option B
Explanation : The famous matterhorn peak is found in Switzerland. Matterhorn mountain is situated in the Pennine Alps on the border between Switzerland and Italy is one of the most famous and distinctive of all Alpine peaks.


526. Berlin is situated on the river canal
A. Danube
B. Elbe
C. Spree
D. Nile

Answer

Answer: Option C
Explanation : Berlin is situated on the river canal Spree. It flows through the city centre of Berlin to join the River Havel in Spandau, one of Berlin’s western boroughs, which itself ultimately merges with the Elbe to enter the sea in Cuxhaven, after flowing through Hamburg.


527. The region with the lowest mean winter temperature (-50º C) is found in
A. Greenland
B. Northern Canada
C. North Pole
D. North Eastern Siberia

Answer

Answer: Option B
Explanation : The region with the lowest mean winter temperature (-50º C) is found in Northern Canada.


528. Assemblage or ideas, beliefs, skills, institutions is represented by
A. Civilization
B. Culture
C. Mode of life
D. None of these

Answer

Answer: Option B
Explanation : Assemblage or ideas, beliefs, skills, institutions is represented by Culture.


529. Lakes formed due to marine deposits are
A. Crater lakes
B. Cirque lakes
C. Ox-bow lakes
D. Lagoons

Answer

Answer: Option D
Explanation : Lakes formed due to marine deposits are Lagoons. action of wind & waves may isolate lagoons along the coasts, enclosed by narrow spit of land known as lagoon lakes. Lagoonis a shallow body of water separated from a larger body of water by barrier islands or reefs.


530. The relative humidity or the atmospheric air
A. Increases with increase in temperature
B. Decreases with increase in temperature
C. Decreases with decrease in temperature
D. Is independent of temperature

Answer

Answer: Option B
Explanation : The relative humidity or the atmospheric air decreases with increase in temperature. If you increase the temperature, however, the amount of water vapor the air can hold increases, so the relative humidity decreases.


531. Which of the following countries is known as the ‘Sugar bowl of the World’ ?
A. India
B. Cuba
C. Brazil
D. U.S.
E.

Answer

Answer: Option B
Explanation : Until 1960 Cuba is the largest Sugar Exporter in world so it is been called as Sugar Bowl of the World. In 2017-18 Year Brazil leading in Suagr production 38.9 million metric tons. India produced 32.5 million metric tons.


532. Which of the following is not a minor product derived from forests?
A. Nuts and Gums
B. Tannin
C. Pitch and Turpentine
D. Pulp

Answer

Answer: Option D
Explanation : Pulp is not a minor product derived from forests. Pulp is a lignocellulosic fibrous material prepared by chemically or mechanically separating cellulose fibres from wood, fiber crops, waste paper, or rags. Many kinds of paper are made from wood with nothing else mixed into them.


533. The indigenous people living in Fiji and Papua New Guinea belong to ________
A. Mongolian group
B. Polynesian group
C. Micronesian group
D. Melanesian group

Answer

Answer: Option D
Explanation : The indigenous people living in Fiji and Papua New Guinea belong to Melanesian group. Melanesian is a subregion of Oceania extending from New Guinea island in the southwestern Pacific Ocean to the Arafura Sea, and eastward to Tonga.


534. In which of the following country has the maximum population of Muslims?
A. India
B. Pakistan
C. Indonesia
D. Saudi Arabia

Answer

Answer: Option C
Explanation : Indonesia has world’s largest Muslim population i.e. 202.9 million (approx) which is 87.2% of its total population.


535. The amount of dew is measured by
A. Pluviometer
B. Drosometer
C. Anemometer
D. Hygrometer

Answer

Answer: Option B
Explanation : The amount of dew is measured by Drosometer. A drosometer is an apparatus for measuring the quantity of dew formed in a unit of time per unit area of surface.


536. Mountain of accumulation is
A. Volcanic mountains
B. Block mountains
C. Residual mountains
D. Fold mountains

Answer

Answer: Option A
Explanation : Mountain of accumulation is Volcanic mountains. The mountains formed by the accumulation of volcanic materials are called as Volcanic Mountains or Mountains of accumulation.


537. The home or the fur-bearing animals is the
A. Coniferous forests
B. Temperate deciduous forests
C. Tundra
D. Temperate grasslands

Answer

Answer: Option A
Explanation : The home of the fur-bearing animals is the Coniferous forests. Thick coniferous forests are found on the mountains region. Fur bearing animals like squirrel, bear and fox are found.


538. Name the country which shares the benefits of the Kosi Multipurpose Project with India
A. Bangladesh
B. Pakistan
C. Nepal
D. Bhutan

Answer

Answer: Option C
Explanation : Nepal shares the benefits of the Kosi Multipurpose Project with India. Nepal agreed to the 1920 Sarda treaty, involving an exchange of territory, but not an advantageous one for Nepal. India enjoys most of the benefits of the Kosi and Gandak treaties (of 1954 and 1959), leading to the construction of dams primarily irrigating and protecting Indian lands.


539. The majority of the coal reserves found in India are of
A. Tertiary formation
B. Gondwana formation
C. Quaternary formation
D. Jurassic formation

Answer

Answer: Option B
Explanation : The majority of the coal reserves found in India are of Gondwana formation. Gondwana coal has overwhelmingly higher share (99%) in India’s coal resources and the entire coal mined in the peninsular plateau part belongs to this category.


540. Grasslands in Australia are known as the
A. Downs
B. Steppes
C. Veldt
D. Prairies

Answer

Answer: Option A
Explanation : Grasslands in Australia are known as the Downs. Temperate grasslands are areas of open grassy plains that are sparsely populated with trees. Various names of temperate grasslands include pampas, downs, and veldts.


541. Which among the following statements about the earth is not true?
A. The Earth makes one complete revolution round the Sun every 365 days 5 hours 48 minutes and 46 seconds
B. The Earth makes one complete revolution on its axis in 48 hours
C. The Earth revolves in its orbit round the Sun at a speed of 66000 miles per hour
D. The Earth rotates on its axis at an equatorial speed of more than 1000 miles per hour

Answer

Answer: Option B
Explanation : It actually takes the Earth slightly over 23 hours and 56 minutes to rotate once around this axis.


542. Which strait divides Europe from Africa?
A. Bosporus
B. Gibraltar
C. Bering
D. Dover

Answer

Answer: Option B
Explanation : Gibraltar strait divides Europe from Africa. The Strait of Gibraltar is the entry point into the Mediterranean Sea from the Atlantic Ocean. It is bordered by the continents of Africa and Europe, and the countries of Morocco, Spain, the British colony of Gibraltar, and the Spanish exclave of Ceuta.


543. The longest railway platform is ________
A. London (U.K.
B. Moscow (Russia)
C. Kharagpur (India)
D. New York (U.S.
E.)

Answer

Answer: Option C
Explanation : The longest railway platform is Kharagpur (India). Kharagpur Junction railway station has the world’s third longest railway platform with a length of 1,072.5 metres (3,519 ft).


544. People living in Sweden generally belong to
A. Mediterranean
B. Alpine
C. Nordic
D. Mongoloid

Answer

Answer: Option C
Explanation : People living in Sweden generally belong to Nordic. Swedes are a North Germanic ethnic group native to Sweden. They mostly inhabit Sweden and the other Nordic countries, in particular Finland, with a substantial diaspora in other countries, especially the United States.


545. What is a “shaft mine”?
A. Aquery
B. An open pit mine
C. Amine dug deep into the earth
D. Amine for descending into which a shaft is generally used

Answer

Answer: Option C
Explanation : A mine dug deep into the earth is a “shaft mine”. Shaft mining is a type of underground mining where shafts are pushed vertically start to finish to uncover the metals and minerals. It is additionally called shaft sinking.


546. What is a graticule?
A. The network of intersecting parallels and meridians on a map
B. The network of intersections of Equator and Prime Meridian
C. The intersection of roads
D. The intersection of Tropic of Cancer and Tropic of Capricorn

Answer

Answer: Option A
Explanation : The network of intersecting parallels and meridians on a map is known as graticule. The graticule of latitude and longitude lines is an angular measurement system. All features on the earth’s surface are located using measurements that are relative to the center of the earth. Latitude lines are parallel to each other while longitude lines converge at the poles.


547. White coal refers to
A. Hydroelectricity
B. Petroleum
C. Uranium
D. Coal

Answer

Answer: Option A
Explanation : White coal refers to Hydroelectricity. White coal was used in England to smelt lead ore from the mid-16th to the late 17th centuries. It produces more heat than green wood but less than charcoal and thus prevents lead evaporating. White coal was produced in distinctive circular pits with a channel, known as Q-pits.


548. Which country is known as the “Emerald Isle”?
A. Great Britain
B. Ireland
C. Japan
D. New Zealand

Answer

Answer: Option B
Explanation : Ireland is known as the “Emerald Isle”. Ireland’s beautiful, lush countryside is what earned the country the nickname, “The Emerald Isle.” The island is located in the Northern Atlantic Ocean, just west of Great Britain. Ireland’s only bordering country is Northern Ireland.


549. The depositional feature at the point where the river comes down from hills to plain is
A. Meander
B. Water fall
C. Levee
D. Alluvial fan

Answer

Answer: Option D
Explanation : The depositional feature at the point where the river comes down from hills to plain is Alluvial fan. They are found in the middle course of a river at the foot of slope/ mountains. When the stream moves from the higher level break into foot slope plain of low gradient, it loses its energy needed to transport much of its load.


550. In which of the following countries, is the ratio of hydroelectric generation to the total electricity generation the highest?
A. Norway
B. Sweden
C. Italy
D. Denmark

Answer

Answer: Option A
Explanation : In Norway the ratio of hydroelectric generation to the total electricity generation the highest. Norway is one of the world’s leading countries for renewable energy, and most of it comes from the water.


551. Which one of the following cities is not a former capital of the given country (Country given in the brackets)?
A. Karachi (Pakistan)
B. Auckland (New Zealand)
C. Kyoto (Japan)
D. Brisbane (Australia)

Answer

Answer: Option D
Explanation : Melbourne was the initial capital following the 1901 Federation of Australia. In 1913, the seat of national government was moved to the newly created city of Canberra, which continues to serve as the national capital.


552. Madagascar is the only home of many species of
A. Giraffe
B. Hippopotamus
C. Lemur
D. Orangutan

Answer

Answer: Option C
Explanation : Madagascar is the only home of many species of Lemur. The ring-tailed lemur (Lemur catta) is one of 105 recognized species and subspecies of lemur found only in Madagascar.


553. Western Gateway to the United States is
A. Seattle
B. San Diego
C. San Francisco
D. Los Angeles

Answer

Answer: Option C
Explanation : Western Gateway to the United States is San Francisco. San Francisco, city and port, coextensive with San Francisco county, northern California, U.S., located on a peninsula between the Pacific Ocean and San Francisco Bay. It is a cultural and financial centre of the western United States and one of the country’s most cosmopolitan cities.


554. “Limnology” refers to the study of
A. Limestone
B. Lignite of brown coal
C. Lakes and ponds
D. Laterites

Answer

Answer: Option C
Explanation : “Limnology” refers to the study of Lakes and ponds. Limnology is the study of inland waters – lakes (both freshwater and saline), reservoirs, rivers, streams, wetlands, and groundwater – as ecological systems interacting with their drainage basins and the atmosphere.


555. The New name of Burma is Myanmar and its capital is
A. Naypyitaw
B. Arakan
C. Rangoon
D. Ava

Answer

Answer: Option A
Explanation : In the north of former capital, Yangon, lies the new capital of Myanmar named as Naypyitaw.


556. The total length of the oceanic ridge systems is of the order of
A. 40000 km
B. 65000 km
C. 95000 km
D. 150000 km

Answer

Answer: Option B
Explanation : The total length of the oceanic ridge systems is of the order of 65000 km. The massive Mid-Ocean Ridge system wraps around the globe like seams on a baseball, stretching an impressive 65,000 kilometers (40,390 miles). The majority of the system is underwater, with an average water depth to the top of the ridge of 2,500 meters (8,200 feet).


Exit mobile version